Download as pdf or txt
Download as pdf or txt
You are on page 1of 214

VTU

Common to All Branches


1 & 2 Semester
Physics Cycle
2010 Scheme

Engineering
Mathematics - 1
[10MAT11]

Compiled by studyeasy.in

Download Notes, Question Banks and other Study Material


www.studyeasy.in
Branch Name: Common to all branches
SEM: 1/2
University: VTU
Syllabus: 2010

Table of Contents:
Engineering Mathematics - 1 (10MAT11):

Sl. No. Units


1 Differential Calculus 1
2 Differential Calculus 2
3 Differential Calculus 3
4 Vector Calculus
5 Integral Calculus
6 Differential Equations
7 Linear Algebra 1
8 Linear Algebra 2

Download notes for other subjects from the link below:


www.studyeasy.in

T&C - All rights reserved-studyeasy.in - 2014


Compiled by www.studyeasy.in

Engineering Mathematics – I
(10 MAT11)

LECTURE NOTES
(FOR I SEMESTER B E OF VTU)

.in
sy
y ea
ud
st

Compiled by www.studyeasy.in
Compiled by www.studyeasy.in

2011

ENGNEERING MATHEMATICS – I

Content

CHAPTER

.in
UNIT I DIFFERENTIAL CALCULUS – I

UNIT II
sy
DIFFERENTIAL CALCULUS – II
ea
UNIT III DIFFERENTIAL CALCULUS – III
y
ud
st

Compiled by www.studyeasy.in
Compiled by www.studyeasy.in

2011

UNIT - I
DIFFERENTIAL CALCULUS – I
Introduction:
The mathematical study of change like motion, growth or decay is calculus. The Rate of change of
given function is derivative or differential.

The concept of derivative is essential in day to day life. Also applicable in Engineering, Science,
Economics, Medicine etc.

.in
Successive Differentiation:
Let y = f (x) --(1) be a real valued function.
dy
The first order derivative of y denoted by sy
dx 2
or y’ or y1 or ∆1
The Second order derivative of y denoted by d yor y’’ or y2 or ∆2
dx 2
ea
Similarly differentiating the function (1) n-times, successively,
dny
the n order derivative of y exists denoted by n or yn or yn or ∆n
th
dx
The process of finding 2nd and higher order derivatives is known as Successive Differentiation.
y
ud

nth derivative of some standard functions:


st

1. y = eax
Sol : y1 = a eax
y2 = a2 eax
Differentiating Successively

yn = an eax
ie. Dn[eax] = an eax
For, a =1 Dn[ex] = ex

Compiled by www.studyeasy.in
Compiled by www.studyeasy.in

.in
sy
y ea
ud
st

Compiled by www.studyeasy.in
Compiled by www.studyeasy.in

2011

.in
sy
y ea
ud
st

Compiled by www.studyeasy.in
Compiled by www.studyeasy.in

.in
sy
y ea
ud
st

Compiled by www.studyeasy.in
Compiled by www.studyeasy.in

.in
sy
y ea
ud
st

7
Compiled by www.studyeasy.in
Compiled by www.studyeasy.in

2011

Leibnitz’s Theorem :
It provides a useful formula for computing the nth derivative of a product of two functions.
Statement : If u and v are any two functions of x with un and vn as their nth derivative. Then the nth
derivative of uv is

(uv)n = u0vn + nC1 u1vn-1 + nC2u2vn-2 + …+nCn-1un-1v1+unv0


Note : We can interchange u & v (uv)n = (vu)n,
nC = n , nC2 = n(n-1) /2! , nC3= n(n-1)(n-2) /3! …
1

1. Find the nth derivations of eax cos(bx + c)

.in
Solution: y1 = eax – b sin (bx +c) + a eax cos (b x + c), by product rule.
.i.e, y1 = eax [a cos (bx + c ) − b sin (bx + c )]
Let us put a = r cos θ , and b = r sin θ .
∴ a 2 + b 2 = r 2 and tan θ = b / a
sy
ea
.ie., r = a 2 + b 2 and θ = tan-1 (b/a)
Now, y1 = e ax [r cos θ cos(bx + c) − r sin θ sin( bx + c)]
y

Ie., y1 = r eax cos (θ + bx + c )


ud

where we have used the formula cos A cos B – sin A sin B = cos (A + B)
Differentiating again and simplifying as before,
st

y2 = r2 eax cos (2θ + bx + c ) .


Similarly y3 = r3 e ax cos (3θ + bx + c ) .
………………………………………
Thus y n = r n e ax cos(nθ + bx + c )

Where r = a 2 + b 2 and θ = tan-1 (b/a).


Thus Dn [eax cos (b x + c)]

[ [
= ( a 2 + b 2 ) n e ax cos n tan −1 (b / a ) + bx + c ]]

Compiled by www.studyeasy.in
Compiled by www.studyeasy.in

2. Find the nth derivative of log 4x2 + 8x + 3

Solution : Let y = log 4 x 2 + 8x + 3 = log (4x2 + 8x +3) ½


1
ie., y = log (4x2 + 8x +3) ∵ log xn = n log x
2
1
y= log { (2x + 3) (2x+1)}, by factorization.
2
1
∵ y= {log (2x + 3) + log (2x + 1)}
2

1 ⎧ (− 1) (n − 1)!2 n (− 1) (n − 1)!2 n ⎫
n −1 n −1

Now y n = ⎨ + ⎬
2 ⎩ (2x + 3)n (2x + 1)n ⎭

.in
⎧ 1 1 ⎫
Ie., yn = 2n-1 (-1) n-1 (n-1) ! ⎨ + n ⎬
⎩ (2 x + 3) (2 x + 1) ⎭
n

sy
3. Find the nth derivative of log 10 {(1-2x)3 (8x+1)5}
ea
Solution : Let y = log 10 {1-2x)3 (8x+1)5}
It is important to note that we have to convert the logarithm to the base e by the property:
y

log e x
log10 x =
ud

log e 10

Thus y =
1
log e 10
{
log e (1 − 2 x ) (8x + 1)
3 5
}
st

Ie., y =
1
{3 log(1 − 2x ) + 5 log(8x + 1)}
log e 10

1 ⎧ (− 1) (n − 1)!(− 2) (− 1)n −1 (n − 1)!8n ⎫


n −1 n
∴ yn = ⎨3. + 5 ⎬
loge 10 ⎩ (1 − 2x )n (8x + 1)n ⎭

Ie., yn =
(− 1) (n − 1)!2 n ⎧ 3(− 1)
n −1 n
+
5(4) ⎫
n

⎨ n ⎬
⎩ (1 − 2x ) (8x + 1) ⎭
n
log e 10

Compiled by www.studyeasy.in
Compiled by www.studyeasy.in

4. Find the nth derivative of e2x cos2 x sin x


⎡1 + cos 2 x ⎤
Solution : >> let y = e2x cos2 x sin x = e2x ⎢ ⎥ sin x
⎣ 2 ⎦
e2x
ie., y = (sin x + sin x cos 2x)
2
e2x ⎧ ⎫
⎨sin x + [sin 3x + sin (− x )]⎬
1
=
2 ⎩ 2 ⎭
e2x
= (2 sin x + sin 3x − sin x )∵ sin (-x) = -sin x
4
e2x
∴y = (sin x + sin 3x)

.in
4

Now y n = { (
1 n 2x
4
)
D e sin x + D n e 2 x sin 3x ( )}
Thus y n =
1
4
{( 5 ) en 2x
[
sy
sin n tan −1 (1 2 ) + x + ] ( 13 ) e n 2x
[
sin n tan −1 (3 2 ) + 3x ]}
ea
∴ yn =
e2x
4
{( 5 ) sin[n tan
n −1
(1 2) + x ]+ ( ) [
13 sin n tan −1 (3 2) + 3x
n
]}
y
ud

5. Find the nth derivative of e2x cos 3x


1
Solution : Let y=e2x cos3 x = e 2x. (3 cos x + cos 3x)
4
st

1
Ie., y= (3 e2x cos x + e2x cos 3x)
4
1
∴ yn = {3Dn (e2x cos x) + Dn (e2x cos 3x)}
4

yn =
1
4
{( )
n
[
3 5 e 2 x cos n tan −1 (1 2 ) + x + ] ( 13 ) e n 2x
[
cos n tan −1 (3 2 ) + 3 x ]}

Thus y n =
e2x
4
{( ) [ ] ( )
3 5 cos n tan −1 (1 2 ) + x + 13 cos n tan −1 (3 2) + 3x
n n
[ ]}

Compiled by www.studyeasy.in
Compiled by www.studyeasy.in

x2
6. Find the nth derivative of
(2x + 1)(2x + 3)
x2
Solution : y= is an improper fraction because; the degree of the
(2x + 1)(2x + 3)
numerator being 2 is equal to the degree of the denominator. Hence we must divide and
rewrite the fraction.
x2 1 4x 2
y= = . for convenience.
4 x 2 + 8x + 3 4 4 x 2 + 8x + 3

1
2

.in
4x
4 x 2 + 8x + 3
4x2 +8x +3
− 8x − 3
sy
ea
1⎡ − 8x − 3 ⎤
∴ y= 1+ 2
4 ⎣ 4 x + 8x + 3 ⎥⎦

1 1 ⎡ 8x + 3 ⎤
y

Ie., y = −
4 4 ⎢⎣ 4 x 2 + 8x + 3 ⎥⎦
ud

The algebraic fraction involved is a proper fraction.


1 ⎡ 8x + 3 ⎤
Now yn = 0 − Dn ⎢ 2 ⋅
st

4 ⎣ 4 x + 8x + 3 ⎥⎦
8x + 3 A B
Let = +
(2x + 1)(2x + 3) 2x + 1 2x + 3
Multiplying by (2x + 1) (2x + 3) we have, 8x + 3 = A (2x + 3) + B (2x + 1)
................(1)
By setting 2x + 1 = 0, 2x + 3 = 0 we get x = -1/2, x = -3/2.
Put x = -1/2 in (1): -1 -1 + A (2) ⇒ A = -1/2
Put x = -3/2 in (1): -9 = B (-2) ⇒ B = 9/2
1 ⎧ 1 ⎡ 1 ⎤ 9 n ⎡ 1 ⎤⎫
∴ y n = − ⎨− D n ⎢ + D ⎬
4 ⎩ 2 ⎣ 2x + 1⎥⎦ 2 ⎢⎣ 2x + 3 ⎥⎦ ⎭

Compiled by www.studyeasy.in
Compiled by www.studyeasy.in

1⎧
= − ⎨(− 1) ⋅
(− 1)n n!2n + 9 ⋅ (− 1)n n!2n ⎫

8⎩ (2x + 1)n +1 (2x + 3)n +1 ⎭

ie., y n =
(− 1) n!2 n ⎧
n +1
1
+
9 ⎫
⎨ n +1 ⎬
8 ⎩ (2x + 1)
n +1
(2x + 3) ⎭

x4
7. Find the nth derivative of
( x + 1) ( x + 2)

x4
Solution : y = is an improper fraction.
( x + 1) ( x + 2)
(deg of nr. = 4 > deg. of dr. = 2)

.in
On dividing x4 by x2 + 3 x + 2, We get
⎡ − 15 x − 14 ⎤ sy
y = ( x2 – 3x + 7 ) + ⎢ 2 ⎥
⎣ x + 3x + 2 ⎦
⎡ 15 x − 14 ⎤
ea
∴ yn = Dn (x2-3x+7)-Dn ⎢ 2 ⎥
⎣ x + 3x + 2 ⎦
But D = ( x2 – 3x + 7 ) = 2x – 3, D2 ( x2 – 3x + 7 ) = 2
y

D3( x2 – 3x + 7 ) = 0......... Dn ( x2 – 3x + 7 ) = 0 if n > 2


ud

⎡ 15 x + 14 ⎤
Hence yn = -Dn ⎢ ⎥
⎣ ( x + 1) ( x + 2) ⎦
st

15 x + 14 A B
Now, let Dn = +
x + 3x + 2 ( x + 1) ( x + 2)
2

=> 15x+ 14 = A(x+2) + B(x+ 1 )


Put x = - 1 ; - 1 = A ( 1 ) or A = - 1
Put x = - 2 ; - 16 = B ( - 1 ) or B = 16
⎧ ⎡ 1 ⎤ ⎡ 1 ⎤⎫
Yn = ⎨− D n ⎢ ⎥ + 16 D n ⎢ ⎬
⎩ ⎣ x + 1⎦ ⎣ x + 2 ⎥⎦ ⎭

Compiled by www.studyeasy.in
Compiled by www.studyeasy.in

(−1) n n ! 1n (−1) n n ! 1n
= − 16
( x + 1) n +1 ( x + 2) n +1
⎧ 1 16 ⎫
yn = (−1) n n ! ⎨ n +1
− ⎬n> 2
⎩ ( x + 1) ( x + 2) n +1 ⎭

8. Show that
d n ⎛ log x ⎞ (−1) n n! ⎧ 1 1 1⎫
n ⎜ ⎟= n +1 ⎨log x − 1 − − − ⎬
dx ⎝ x ⎠ x ⎩ 2 3 n⎭
log x 1 1
Solution : Let y = = log x. and let u = log x, v =
x x x
We have Leibnitz theorem,

.in
(uv)n = uvn + nC1 u1v n −1 + nC2 u 2 v n − 2 + .... + u n v …… (1)

(−1) n−1 (n − 1)!


Now, u = log x ∴ un =

1
v = ∴ vn =
( −1) n n!
xn sy
x n+1
ea
x
Using these in (1) by taking appropriate values for n we get,
⎛ log x ⎞ (−1) n n! 1 (−1) n−1 (n − 1)!
y

Dn = ⎜ ⎟ = log x.. n +1 + n .
⎝ x ⎠ x x xn
ud

n(n − 1) ⎛ 1 ⎞ (−1) n−2 (n − 2)!


+ ⎜− ⎟
1. 2 ⎝ x 2 ⎠ x n −1
st

(−1) n−1 (n − 1)! 1


+ ...... + .
xn x
(−1) n n! (−1) n−1 n!
Ie.. = log x +
x n+1 x n+1
(−1) n − 2 n! (−1) n −1 (n − 1)!
− + .... +
2 x n +1 x n +1
(−1) n−2 n! ⎡ −1 (−1) −2 (−1) −1 (n − 1)!⎤
− log x(−1) − + .... +
x n+1 ⎢⎣ 2 n1 ⎥

1 1
Note : (-1)-1 = = −1; (−1) −2 = =1
−1 (−1) 2

Compiled by www.studyeasy.in
Compiled by www.studyeasy.in

2011
(n − 1)! (n − 1)! 1
Also = =
n! n (n − 1)! n

d n ⎡ log x ⎤ ( −1) n n! ⎡ 1 1 1⎤
∴ n ⎢ ⎥ = n +1 ⎢ log x − 1 − − ... − ⎥
dx ⎣ x ⎦ x ⎣ 2 3 n⎦

9. If yn= Dn (xn logx)


Prove that yn = n yn-1+(n-1)! and hence deduce that
⎛ 1 1 1⎞
yn = n ⎜ log x + 1 + + + .... + ⎟
⎝ 2 3 n⎠
Solution : yn = Dn(xn log x) = Dn-1 {D (xn log x}
⎧ n 1 n −1 ⎫
= Dn-1 ⎨ x . + nx log x ⎬

.in
⎩ x ⎭

= Dn-1(xn-1) + nDn-1 (xn-1 log x}


sy
∴ yn = (n-1)! +nyn-1. This proves the first part.
Now Putting the values for n = 1, 2, 3...we get
ea
y1 = 0! + 1 y0 = 1 + log x = 1! (log x + 1 )
y2 = l! + 2y1 = l+2 (l + log x)
y

⎛ 1⎞
ie., y2 = 21og x + 3 = 2(log x + 3/2) = 2! ⎜ log x + 1 + ⎟
⎝ 2⎠
ud

y3 = 2! + 3y2 = 2 + 3(2 log x + 3)


⎛ 1 1⎞
ie., y3 = 61og x+ll = 6 (log x + ll/6) = 3! ⎜ log x + 1 + + ⎟
st

⎝ 2 3⎠
…………………………………………………………………………..
⎛ 1 1 1⎞
y n = n!⎜ log x + 1 + + + ... + ⎟
⎝ 2 3 n⎠

10. If y = a cos (log x) + b sin ( log x), show that


x2y2 + xy1 + y = 0. Then apply Leibnitz theorem to differentiate this result n times.
or
If y = a cos (log x) + b sin (log x ), show that
x2yn + 2 + (2n+l)xyn + l+(n2+1)yn = 0. [July-03]

Compiled by www.studyeasy.in
Compiled by www.studyeasy.in

Solution : y = a cos (log x) + b sin (log x)


Differentiate w.r.t x
1 1
∴ y1 = -a sin (log x) + b cos (log x).
x x
(we avoid quotient rule to find y2) .
=> xy1 = - a sin (log x) + b cos (log x)
Differentiating again w.r.t x we have,
1
xy2 + 1 y1 = - a cos (log x) + b sin ( log x)
x
or x2y2 + xy1 = - [ a cos (log x) + b sin (log x) ] = -y
∴ x2y2+xy1+y = 0
Now we have to differentiate this result n times.

.in
ie., Dn (x2y2) + Dn (xy1) + Dn (y) = 0
We have to employ Leibnitz theoreom for the first two terms.
Hence we have,
sy
⎧ 2 n n(n − 1) ⎫
ea
n −1
⎨ x . D ( y 2 ) + n. 2 x. D ( y 2 ) + . 2 . D n−2 ( y 2 ) )⎬
⎩ 1. 2 ⎭
{x. D n
}
( y1 ) + n. 1 . D n −1 ( y1 ) + y n = 0
y

ie., {x2yn + 2 + 2n x yn + 1 + n (n – 1)yn} + {xyn+1+nyn}+yn = 0


ud

ie., x2yn + 2 + 2n x yn + 1 + n2yn - nyn + xyn+1+nyn+yn = 0


ie., x2yn + 2 + (2n+l)xy n+l + (n2+l)yn = 0
st

11. If cos-1 (y/b ) = log (x/n)n, then show that


x2yn + 2 + (2n+l) xy n+l + 2n2yn = 0
Solution :By data, cos-1 (y/b) = n log (x/n) ∴ log(am) = m log a
y
=> = cos [n log (x/n )]
b
or y = b . cos [ n log (x/n)]
Differentiating w.r.t x we get,

Compiled by www.studyeasy.in
Compiled by www.studyeasy.in

1 1
y1 = -b sin [n log (x/n)] ⋅ n ⋅ ⋅
(x / n ) n
or xy1 = - n b sin [n log (x/n )]
Differentiating w.r.t x again we get,
1 1
xy2 + 1. y1 = - n . b cos [ n log (x/n )] n .
( x / n) n
or x (xy2+y1) = n2b cos [n log (x/n) ] =-n2y, by using (1).
or x2y2 +xyl + n2y = 0
Differentiating each term n times we have,
D(x2y2) + Dn(xy1) + n2Dn (y) = 0
Applying Leibnitz theorem to the product terms we have,

.in
⎧ 2 n( n − 1) ⎫
⎨ x y n + 2 + n. 2 x. y n +1 + . 2 . yn ⎬
⎩ 1. 2 ⎭
+ {xy n +1 + n. 1 . y n }+ n y n = 0
2
sy
ie x2yn+2 + 2 x yn+1 + n2yn + xy n+1+ nyn + n2yn=0
ea
or x2 yn+2 + (2n + l) xyn+1 + 2n2yn = 0

12. If y = sin( log (x2 + 2 x + 1)),


y

or [Feb-03]
ud

If sin-1 y = 2 log (x + 1), show that


(x+l)2yn + 2 + (2n+1)(x+1)yn+l + (n2 + 4)yn = 0
st

Solution : By data y = sin log (x2 + 2 x + 1 )


1
∴ y1 = cos log (x2 + 2 x + 1) 2x + 2
( x + 1) 2
1
ie., y1 = cos log (x2 + 2 x + 1) 2 (x + 1)
x + 2x +1
2

2 cos log( x 2 + 2 x + 1 )
ie., y1 =
( x + 1)
or (x + 1) y1 = 2 cos log (x2 + 2 x + 1 )
Differentiating w.r.t x again we get

Compiled by www.studyeasy.in
Compiled by www.studyeasy.in

1
(x+1)y2 + 1 y1 = -2 sin log (x2 + 2x + 1) . 2( x + 1)
( x + 1) 2
or (x + 1)2y2 + (x+1) y1 = -4y
or (x+l )2y2 + (x+l) y1 + 4y = 0 ,
Differentiating each term n times we have,
Dn [(x + 1)2y2] +Dn [(x+ 1)y1] + Dn [y] = 0
Applying Leibnitz theorem to the product terms we have,
⎧ n(n − 1) ⎫
⎨( x + 1) y n+ 2 + n. 2( x + 1). y n+1 +
2
.2 . y n ⎬
⎩ 1. 2 ⎭
+ {(x+l) yn + 1+n. 1 .yn} + 4yn = 0
ie., (x+l)2yn + 2 + 2n (x+1)yn+1

.in
+ n2yn-nyn + (x+l)yn+l + nyn + 4yn = 0
ie., (x+l)2yn + 2 + (2n + l) (x + l) y n+ 1 + (n2 + 4)yn = 0

( )
13. If = log x + 1 + x 2 prove that
sy
ea
(1 + x2) yn+2 + (2n + 1) xyn+1 + n2yn = 0

>> By data, y = log x + 1 + x 2 ( )


y

1 ⎧⎪ 1 ⎫⎪
ud

∴ y1 = ⎨1 + . 2 x ⎬
( x + 1 + x 2 ) ⎪⎩ 2 1+ x 2 ⎪⎭

1+ x2 + x
st

1 1
Ie., y1 =
( x + 1+ x ) 2
1+ x 2
1+ x2

or 1 + x 2 y1 = 1
Differentiating w.r.t.x again we get
1
1 + x 2 y2 + .2 x. y1 = 0
2 1+ x 2 )

or (1+x2)y2 + xy1 = 0
Now Dn [(l+x2)y2] + Dn[xy1] = 0
Applying Leibnitz theorem to each term we get,

17
Compiled by www.studyeasy.in
Compiled by www.studyeasy.in

⎧ n(n − 1) ⎫
⎨(1 + x ) y n+ 2 + n. 2 x . y n+1 +
2
.2 . y n ⎬
⎩ 1 .2 ⎭
+ [x . yn + 1+n .1 yn] = 0
Ie., (1 + x2) yn +2 + 2 n x yn + 1 + n2yn – nyn + xyn+l+ nyn = 0
or (l+x2)yn + 2 + (2n + l)xyn+1+n2yn = 0

14. If x = sin t and y = cos mt, prove that


(l-x2)yn + 2-(2n+1)xyn+l + (m2-n2)yn = 0. [Feb-04]
Solution : By data x = sin t and y = cos mt
x = sin t => t = sin-1 x and y = cos mt becomes
y = cos [ m sin-1x)

.in
Differentiating w.r.t.x we get
m
y1 = - sin (m sin-1x)
1− x2

or 1 + x 2 y1 = - m sin (m sin-1x)
sy
ea
Differentiating again w.r.f .x we get,
1 m
1 − x 2 y2 + (−2 x ) y1 = − m cos (m sin −1 x ).
y

2 1− x 2
1− x2
ud

or (1 -x2)y2-xyl = -m2y
or (1 -x2)y2 –xy1 +m2y = 0
st

Thus (1-x2)yn+2-(2n+1)xyn+1+(m2-n2)yn=0

15. If x = tan ( log y), find the value of


(l+x2)yn+1 + (2nx-l) yn+n(n-1)yn-1 [July-04]
Solution : By data x = tan(log y) => tan-1 x = log y or y = etan-1 x Since the desired relation involves
yn+1, yn and yn-1 we can find y1 and differentiate n times the result associated with y1 and y.
−1 −1 1
Consider y = e tan x
⋅ ∴ y. = e tan x

1+ x2
or (1 +x2)y1 = y
Differentiating n times we have

Compiled by www.studyeasy.in
Compiled by www.studyeasy.in

2011
Dn[(l+x2)y1]=Dn[y]
Anplying Leibnitz theorem onto L.H.S, we have,
{(l+x2)Dn(y1) + n .2x .Dn-1 (y1)
n( n − 1)
+ .2 .D n − 2 ( y1 )} = y n
1 .2
Ie., (1+x2)yn+1+2n x yn + n (n-1) yn-1-yn=0
Or (l+x2)yn + 1 + (2nx-l)yn + n(n-l)yn-1 = 0

.in
sy
y ea
ud
st

Compiled by www.studyeasy.in
Compiled by www.studyeasy.in

Continuity & Differentiability

Some Fundamental Definitions

A function f (x) is defined in the interval I, then it is said to be continuous at a point x = a


if lim f ( x) = f (a)
x→a f ( x + h) − f (a)
A function f (x) is said to be differentiable at x = a if lim = f '(a) exists a ∈I
h→0 h

⎧− x −1 ≤ x ≤ 0
Ex : Consider a function f (x) is defined in the interval [-1,1] by f (x) = x =⎨
⎩ x 0 ≤ x ≤1

It is continuous at x = 0

.in
But not differentiable at x = 0

sy
Note : If a function f (x) is differentiable then it is continuous, but converse need not be true.
Geometrically :

(1) If f (x) is Continuous at x =a means, f (x) has no breaks or jumps at the point x = a
ea
⎧−1 −1 ≤ x ≤ 0
Ex : f (x) = ⎨
⎩ x 0 < x ≤ 1
y

Is discontinuous at x=0
ud

(2) If f (x) is differentiable at x = a means, the graph of f (x) has a unique tangent at the point or graph
is smooth at x = a
st

1. Give the definitions of Continuity & Differentiability:

Solution: A function f (x) is said to be continuous at x = a, if corresponding to an arbitrary positive


number ε, however small, their exists another positive number δ such that.

⏐f (x) – f (a)⏐ < ε, where ⏐x - a⏐ < δ

It is clear from the above definition that a function f (x) is continuous at a point ‘a’.

If (i) it exists at x = a

(ii) Lt f (x) = f (a)


x→a

i.e, limiting value of the function at x = a is to the value of the function at x = a

Compiled by www.studyeasy.in
Compiled by www.studyeasy.in

Differentiability:
A function f (x) is said to be differentiable in the interval (a, b), if it is differentiable at
every point in the interval.
In Case [a,b] the function should posses derivatives at every point and at the end points a & b i.e., Rf1
(a) and Lf1 (a) exists.

2. State Rolle’s Theorem with Geometric Interpretation.


Statement: Let f (x) be a function is defined on [a,b] & it satisfies the following Conditions.

(i) f (x) is continuous in [a,b]

(ii) f (x) is differentiable in (a,b)

(iii) f (a) = f (b)


Then there exists at least a point C∈ (a,b), Here a < b such that f1 ( c ) = 0

.in
Proof:
Geometrical Interpretation of Rolle’s Theorem:

Y y = f (x)
sy
P R
ea
A B A B
y

f(a) Q S
→f(a) → f(b)
ud

O x=a x=c x=b


x =a c1 c2 c3 c4 x = b
st

Let us consider the graph of the function y = f (x) in xy – plane. A (a,.f(a)) and
B (b, f( b ) ) be the two points in the curve f (x) and a, b are the corresponding end points of A & B
respectively. Now, explained the conditions of Rolle’s theorem as follows.

(i) f (x) is continuous function in [a,b], Because from figure without breaks or jumps in
between A & B on y = f (x).

(ii) f (x) is a differentiable in (a,b), that means let us joining the points A & B, we
get a line AB.

∴ Slope of the line AB = 0 then ∃ a point C at P and also the tangent at P (or Q or R or S) is
Parallel to x –axis.

∴ Slope of the tangent at P (or Q or R or S) to be Zero even the curve y = f (x) decreases or
increases, i.e., f (x) is Constant.

Compiled by www.studyeasy.in
Compiled by www.studyeasy.in

f1 (x) = 0

∴f1 (c) = 0
(iii) The Slope of the line AB is equal to Zero, i.e., the line AB is parallel to x – axis.

∴ f (a) = f (b)

3. Verify Rolle’s Theorem for the function f (x) = x2 – 4x + 8 in the internal [1,3]

Solution: We know that every Poly nominal is continuous and differentiable for all points and hence f
(x) is continuous and differentiable in the internal [1,3].

Also f (1) = 1 – 4 + 8 = 5, f (3) = 32 – 43 + 8 = 5

Hence f (1) = f (3)

.in
Thus f (x) satisfies all the conditions of the Rolle’s Theorem. Now f1 (x) = 2x – 4 and f1 (x) = 0
⇒ 2x – 4 = 0 or x = 2. Clearly 1 < 2 < 3. Hence there exists 2t (1,3) such that f1 (2) = 0. This shows
that Rolle’s Theorem holds good for the given function f (x) in the given interval.
sy
4. Verify Rolle’s Theorem for the function f (x) = x (x + 3) e
−x
2
in the interval [-3, 0]
ea
Solution: Differentiating the given function W.r.t ‘x’ we get

⎛ 1 ⎞ −x −x
f 1 ( x) = ( x 2 + 3 x)⎜ − ⎟e 2 + (2 x + 3)e 2
y

⎝ 2⎠
ud

1 −x
= − ( x 2 − x − 6) e 2
2
∴ f (x) exists (i.e finite) for all x and hence continuous for all x.
1
st

Also f (-3) = 0, f (0) = 0 so that f (-3) = f (0) so that f (-3) = f (0). Thus f (x) satisfies all the conditions of
the Rolle’s Theorem.

Now, f1 (x) = 0

1 −x
⇒ − ( x 2 − x − 6) e 2 = 0
2
Solving this equation we get x = 3 or x = -2

Clearly –3 < -2 < 0. Hence there exists –2∈ (-3,0) such that f1 (-2) = 0

This proves that Rolle’s Theorem is true for the given function.

Compiled by www.studyeasy.in
Compiled by www.studyeasy.in

2011

5. Verify the Rolle’s Theorem for the function Sin x in [-π, π]

Solution: Let f (x) = Sin x

Clearly Sinx is continuous for all x.

Also f1 (x) = Cos x exists for all x in (-π, π) and f (-π) = Sin (-π) = 0; f (π) = Sin (π) = 0 so that f (-π) = f
(π)

Thus f (x) satisfies all the conditions of the Rolle’s Theorem .

Now f1 (x) = 0 ⇒ Cos x = 0 so that

π
X=±
2

.in
π
Both these values lie in (-π,π). These exists C = ±
2
Such that ( c ) = 0
f1

Hence Rolle’s theorem is vertified.


sy
ea
6. Discuss the applicability of Rolle’s Theorem for the function f (x) = ІxІ in [-1,1].

Solution: Now f (x) = ⏐x⏐= x for 0 ≤ x ≤ 1


y

-x for –1 ≤ x ≤ 0
ud

f (x) being a linear function is continuous for all x in [-1, 1]. f(x) is differentiable for all x in (-
1,1) except at x = 0. Therefore Rolle’s Theorem does not hold good for the function f (x) in [-1,1].
st

Graph of this function is shown in figure. From which we observe that we cannot draw a tangent to the
curve at any point in (-1,1) parallel to the x – axis.

Y
y = ⏐x⏐

x
-1 0 1

Compiled by www.studyeasy.in
Compiled by www.studyeasy.in

Exercise:
7. Verify Rolle’s Theorem for the following functions in the given intervals.

a) x2 – 6x + 8 in [2,4]

b) (x – a)3 (x – b)3 in [a,b]

⎧ x 2 + ab ⎫
c) log ⎨ ⎬ in [a,b]
⎩ ( a + b) x ⎭
8. Find whether Rolle’s Theorem is applicable to the following functions. Justify your
answer.

a) f (x) = ⏐x – 1 ⏐ in [0,2]

b) f (x) = tan x in [0, π] .

.in
9. State & prove Lagrange’s (1st) Mean Value Theorem with Geometric meaning.

Statement: Let f (x) be a function of x such that

(i) If is continuous in [a,b]


sy
ea
(ii) If is differentiable in (a,b)

Then there exists atleast a point (or value) C∈ (a,b) such that.
y

f (b ) − f ( a )
f 1 (c ) =
b−a
ud

i.e., f (b) = f (a) + (b – a) f1 (c)


Proof:
st

[b,f(b)]

[a,f(a)]

x
a c b

Compiled by www.studyeasy.in
Compiled by www.studyeasy.in

Define a function g (x) so that g (x) = f (x) – Ax ---------- (1)

Where A is a Constant to be determined.

So that g (a) = g (b)

Now, g (a) = f (a) – Aa

G (b) = f (b) – Ab

∴ g (a) = g (b) ⇒ f (a) – Aa = f (b) – Ab.

f (b) − f ( a )
i.e., A = ---------------- (2)
b−a
Now, g (x) is continuous in [a,b] as rhs of (1) is continuous in [a,b]

.in
G(x) is differentiable in (a,b) as r.h.s of (1) is differentiable in (a,b).

Further g (a) = g (b), because of the choice oif A.


sy
Thus g (x) satisfies the conditions of the Rolle’s Theorem.

∴ These exists a value x = c sothat a < c < b at which g1 ( c ) = 0


ea
∴Differentiate (1) W.r.t ‘x’ we get
y

g1 (x) = f 1 (x) – A
ud

∴ g1 ( c ) = f1 ( c )- A (∵ x =c)
st

⇒ f 1 ( c ) - A = 0 (∵g1 ( c ) = 0)
∴f1 ( c ) = A -------------- (3)

From (2) and (3) we get

f (b ) − f ( a )
f 1 (c ) = (or) f (b) = f (a) + (b – a) f1 (c) For a < c < b
b−a

Corollary: Put b – a = h
i.e., b = a + h and c = a + θ h
Where 0 < θ < 1
Substituting in f (b) = f (a) + (b – a) f1 ( c )

∴ f (a + h) = f (a) + h f (a + θ h), where 0 < θ < 1.

Compiled by www.studyeasy.in
Compiled by www.studyeasy.in

Geometrical Interpretation:-
Since y = f (x) is continuous in [a,b], it has a graph as shown in the figure below,
At x = a, y = f (a)
At x = b, y = f (b)

Y B Y B
P
α

.in
A C A
Q

0 a c b
x
sy
0 Figure (ii)
X
ea
Figure (i)

Slope of the line joining the points A (a,f(a)) and B ( (b,f (b))
y

f (b) − f (b)
ud

Is (∴ Slope = m = tan θ)
b−a

= tan α
st

Where α is the angle mode by the line AB with x – axis

= Slope of the tangent at x = c

= f1 ( c ), where a < c < b

Geometrically, it means that there exists at least are value of x = c, where a < c < b at which the
tangent will be parallel to the line joining the end points at x = a & x = b.

Note: These can be more than are value at which the tangents are parallel to the line joining points A &
B (from Fig (ii)).

Compiled by www.studyeasy.in
Compiled by www.studyeasy.in

10. Verify Lagrange’s Mean value theorem for f(x) = (x – 1) (x – 2) (x – 3) in [0,4].

Solution: Clearly given function is continuous in [0,4] and differentiable in (0,4), because f (x) is in
polynomial.

f (x) = (x – 1) (x – 2) (x – 3)

f (x) = x3 – 6x2 + 11x – 6

and f (0) = 03 – 6(0)2 + 11 (0) – 6 = -6

f(4) = 43 – 6 (4)2 + 11 (4) – 6 = 6


Differentiate f (x) W.r.t x, we get

F1 (x) = 3x2 – 6x + 11

.in
Let x = c, f1 ( c) = 3c2 – 6c + 11

By Lagrange’s Mean value theorem, we have

f (b) − f (a) f (4) − f (0)


f 1 (c ) = =
sy
b−a (4 − 0)
ea
6 − ( − 6)
= =3
4
y

∴3c2 – 6c + 11 = 3
ud

⇒ 3c2 – 6c + 8 = 0
st

Solving this equation, we get

2
C=2± ∈ (0,4)
3
Hence the function is verified.

11. Verify the Lagrange’s Mean value theorem for f (x) = logx in [1,e].

Solution: Now Logx is continuous for all x > 0 and hence [1,e].
1
Also f 1 ( x) = which exists for all x in (1,e)
x
Hence f (x) is differentiable in (1,e)

∴by Lagrange’s Mean Value theorem, we get

Compiled by www.studyeasy.in
Compiled by www.studyeasy.in

Loge − Log1 1 1 1
= ⇒ =
e −1 c e −1 c
⇒C=e–1

⇒1<e- <2<e

Since e∈ (2,3)
∴ So that c = e – 1 lies between 1 & e

Hence the Theorem.

12. Find θ for f (x) = Lx2 + mx + n by Lagrange’s Mean Value theorem.

Solution: f (x) = Lx2 + mx + n

∴f1 (x) = 2 Lx + m

.in
We have f (a + h) = f (a) + hf1 (a + θ h)

Or f (a + h) – f (a) = hf1 (a + θ h) sy
i.e., { (a + h)2 + m (a + h) + n} – { a2 + ma + n} = h {2 (a + θ h) + m}
ea
Comparing the Co-efficient of h2, we get
y

1
1 = 2θ ∴θ = ∈ (0,1)
2
ud

Exercise:
⎡ π⎤
13. Verify the Lagrange’s Mean Value theorem for f (x) = Sin2x in ⎢0, ⎥
⎣ 2⎦
st

b−a b−a
14. Prove that, < tan-1 b – tan-1 a < if 0 < a < b and reduce that
1+ b 2
1+ a2
π 3 4 π 1
+ < tan −1 < +
4 25 3 4 6
2 Sinx ⎛ π⎞
15. Show that < < 1 in ⎜ 0, ⎟
π x ⎝ 2⎠

b−a b−a
16. Prove that < Sin −1b − Sin −1 a < Where a < b. Hence reduce
1− a2 1− b2
π 1 1 π 1
− < Sin −1 < − .
6 2 3 4 6 15

Compiled by www.studyeasy.in
Compiled by www.studyeasy.in

2011

17. State & prove Cauchy’s Mean Value Theorem with Geometric meaning.

Proof: The ratio of the increments of two functions called Cauchy’s Theorem.
Statement: Let g (x) and f (x) be two functions of x such that,

(i) Both f (x) and g (x) are continuous in [a,b]

(ii) Both f (x) and g (x) are differentiable in (a,b)

(iii) g1 (x) ≠ 0 for any x ε (a,b)

These three exists at least are value x = c ε (a,b) at which

f 1 (c) f (b) − f (a)


=
g 1 (c) g (b) − g (a )

.in
Proof: Define a function,

φ (x) = f (x) – A. g (x) sy


------------------ (1)

So that φ (a) = φ (b) and A is a Constant to be determined.


ea
Now, φ (a) = f (a) – A g (a)
y

φ (b) = f (b) – A. g (b)


ud

∴ f (a) – A g (a) = f (b) – A. g (b)


f (b) − f (b)
⇒ A= -------------------- (2)
g (b) − g (a )
st

Now, φ is continuous in [a,b] as r.h.s of (1) is continuous in [a,b] and φ (x) is differentiable in
(a,b) as r.h.s of (1) is differentiable in [a,b].

Also φ (a) = φ (b)

Hence all the conditions of Rolle’s Theorem are satisfied then there exists a value x = c ∈ (a,b)
such that φ1 ( c ) = 0.

Now, Differentiating (1) W.r.t x, we get

φ1 (x) = f1 (x) – A.g1 (x)

at x = c ∈ (a,b)

Compiled by www.studyeasy.in
Compiled by www.studyeasy.in

∴φ1 ( c ) = f1 ( c ) – A g1 (c)

0 = f1 ( c) – A g1 ( c ) (∵ g1 (x) ≠ 0)

f 1 (c )
⇒ A= --------------- (3)
g 1 (c )
Substituting (3) in (2), we get
f 1 (c) f (b) − f (a)
= , where a < c < b
g 1 (c) g (b) − g (a )
Hence the proof.

18. Verify Cauchy’s Mean Value Theorem for the function f (x) = x2 + 3, g (x) = x3 + 1 in [1,3]

Solution: Here f (x) = x2 + 3, g (x) = x3 + 1

.in
Both f (x) and g (x) are Polynomial in x. Hence they are continuous and differentiable for all x
and in particular in [1,3]

Now, f1 (x) = 2x, g1 (x) = 3x2


sy
ea
Also g 1 (x) ≠ 0 for all x ∈ (1,3)

Hence f (x) and g (x) satisfy all the conditions of the cauchy’s mean value theorem.
y

Therefore
ud

f (3) − f (1) f 1 (c)


= , for some c : 1 < c < 3
g (3) − g (1) g 1 (c)
st

12 − 4 26
i.e., =
28 − 2 3c 2

2 1 13 1
i.e., = ⇒C = =2
13 3c 6 6

1
Clearly C = 2 lies between 1 and 3.
6

Hence Cauchy’s theorem holds good for the given function.

Compiled by www.studyeasy.in
Compiled by www.studyeasy.in

2011

⎡ π⎤
19. Verify Cauchy’s Mean Value Theorem for the functions f (x) Sin x, g (x) = Cos x in ⎢0, ⎥
⎣ 2⎦

Solution: Here f (x) = Sin x, g (x) = Cos x so that

f1 (x) = Cos x ,g1 (x) = - Sinx

⎡ π⎤
(
Clearly both f (x) and g (x) are continuous in ⎢0, ⎥ , and differentiable in 0, π
⎣ 2⎦ 2
)
(
Also g1 (x) = -Sin x ≠ 0 for all x ∈ 0, π
2
)
∴ From cauchy’s mean value theorem we obtain

⎛π ⎞

.in
f ⎜ ⎟ − f (0)
⎝2⎠ f 1 (c ) π
= 1 for some C : 0 < C <
⎛π ⎞ g (c ) 2
g ⎜ ⎟ − g (0)
⎝2⎠ sy
1 − 0 Cosc
i.e., = i.e., -1 = - Cot c (or) Cot c = 1
ea
0 − 1 − Sinc

π π π
∴C= , clearly C = lies between 0 and
y

4 4 2
ud

Thus Cauchy’s Theorem is verified.

Exercises:
st

20. Find C by Cauchy’s Mean Value Theorem for

a) f (x) = ex, g (x) = e-x in [0,1]

b) f (x) = x2, g (x) = x in [2,3]

21. Verify Cauchy’s Mean Value theorem for

⎡ 1 ⎤
a) f (x) = tan-1 x, g (x) = x in ⎢ ,1⎥
⎣ 3 ⎦
1
b) f(x) = log x, g (x) = in [1,e]
x

31
Compiled by www.studyeasy.in
Compiled by www.studyeasy.in

2011

Generalized Mean Value Theorem:

22. State Taylor’s Theorem and hence obtain Maclaurin’s expansion (series)

Statement: If f (x) and its first (n – 1) derivatives are continuous in [a,b] and its nth derivative exists in
(a,b) then

(b − a) 2 11 (b − a ) n −1 n-1 (b − a) n n
f(b) = f(a) + (b – a) (a) +
f1 f (a) + ---------+ f (a) + f (c )
2! (n − 1)! n!

Where a < c < b

Remainder in Taylor’s Theorem:

We have

.in
( x − a) 2 11 ( x − a) n −1 n-1
f (x) = f (a) + (x – a) f 1 (a) + f (a) + --------- + f (a) +
2! (n − 1)!
( x − a) n n
∠n
f [a + (x – a) θ}
sy
ea
f (x) = S n (x) + R n (x)

( x − a) n n
Where R n (x) = f [a + (x – a) θ] is called the Largrange’s form of the Remainder.
y

n!
ud

xn n
Where a = 0, R n (x) = f (θx), 0 < θ < 1
n!
Taylor’s and Maclaurin’s Series:
st

We have f (x) = Sn (x) + Rn (x)

∴ Lim[ f ( x ) − S n ( x )] = Lim Rn ( x )
n →∞ n→∞

If Lim Rn ( x) = 0thenf ( x) Lim S n ( x )


n →∞ n →∞

Thus Lim S n ( x) converges and its sum is f (x).


n →∞

This implies that f (x) can be expressed as an infinite series.

( x − a) 2 11
i.e., f (x) = f (a) + (x – a) f 1 (a) + f (a) + ---------- to ∞
2!
This is called Taylor’s Series.

Compiled by www.studyeasy.in
Compiled by www.studyeasy.in

Putting a = 0, in the above series, we get


( x) 2 11
F (x) = f (0) + x f 1 (0) + f (0) + --------- to ∞
2!
This is called Maclaurin’s Series. This can also denoted as
( x) 2 ( x) n
Y = y (0) + x y 1 (0) + y 2 (0) + -------- + y n (0) ---------- to ∞
2! n!
Where y = f (x), y1 = f 1 (x), -------------- y n = f n (x)

23. By using Taylor’s Theorem expand the function e x in ascending powers of (x – 1)

Solution: The Taylor’s Theorem for the function f (x) is ascending powers of (x – a) is
( x − a ) 2 11
f (x) = f (a) + (x – a) f 1 (a) + f (a) + ------------ (1)
∠2
Here f (x) = e x and a = 1
f 1 (x) = e x ⇒ f 1 (a) = e

.in
f 11 (x) = e x ⇒ f 11 (a) = e

∴ (1) becomes
e = e + (x – 1) e +
x
( x − 1) 2
2
e + -------------
sy
( x − 1) 2
ea
= e { 1 + (x – 1) + + --------}
2

24. By using Taylor’s Theorem expand log sinx in ascending powers of (x – 3)


y

Solution: f (x) = Log Sin x, a = 3 and f (3) = log sin3


ud

Cosx
Now f 1 (x) = = Cotx, f 1 (3) = Cot3
Sinx
st

f 11 (x) = - Cosec 2x, f 11 (3) = - Cosec 23


f 111 (x) = - 2Cosecx (-Cosecx Cotx) = 2Cosec 3x Cotx

∴ f 111 (3) = 2Cosec 33 Cot3


( x − a) 2 11 ( x − a)3 111
∴ f (x) = f (a) + (x – a) f 1 (a) + f (a) + f (a) + ------------
2! 3!

( x − 3) 2 11 ( x − 3)3 111
∴Log Sinx = f (3) + (x – 3) f 1(3) + f (3) + f (3) + -----------
2! 3!

( x − 3) 2 ( x − 3)3
= logsin3 + (x – 3) Cot3 + (-Cosec 3) +
2 2 Cosec 33Cot3 + ---
2! 3!

Compiled by www.studyeasy.in
Compiled by www.studyeasy.in

2011

Exercise:
⎛ π⎞
25. Expand Sinx is ascending powers of ⎜ x − ⎟
⎝ 2⎠
26. Express tan x in powers of (x – 1) up to the term containing (x – 1) 3
–1

27. Apply Taylor’s Theorem to prove

⎡ h 2 h3 ⎤
e x + h = e x ⎢1 + h + + + − − − − − − − − −⎥
⎣ 2! 3! ⎦

Problems on Maclaurin’s Expansion:

.in
28. Expand the log (1 + x) as a power series by using Maclaurin’s theorem.

Solution: Here f (x) = log (1 + x), Hence f (0) = log 1 = 0

We know that
sy
d n −1 ⎧ 1 ⎫
ea
dn
f n ( x) = {log(1 + x )} = ⎨ ⎬
dx n dx n −1 ⎩1 + x ⎭
y

(−1) n −1 (n − 1)!
= , n = 1,2,---------
(1 + n) n
ud

Hence f n (0) = (-1) n-1 (n – 1) !


st

f 1 (0) = 1, f 11 (0) = -1, f 111 (0) = 3!, f 1v (0) = - 3!

Substituting these values in

x 2 11 xn n
f (x) = f (0) + x f (0) +
1 f (0) + -------- + f (0) + ------------
2! n!

x2 x3 x4
∴ log (1+x) = 0 + x . 1 + (-1) + 2! + - 3! + ---------------
2! 3! 4!

x2 x3 x4
=x- + - + ---------
2 3 4
This series is called Logarithmic Series.

Compiled by www.studyeasy.in
Compiled by www.studyeasy.in

2011

29. Expand tan –1 x by using Macluarin’s Theorem up to the term containing x 5


Solution: let y = tan –1 x, Hence y (0) = 0
1
We find that y 1 = which gives y 1 (0) = 1
1+ x2
Further y 1 (1 + x 2) = 1, Differentiating we get

Y 1 . 2x + (1 + x 2) y 2 = 0 (or) (1 + x2) y 2 + 2xy 1 = 0


Hence y 2 (0) = 0
Taking n th derivative an both sides by using Leibniz’s Theorem, we get
n( n − 1)
(1 + x 2) y n + 2 + n . 2xy n +1 + . 2. y n + 2xy n – 1 + n.2.y n = 0
1 .2
i.e., (1 + x 2) y n + 2 + 2 (n +1) x y n + 1 + n (n + 1) y n = 0

Substituting x = 0, we get, y n + 2 (0) = -n (n + 1) y n (0)

.in
For n = 1, we get y 3 (0) = - 2y 1 (0) = - 2

For n = 2, we get y 4 (0) = - 2 .3.y 2 (0) = 0


sy
For n = 3, we get y 5 (0) = - 3.4.y 3 (0) = 24
Using the formula
ea
x2 x3
Y = y (0) + x y 1 (0) + y 2 (0) + y 3 (0) + ---------
2! 3!
x3 x5
We get tan –1 x = x - + - -------------
y

3 5
Exercise:
ud

30. Using Maclaurin’s Theorem prove the following:


x2 5x 4
st

a) Secx = 1 + + + --------
2! 4!
x 3 3x 5
b) Sin –1 x = x + + + --------------
6 40
x3
c) e x Cos x = 1 + x - + ---------------
3

d) Expand e ax Cos bx by Maclaurin’s Theorem as far as the term containing x 3

Compiled by www.studyeasy.in
Compiled by www.studyeasy.in

2011

Exercise : Verify Rolle’s Theorem for

⎡ π 5π ⎤
(i) f ( x) = e x (sin x − cos x) in ⎢ , ⎥ ,
⎣4 4 ⎦

(ii) f ( x) = x( x − 2)e x / 2 in [0,2]

sin 2 x ⎡ π 5π ⎤
(iii) f ( x) = in ⎢ , ⎥ .
e2 x ⎣4 4 ⎦

Exercise : Verify the Lagrange’s Mean Value Theorem for

.in
⎡ 1⎤
(i) f ( x ) = x ( x − 1)( x − 2) in ⎢ 0, ⎥
⎣ 2⎦

(ii) f ( x) = Tan −1x in [ 0,1]


sy
ea
Exercise : Verify the Cauchy’s Mean Value Theorem for
y

1 ⎡1 ⎤
(i) f ( x ) = x and g ( x) = in ⎢ ,1⎥
ud

x ⎣4 ⎦

1
in [ a, b]
1
(ii) f ( x) = and g ( x) =
st

2 x
x

(iii) f ( x ) = Sin x and g ( x ) = Cos x in [ a, b]

Compiled by www.studyeasy.in
Compiled by www.studyeasy.in

2011

UNIT – II

DIFFERENTIAL CALCULUS-II

Give different types of Indeterminate Forms.

If f (x) and g (x) be two functions such that Lim f (x ) and Lim g (x ) both exists, then
x→ a x→ a

f ( x) Lim f ( x)
Lim = x→a
x→a g ( x) Lim g ( x)
x→a

.in
If Lim f (x ) = 0 and Lim g (x ) = 0 then
x→ a x→ a

f ( x) 0
Lim = Which do not have any definite value, such an expression is called
sy
x →a g ( x) 0

indeterminate form. The other indeterminate forms are ,0 × ∞, ∞ − ∞ , 00, ∞0 and 1 ∞

ea
1. State & prove L’ Hospital’s Theorem (rule) for Indeterminate Forms.
y

0 ∞
L’Hospital rule is applicable when the given expression is of the form or
ud

0 ∞

Statement: Let f (x) and g (x) be two functions such that


st

(1) Lim f (x ) = 0 and Lim g (x ) = 0


x→ a x→ a

(2) f1 (a) and g1 (a) exist and g1 (a) ≠ 0

f ( x) Lim f 1 ( x)
Then Lim = x →a
x→a g ( x) Lim g 1 ( x)
x→a

⎡ 1 x ⎤
= Lim ⎢
f ( x) g ⎥ 0
Proof: Now Lim , which takes the indeterminate form . Hence applying the
x →a g ( x) x→a ⎢ 1 ⎥ 0
⎢⎣ f ( x) ⎥⎦
L’ Hospitals theorem, we get

Compiled by www.studyeasy.in
Compiled by www.studyeasy.in

2011
− g 1 ( x) 2
f ( x) [ g ( x)]2 g 1 ( x) ⎡ f ( x) ⎤
Lim = Lim = Lim ⎢ ⎥
x →a − f 1 ( x) x →a f 1 ( x) g ( x)
x →a g ( x)
2
⎣ ⎦
[ f ( x)]

2
⎡ g 1 ( x) ⎤ ⎡ f ( x) ⎤
= ⎢ Lim 1 ⎥ ⎢ Lim ⎥
⎣ x →a f ( x) ⎦ ⎣ x→ a g ( x) ⎦

f ( x)
If Lim ≠ 0and ≠ ∞ then
x→a g ( x)

⎡ g 1 ( x) ⎤ ⎡ f ( x) ⎤
1 = ⎢ Lim 1 ⎥ ⎢ Lim ⎥
⎣ x→a f ( x) ⎦ ⎣ x→a g ( x) ⎦

.in
f ( x) f 1 ( x)
i.e Lim = Lim 1
x→a g ( x) x →a g ( x)

f ( x)
sy
If Lim = 0 or ∞ the above theorem still holds good.
ea
x→ a g ( x)

Sinx 0
2. Evaluate Lim = form
y

x→a x 0
ud

Solution: Apply L’Hospital rule, we get

Cosx Cosθ 1
Lim = = =1
st

x→a 1 1 1

Sinx
∴ Lim =1
x →a x

log Sinx
3. Evaluate Lim
x→a Cotx
log Sinx log Sin0 log 0 − ∞
Solution: Lim = = = form
x→a Cotx Cot 0 ∞ ∞

Apply L’ Hospital rule

− Cosce 2 x
= Lim
x→a − 2Co sec xCo sec xCotx

Compiled by www.studyeasy.in
Compiled by www.studyeasy.in

1
= Lim =0
x→a 2Cotx

log Sinx
∴ Lim =0
x→a Cotx

Exercise: 1

Evaluate

tan x
a) Lim
x →0 x

1
b) Lim(1 + x) x
x →0

.in
a x −1
c) Lim
x →∞ x

xn − an
sy
d ) Lim
ea
x →0 x−a

4. Explain ∞ - ∞ and 0 × ∞ Forms:


y

Solution: Suppose Lim f (x) = 0 and Lim g (x) = ∞ in this case


ud

x→ a x→a

0 ∞
Lim f (x) - g(x) = 0 × ∞, reduce this to or form
x→ a 0 ∞
st

⎧ ⎫
⎪ f ( x) ⎪ 0
Let Lim[ f ( x).g ( x)] = Lim ⎨ ⎬ = form
⎪ 1 g ( x) ⎪ 0
x→a x→a
⎩ ⎭
⎧ ⎫
⎪ g ( x) ⎪ ∞
Or Lim[ f ( x).g ( x)] = Lim ⎨ ⎬= form
x→a x→a 1
⎪ f ( x) ⎪ ∞
⎩ ⎭
L’ Hospitals rule can be applied in either case to get the limit.

Suppose Lim f (x) = ∞ and Lim g (x) = ∞ in this case Lim[ f ( x).g ( x)] = ∞ − ∞ form, reduce
x→ a x→a x→a

0 ∞
this or form and then apply L’Hospitals rule to get the limit
0 ∞

Compiled by www.studyeasy.in
Compiled by www.studyeasy.in

2011
⎧ 1 log(1 + x) ⎫
5. Evaluate Lim ⎨ − ⎬

x →0 x x2 ⎭

⎧ 1 log(1 + x) ⎫
Solution: Given Lim ⎨ − ⎬ = ∞ - ∞ form

x →0 x x2 ⎭

⎧ x − log(1 + x ⎫ 0
∴ Required limit = Lim ⎨ ⎬ = form
x →0
⎩ x2 ⎭ 0

Apply L’Hospital rule.

1
1−
= Lim 1+ x
x →0 2x

.in
x
= Lim 1 + x = Lim
1 1
=
x →0 2x x → 0 2(1 + x) 2 sy
⎧1 ⎫
ea
6. Evaluate Lim ⎨ − Cotx ⎬

x →0 x

Solution: Given limit is ∞ - ∞ form at x = 0. Hence we have


y
ud

⎧ 1 Cosx ⎫
Required limit = Lim ⎨ − ⎬

x→0 x Sinx ⎭
st

⎛ Sinx − xCosx ⎞ ⎛ 0 ⎞
= Lim⎜ ⎟ = ⎜ ⎟ form
x →0
⎝ xSinx ⎠ ⎝0⎠

Apply L’ Hospital’s rule

Cosx − Cosx + xSinx


= Lim
x →0 xCosx + Sinx

xSinx ⎛0⎞
= Lim = ⎜ ⎟ form
x →0 xCosx + Sinx ⎝ 0 ⎠

Apply L’ Hospitals rule

Compiled by www.studyeasy.in
Compiled by www.studyeasy.in

2011
xCosx + Sinx
= Lim
x →0 Cosx − xSinx + Cosx

0 0
= = =0
2−0 2

7. Evaluate Lim tan x log x


x →0

Solution: Given limit is (0 × - ∞) form at x = 0

log x ⎡ − ∞ ⎤
∴ Required limit = Lim = form
x →0 Cotx ⎢⎣ ∞ ⎥⎦

Apply L’ Hospitals rule

.in
= Lim x
x →0 − Co sec 2 x

= Lim
x →0
− Sin 2 x ⎛ 0 ⎞
x ⎝0⎠
⎜ ⎟ form
sy
ea
Apply L Hospitals rule

− 2 SinxCosx
= Lim =0
y

x→0 1
ud

⎛ πx ⎞
8. Evaluate Lim Sec⎜ ⎟. log x
x →1
⎝ 2⎠
Solution: Given limit is (∞ × 0) form at x = 1
st

log x 0
∴ Required limit = Lim form
x →1 πx 0
Cos
2

Apply L’ Hospitals rule

1
x 2
= Lim =−
x →1 πx π π
− Sin .
2 2

Compiled by www.studyeasy.in
Compiled by www.studyeasy.in

2011

Exercise: 2

Evaluate

⎛ x 1 ⎞
a) Lim⎜⎜ − ⎟
x →1 x − 1
⎝ log x ⎟⎠

⎛a ⎛ x ⎞⎞
b) Lim⎜⎜ − Cot ⎜ ⎟ ⎟⎟

x →0 x
⎝ a ⎠⎠

⎛ 1 ⎞
c) Lim⎜ Secx − ⎟
x→ ⎝
π 1 − Sinx ⎠
2

d) Lim⎛⎜ a x − 1⎞⎟ x
1

.in
x →∞ ⎝ ⎠
9. Explain Indeterminate Forms 0 0 , 1∞ , ∞ 0 , 0 ∞

Solution: At x = a, [ f ( x)] sy
takes the indeterminate form
g ( x)

(i) 0 0 if Lim f(x) = 0 and Lim g (x) = 0


ea
x→a x→a

(ii) 1∞ if Lim f(x) = 1 and Lim g (x) = ∞


x→a x→a
y

(iii) ∞ 0 if Lim f(x) = ∞ and Lim g(x) = 0 and


ud

x→a x→a

(iv) 0 ∞ if Lim f(x) = 0 and Lim f (x) = ∞


x→a x→a
st

In all these cases the following method is adopted to evaluate Lim [ f ( x)]
g ( x)
x→a

Let L = Lim [ f ( x)] so that


g ( x)
x→a

Log L = Lim [g(x) log f (x)] = 0 × ∞


x→a

0 ∞
Reducing this to or and applying L’ Hospitals rule, we get Log L = a Or
0 ∞
L = ea

Compiled by www.studyeasy.in
Compiled by www.studyeasy.in

2011

10. Evaluate Lim x Sinx


x→0

Solution: let L = Lim x Sinx ⇒ 00 form at x = 0


x→0

Hence Log L = Lim Sinx log x ⇒ 0 × ∞ form


x→0

log x log x ⎛ ∞ ⎞
∴ LogL = Lim = Lim ⎜ ⎟ form
x → 0 Coscex ∞
x→0 1 ⎝ ⎠
Sinx

Apply L’ Hospital rule,

( )
1
x − Sinx. tan x 0

.in
= Lim = Lim form
x →0 − CoscexCotx x→0 x 0

Apply L’ Hospitals rule we get


sy
ea
sin xSec 2 x − Cosx tan x
= Lim = −∞
x →0 1
y

1 1
ud

∴ LogL = −∞ ⇒ L = e −∞ = ∞
= =0
e ∞

∴L=0
st

1
11. Evaluate Lim( x) 1− x
x →1

1 −x
Solution: let L = Lim( x) 1
is 1∞ form
x →1

⎛ 1 ⎞ ⎛0⎞
∴ LogL = Lim⎜ log x ⎟ ≡ ⎜ ⎟ form
x →1 1 − x
⎝ ⎠ ⎝0⎠

Apply L’ Hospitals rule

1
1
= Lim x = Lim = −1
x →1 − 1 x →1 − x

Compiled by www.studyeasy.in
Compiled by www.studyeasy.in

2011
∴ Log L = -1

1
⇒ L = e −1 =
e
1
⎛ tan x ⎞ x2
12. Evaluate Lim⎜ ⎟
x →0
⎝ x ⎠
1
⎛ tan x ⎞ x2
Solution: let L = Lim⎜ ⎟ ≡ 1∞ form
x →0
⎝ x ⎠

⎧1 ⎛ tan x ⎞⎫
∴ LogL = Lim ⎨ 2 log⎜ ⎟⎬ ≡ (∞ × 0) form

x →0 x
⎝ x ⎠⎭

⎧ log tan x
⎪ (x
)⎫⎪ ≡ ⎛ 0 ⎞ form

.in
∴ LogL = Lim ⎨ ⎬ ⎜ ⎟
⎝0⎠
x →0 2
⎪⎩ x ⎪⎭

Apply L’ Hospitals rule

⎧ xSec 2 x − tan x ⎫
sy
⎪⎪ 1 ⎪⎪
ea
= Lim ⎨ x2 ⎬
2x
⎪ tan x
x→0

⎩⎪ x ⎭⎪
y

1 ⎡ xSec 2 x − tan x ⎤ ⎛ 0 ⎞
ud

LogL = Lim ⎢ ⎥ = ⎜ ⎟ form


2 x →0 ⎣ x3 ⎦ ⎝0⎠

Apply L’ Hospital rule, we get


st

1 Sec 2 x + 2 xSec 2 x tan x − Sec 2 x


= Lim
2 x →0 3x 2

1 ⎛ tan x ⎞
= Lim( Sec 2 x)⎜ ⎟
3 x → 0
⎝ x ⎠

1
Log L =
3
1
∴L = e 3

Compiled by www.studyeasy.in
Compiled by www.studyeasy.in

2011

Exercise: 3
Evaluate the following limits.

⎛ πx ⎞
tan ⎜ ⎟
⎛ x⎞ ⎝ 2a ⎠
a) Lim( Secx) Cotx b) Lim⎜ 2 − ⎟
x →0 x→a
⎝ a⎠

1
(1 + x) x − e ∞
c) Lim d) Lim(Cosax) x2
x→0 x x →0

e x − e− x − 2 log(1 + x) log(1 + x3 )
(i ) lim (ii ) lim
x →0 x sin x x →0 sin 3 x

.in
1 + sin x − cos x + log(1 − x ) log sin x
(iii ) lim (iv) lim
x →0 x tan 2 x π π
x→
( x − )2

cosh x + log(1 − x) − 1 + x
sy 2

sin x sin −1 x
2
ea
(v) lim (vi ) lim
x →0 x2 π x2
x→
2

e2 x − (1 + x) 2
y

(vii ) lim
x → 0 x log(1 + x )
ud

Evaluate the following limits.


st

⎛ x⎞
(i ) lim ⎜ 2 − ⎟ cot( x − a ) (ii ) lim ( cos ecx − cot x )
x→a
⎝ a⎠ x →0

⎡ π ⎤ ⎛ 1 ⎞
(iii ) lim ⎢ x tan x − sec x ⎥ (iv) lim ⎜ cot 2 x − 2 ⎟
π
x→ ⎣ 2 ⎦ x →0
⎝ x ⎠
2

⎡1 1 ⎤
(v) lim ⎢ 2 − (vi ) lim [ 2 x tan x − π sec x ]

x →0 x x tan x ⎥⎦ x→
π
2
1
⎛ 1 + cos x ⎞ x2
b
2
(i ) lim(cos ax) x (ii ) lim ⎜ ⎟
x →0 x →0
⎝ 2 ⎠

Compiled by www.studyeasy.in
Compiled by www.studyeasy.in

2011
1
⎛ sin x ⎞
1
x2
2 log(1− x )
(iii ) lim(1 − x ) (iv) lim ⎜ ⎟
x →1 x →0
⎝ x ⎠

(iv ) lim (1 + sin x )


cot x
(v ) lim(sin x ) tan x
x →0 x →0

(viii ) lim ( tan x )


2 tan 2 x
(vii ) lim(cos x)cos ec x
x →0 π
x→
4
1
ax + 1 x ⎛ ax + bx + cx ⎞ x
(ix) lim( ) ( x) lim ⎜ ⎟
x →∞ ax − 1 x →0
⎝ 3 ⎠

Evaluate the following limits.

.in
1
⎛ 1 + cos x ⎞ x2
b
2
(i ) lim(cos ax) x (ii ) lim ⎜ ⎟
x →0 x →0
⎝ 2 ⎠

(iii ) lim(1 − x )
1
2 log(1− x )
(iv) lim ⎜
sy
⎛ sin x ⎞ x2
1


ea
x →1 x →0
⎝ x ⎠

(iv ) lim (1 + sin x )


cot x
(v ) lim(sin x ) tan x
y

x →0 x →0
ud

(viii ) lim ( tan x )


2 tan 2 x
(vii ) lim(cos x)cos ec x
x →0 π
x→
4
1
ax + 1 x ⎛ ax + bx + cx ⎞ x
st

(ix) lim( ) ( x) lim ⎜ ⎟


x →∞ ax − 1 x →0
⎝ 3 ⎠

Compiled by www.studyeasy.in
Compiled by www.studyeasy.in

2011

Polar Curves
If we traverse in a hill section where the road is not straight, we often see caution boards hairpin
bend ahead, sharp bend ahead etc. This gives an indication of the difference in the amount of bending
of a road at various points which is the curvature at various points. In this chapter we discuss about the
curvature, radius of curvature etc.

Consider a point P in the xy-Plane.


r = length of OP= radial distance
θ = Polar angle
( r, θ)→ Polar co-ordinates
Let r = f (θ) be the polar curve

r = x2 + y2 , θ = tan−1 y ( x) −−−−−−(1)

.in
x = r Cos θ y = r Sin θ
sy
Relation (1) enables us to find the polar co-ordinates
( r, θ) when the Cartesian co-ordinates ( x, y) are known.
ea
Expression for arc length in Cartesian form.

Proof: Let P (x,y) and Q (x + δx, Y + δy) be two neighboring points on the graph of the
function y = f (x). So that they are at length S and S + δs measured from a fixed
y

point A on the curve.


ud

Y = f (x)
st

Q
T (Tangent)
δs δy
A δx

N O δx X


From figure, PQ = δS ,

AP = S
∠TPR = ψ and PR =δx, RQ = δy

Compiled by www.studyeasy.in
Compiled by www.studyeasy.in

2011

∴ Arc PQ = δS

From ∆le PQR, we have

[Chord PQ]2 = PR2 + QR2

[Chord PQ]2 = (δx)2 + (δy)2 (∵ from figure)

When Q is very close to point P, the length of arc PQ is equal to the length of Chord PQ.

i.e arc PQ = Chord PQ = δs

∴ (δs)2 = (δx)2 + (δy)2 -------- (1)

÷ (δx)2, we get

.in
⎛ δs ⎞ ⎛ δy ⎞
2 2

⎜ ⎟ = 1+ ⎜ ⎟
⎝ δx ⎠ ⎝ δx ⎠

When Q → P along the curve, δx → 0, δs → 0


sy
ea
⎛ δs ⎞ ⎛ δy ⎞
2 2

∴ Lim⎜ ⎟ = 1 + Lim⎜ ⎟
δx → 0 δx δx → 0 δ x
⎝ ⎠ ⎝ ⎠
y
ud

2 2
⎛ ds ⎞ ⎛ dy ⎞
i.e., ⎜ ⎟ = 1 + ⎜ ⎟
⎝ dx ⎠ ⎝ dx ⎠
st

2
ds ⎛ dy ⎞
∴ = 1+ ⎜ ⎟ --------------(2)
dx ⎝ dx ⎠

Similarly, dividing (1) by δy and taking the limit as δy → 0, we get

2
ds ⎛ dx ⎞
= 1 + ⎜⎜ ⎟⎟ ------------(3)
dy ⎝ dy ⎠

Compiled by www.studyeasy.in
Compiled by www.studyeasy.in

2011

ds ds
Expressions for &
dx dy
Trace a tangent to the curve at the point P, it makes an angle ψ with the x – axis. From ∆le PRT, we
have

dy
tan ψ =
dx
Q
Equation (2) becomes,
Y

Tangent
T T
A P ψ R ψ

.in
P R

ψ
O
syM N X
ea
ds
= 1 + tan 2 ψ
dx
y

= Sec 2ψ = secψ
ud

ds dx
∴ = Sec ψ ( Or) = Cos ψ
dx ds
st

and equation (3) becomes

ds
= 1 + Cot 2ψ = Co sec 2 ψ = Cosec ψ
dy

ds dy
∴ = Cosecψ or = Sin ψ
dy ds

Derive an expression for arc length in parametric form.

Solution: Let the equation of the curve in Parametric from be x = f (t) and y = g (t).

We have,

Compiled by www.studyeasy.in
Compiled by www.studyeasy.in

2011

(δs)2 = (δx)2 + (δy)2

÷ by (δt)2, we get

⎛ δs ⎞ ⎛ δx ⎞ ⎛ δy ⎞
2 2 2

⎜ ⎟ =⎜ ⎟ +⎜ ⎟
⎝ δt ⎠ ⎝ δt ⎠ ⎝ δt ⎠

Taking the limit as δt → 0 on both sides, we get

⎛ δs ⎞ ⎛ δx ⎞ ⎛ δy ⎞
2 2 2

Lim⎜ ⎟ = Lim⎜ ⎟ + Lim⎜ ⎟


δt → 0 ⎝ δt ⎠ δt → 0 ⎝ δt ⎠ δt → 0 ⎝ δ t ⎠

2 2 2
⎛ ds ⎞ ⎛ dx ⎞ ⎛ dy ⎞
∴⎜ ⎟ = ⎜ ⎟ + ⎜ ⎟

.in
⎝ dt ⎠ ⎝ dt ⎠ ⎝ dt ⎠

2 2
ds ⎛ dx ⎞ ⎛ dy ⎞
(Or) = ⎜ ⎟ +⎜ ⎟ ------------ (4)
sy
dt ⎝ dt ⎠ ⎝ dt ⎠

Derive an expression for arc length in Cartesian form.


ea
Solution: Let P (r,θ) and Q (r + δr, θ + δθ) be two neighboring points on the graph of the function
r = f (θ). So that they are at lengths S and s + δs from a fixed point A on the curve.
y

∴ PQ = (S + δs) – s = δs
ud

Draw PN ⊥ OQ
st

From ∆le OPN,

Q
Tangent
N

r + δr P
φ
r
δθ
θ
X
O

Compiled by www.studyeasy.in
Compiled by www.studyeasy.in

2011

PN
= Sinδθ
OP

PN
i.e., = Sin δθ (or) PN = r sinδθ
r

ON
and = Cos δθ
OP

ON
i.e = Cos δθ (or) ON = r Cos δθ
r

When Q is very close to P, the length of arc PQ as equal to δs, where δs as the length of chord
PQ.

.in
In ∆le PQN,

(PQ)2 = (PN)2 + (QN)2 sy


but PN = r sin δθ (∵Sin δθ ≈ δθ)
ea
And QN = OQ – ON

= (r + δr) – r Cos δθ
y
ud

= r + δr – r ( ∵Cos δθ ≈ 1)

∴ QN = δr
st

And (PQ)2 = (PN)2 + (QN)201

(δS)2 + (rδθ)2 + (δr)2 ---------------- (5)

÷ by (δθ)2 we get

⎛ δS ⎞ ⎛ δr ⎞
2 2

⎜ ⎟ = r +⎜ ⎟
2

⎝ δθ ⎠ ⎝ δθ ⎠
When Q → P along the curve δθ → 0 as δS → 0 and δr → 0

Compiled by www.studyeasy.in
Compiled by www.studyeasy.in

⎛ δS ⎞ ⎛ δr ⎞
2 2

Lim⎜ ⎟ = r 2 + Lim⎜ ⎟
δθ → 0 ⎝ δθ ⎠ δθ → 0 ⎝ δθ ⎠

2 2
⎛ ds ⎞ ⎛ dr ⎞
i.e ⎜ ⎟ = r +⎜ ⎟
2

⎝ dθ ⎠ ⎝ dθ ⎠

2
ds ⎛ dr ⎞
i.e., = r2 + ⎜ ⎟ --------------- (6)
dθ ⎝ dθ ⎠

Similarly equation (5) ÷ by (δr)2 and taking the limits as δr → 0 we get

⎛ δS ⎞ ⎛ δθ ⎞
2 2

Lim⎜ ⎟ = Lim r 2 ⎜ ⎟ + 1
δr → 0 δ r
⎝ ⎠ δr → 0 ⎝ δr ⎠

.in
⎛ dθ ⎞
2 2
⎛ ds ⎞
i.e ⎜ ⎟ = r 2 ⎜ ⎟ +1
⎝ dr ⎠ ⎝ dr ⎠


ds
=
⎛ dθ ⎞
2
sy
r ⎜ ⎟ +1 --------------- (7)
2
ea
dr ⎝ dr ⎠

Note: Angle between Tangent and Radius Vector:-


y
ud

We have,


Tan φ = r
st

dr

Sinφ dθ
i.e., =r
Cosφ dr

dθ ds
=r .
ds dr

Sinφ r dθ
= ds
Cosφ dr
ds

dθ dr
∴Sinφ = r and Cos φ =
ds ds

Compiled by www.studyeasy.in
Compiled by www.studyeasy.in

ds ds
Find and for the following curves:-
dx dy
⎛ x⎞
1) y = C Cos h ⎜ ⎟
⎝c⎠

⎛x⎞
Solution: y = C Cos h ⎜ ⎟
⎝c⎠
Differentiating y w.r.t x. we get

dy ⎛x⎞
= Sin h ⎜ ⎟
dx ⎝c⎠

.in
2
ds ⎛ dy ⎞
∴ = 1+ ⎜ ⎟
dx ⎝ dx ⎠

≡ 1 + sinh 2 x ( c) = Cosh 2 x( c) sy
ea
ds ⎛ x⎞
= Cosh ⎜ ⎟
dx ⎝c⎠
y

Again Differentiating y w.r.t y we get


ud

⎛ x ⎞ 1 dx
1 = C Sin h ⎜ ⎟
⎝ c ⎠ C dy
st

dx ⎛x⎞
i.e = Cosech ⎜ ⎟
dy ⎝c⎠


ds
dy
= 1 + Co sec h 2 x ( )
c

2) x3 = ay2

Solution : x3 = ay2

Differentiating w.r.t y and x separately we get

Compiled by www.studyeasy.in
Compiled by www.studyeasy.in

2011
dx dy
3x2 = 2 ay and 3 x2 = 2 ay
dy dx

dx 2ay dy 3 x 2
i.e., = and =
dy 3 x 2 dx 2ay

We know that

2
ds ⎛ dy ⎞
= 1+ ⎜ ⎟
dx ⎝ dx ⎠

2
⎛ 3x 2 ⎞
= 1 ⎜⎜
+ ⎟⎟
⎝ 2 ay ⎠

.in
9x3 x 9ay 2 x
= 1+ = 1 +
4a 2 y 2 4a 2 y 2

ds 9x
sy
= 1+ and
ea
dx 4a
2
⎛ dx ⎞
2
ds ⎛ 2ay ⎞
= 1 + ⎜⎜ ⎟⎟ = 1 + ⎜ 2 ⎟
dy ⎝ dy ⎠ ⎝ 3x ⎠
y
ud

1 1
ds ⎛ 4a 2 y 2 ⎞ 2 ⎛ 4a ⎞ 2
= ⎜1 + ⎟⎟ = ⎜1 + ⎟ (∴x3 = ay2)
dy ⎜⎝ 9x 4 ⎠ ⎝ 9x ⎠
st

3. y = log cos x
Solution . y= log cos x

Differentiating w.r.t x and y separately, we get

dy 1
= ( – Sin x) = - tan x
dx Cosx

dy
i.e., = - tan x
dx

1 dx
and 1 = (- Sin x)
Cosx dy

Compiled by www.studyeasy.in
Compiled by www.studyeasy.in

2011

dx dx
i.e., 1 = - tan x (Or) = - Cot x
dy dy

We have

2
⎛ dx ⎞
2
ds ⎛ dy ⎞ ds
= 1+ ⎜ ⎟ and = 1 + ⎜⎜ ⎟⎟
dx ⎝ dx ⎠ dy ⎝ dy ⎠

ds ds
∴ = 1 + tan 2 x & = 1 + Cot 2 x
dx dy

= Sec 2 x
1
ds ds

.in
= Sec x and = (1 + Cot 2 x) 2
dx dy

ds
Find
dt
for the following Curves:-
1. x = a (Cos t + t Sin t), y = a (Sin t = t Cos t)
sy
ea
2. x = a Sec t , y = b tan t

(
3. x = a Cost + log tan t ), y = a Sin t
y

2
Solution of 1
ud

Given x = a (Cos t + t Sin t), y = a (Sin t – t Cos t )

Differentiating x & y W.r.t ‘t’, we get


st

dx
= a (-Sin t + Sin t + t Cos t)
dt

dx
= a t cos t
dt
dy
and = a (Cos t – Cos t + t Sin t)
dt
dy
= at Sin t
dt
2 2
ds ⎛ dx ⎞ ⎛ dy ⎞
∴ = ⎜ ⎟ +⎜ ⎟
dt ⎝ dt ⎠ ⎝ dt ⎠

Compiled by www.studyeasy.in
Compiled by www.studyeasy.in

2011

= a 2 t 2 Cos 2 t + a 2 t 2 Sin 2 t
ds
= at
dt
Solution of 2
x = a Sec t, x = b tan t
dx dy
∴ = a Sec t tan t, = b Sec2t
dt dt
We have
2 2
ds ⎛ dx ⎞ ⎛ dy ⎞
∴ = ⎜ ⎟ +⎜ ⎟
dt ⎝ dt ⎠ ⎝ dt ⎠
1

.in
= (a2Sec2 t tan2t + b2 Sec4t) 2
1
= [a2 Sec2 t (Sec2t – 1) + b2 Sec4 t] 2

= [a2 Sec4 t– a2 Sec2 t+ b2 Sec4 t]


1
2
sy
ea
1
ds
= [(a2 + b2) Sec4 t – a2 Sec2t] 2
dt
Solution of 3
y

x = a (Cos t + log tan t ), y = a Sin t


ud

2
Differentiating x and y w.r.t ‘t’ we get
⎛ ⎞
st

dx
=a ⎜ − S int + 1 Sec 2 t . 1 ⎟ , dy = a Cos t
dt ⎜⎜ tan t 2 2 ⎟⎟ dt
⎝ 2 ⎠
2 2
ds ⎛ dx ⎞ ⎛ dy ⎞
∴ = ⎜ ⎟ +⎜ ⎟
dt ⎝ dt ⎠ ⎝ dt ⎠
1
⎡ ⎡ Sec 2 t ⎤
2
⎤ 2
⎡ a 2 S int Sec 2 t a 2 Sec 4 t ⎤
⎢ 2⎢
= a − S int + 2 ⎥ + a Cos t ⎥
2 2
, = ⎢a −
2 2 + 2⎥
⎢ ⎢ 2 tan ⎥
t ⎥ ⎢ tan t 4 tan 2 t

⎢⎣ ⎣ 2⎦ ⎥⎦ ⎣ 2 2⎦

= a Cot t

Compiled by www.studyeasy.in
Compiled by www.studyeasy.in

2011
ds dr
Find and for the following curves
dθ dθ

1. r = a (1 – Cos θ)
2 .r2 = a2 Cos 2θ
3. r = a eθCot α
Solution of 1
r = a (1 – Cos θ)
Differentiating r w.r.t θ we get
dr
= a Sin θ

Hence

.in
1

ds ⎧⎪ 2 ⎛ dr ⎞ ⎫⎪ 2
2

= ⎨r + ⎜ ⎟ ⎬
dθ ⎪⎩ ⎝ dθ ⎠ ⎪⎭
1
sy
= {a2 (1 – Cos θ)2 + a2 Sin2 θ} 2
ea
1
= {a2 (1 – 2 Cos θ + Cos2θ) + a2 Sin2 θ} 2
y

1
= {a2 – 2a2 Cos θ + a2} 2
ud

1
= {2a2 – 2a2 Cos θ} 2
st

1
= a{ 2(1 – Cos θ)} 2
1
= a {2 (2 Sin2 θ )} 2
2

= 2 a Sin θ
ds
dθ 2
1

ds ⎧⎪ ⎛ dθ ⎞ ⎫⎪ 2
2

And = ⎨1 + r 2 ⎜ ⎟ ⎬
dr ⎪⎩ ⎝ dr ⎠ ⎪⎭
1
⎧ a (1 − Cosθ ) ⎫
2 2 2
= ⎨1 + ⎬
⎩ a 2 Sin 2θ ⎭

Compiled by www.studyeasy.in
Compiled by www.studyeasy.in

2011
1
⎧ Sin 2θ + (1 − Cosθ ) 2 ⎫ 2
= ⎨ ⎬
⎩ Sin 2θ ⎭

⎛θ ⎞
2Sin⎜ ⎟ 2Sin θ
{2(1 − Cosθ )}
1

=
2
= ⎝ 2⎠
= 2
Sinθ Sinθ 2 Sin θ Cos θ
2 2
ds 1
=
dr Cos θ
2
Solution of 2
r2 = a2 Cos 2θ
Differentiating W.r.t ‘θ’ we get

.in
dr
2r = -a2 Sin 2θ . 2

r
dr

= -a2 Sin 2θ sy
a 2 Sin2 βθ
ea
dr
=
dθ r
Hence
y

1
⎧⎪ 2 ⎛ dr ⎞ 2 ⎫⎪ 2
ud

ds
= ⎨r + ⎜ ⎟ ⎬
dθ ⎪⎩ ⎝ dθ ⎠ ⎪⎭
1
st

⎧⎪ ⎛ − a 2 Sin 2θ ⎞
2
⎫⎪ 2
= ⎨r 2 + ⎜⎜ ⎟⎟ ⎬
⎪⎩ ⎝ r ⎠ ⎪⎭
1
⎧ a 4 Sin 2 2θ ⎫ 2
= ⎨r 2 + ⎬
⎩ r2 ⎭
1
⎧ a 4 Sin 2θ ⎫ 2
= ⎨a 2 Cos 2θ + 2 ⎬
⎩ a Cos 2θ ⎭
1
⎧ a 4 Cos 2 2θ + a 4 Sin 2 2θ ⎫ 2
= ⎨ ⎬
⎩ a 2 Cos 2θ ⎭

Compiled by www.studyeasy.in
Compiled by www.studyeasy.in

2011
1
⎧ a4 ⎫2 a
= ⎨ 2 ⎬ =
⎩ a Cos 2θ ⎭ Cos 2θ
1
ds ⎧ a ⎫2 a2 4
Or = ⎨ 2⎬ = (∵r2 = a2 Cos 2θ)
dθ ⎩ ⎭
r r

1
ds ⎧⎪ ⎛ dθ ⎞ ⎫⎪
2 2

And = ⎨1 + r 2 ⎜ ⎟ ⎬
dr ⎪⎩ ⎝ dr ⎠ ⎪⎭
1
⎧⎪ ⎛ r ⎞ ⎫⎪
2 2

= ⎨1 + r 2 ⎜ ⎟ ⎬
⎪⎩ ⎝ − a Sin 2θ ⎠ ⎪⎭
2

1
⎧ ⎫

.in
r4 2
= ⎨1 + 4 2 ⎬
⎩ a Sin 2θ ⎭
1
⎧ a 4 Sin 2 2θ + a 4 Cos 2 2θ ⎫
=⎨
⎩ a 4 sin 2 2θ


2
sy
ea
1
⎧ a4 ⎫ 2 1
=⎨ 4 ⎬ = = Cosec 2θ
⎩ a sin 2θ ⎭ Sin 2θ
2
y

Solution 3
ud

r = aeθCot α , here α is constant


Differentiating w.r.t ‘θ’ we get
st

dr
= a eθCot α. Cot α

Hence
1

ds ⎧⎪ 2 ⎛ dr ⎞ 2 ⎫⎪ 2
= ⎨r + ⎜ ⎟ ⎬
dθ ⎪⎩ ⎝ dθ ⎠ ⎪⎭
1
= {a2 e2θCotα + a2 e2θCot α cot2 α} 2

1
=a eθCot α {1 + Cot2α} 2
1

=a eθCot α {Cosce2α} 2

Compiled by www.studyeasy.in
Compiled by www.studyeasy.in

2011
ds
= a eθCot α Cosce α

1
ds ⎧⎪ 2 ⎛ dθ ⎞ ⎪

2 2

and = ⎨1 + r ⎜ ⎟ ⎬
dr ⎪⎩ ⎝ dr ⎠ ⎪⎭
1
⎧ 1 ⎫ 2
= ⎨1 + a 2 e 2θCotα 2 2θCotαCot 2α ⎬
⎩ a e ⎭
1

= {1 + tan2 α} 2

= {Sec2 α} 2 = Sec α
Exercises:

.in
ds ds
Find and to the following curves. sy
dr dθ
1. rn = an Cos nθ
ea
2. r (1 + Cos θ) = a
3. rθ = a
y

Note:
ud


We have Sin φ = r and
ds
dr
Cos φ =
st

ds
1
dr
∴ = Cosφ = (1 – Sin2φ) 2
ds
1
⎡ p ⎤ 2 2
= ⎢1 − 2 ⎥ Since P = r Sin φ.
⎣ r ⎦

dr r 2 − p2
=
ds r

Compiled by www.studyeasy.in
Compiled by www.studyeasy.in

2011

ds r
∴ =
dr r − p2
2

r+δr
P(x,y)
φ OR = P
r
O X
P

.in
OR p
R Sin φ = =
OP r

sy ∴ P = r Sin φ

⎛ dθ ⎞
Prove that with usual notations tan φ = r ⎜ ⎟
ea
⎝ dr ⎠
y
ud
st

Let P (r, θ) be any point on the curve r = f (θ)



∴ X O P = θ and OP = r
Let PL be the tangent to the curve at P subtending an angle ψ with the positive direction of
the initial line (x – axis) and φ be the angle between the radius vector OP and the tangent PL.

That is O P L = φ
From the figure we have

Compiled by www.studyeasy.in
Compiled by www.studyeasy.in

2011
ψ = φ+θ
(Recall from geometry that an exterior angle is equal to the sum of the interior opposite angles)
⇒ tan ψ = tan (φ + θ )

tan φ + tan φ
or tan ψ …(1)
1 - tan φ tan θ
Let (x, y) be the Cartesian coordinates of P so that we have,
X = r cos θ , y = r sin θ
Since r is a function of θ , we can as well regard these as parametric equations in terms of θ .
We also know from the geometrical meaning of the derivative that
dy
tan ψ = = slope of the tangent PL
dx

.in
dy dx
ie., tan ψ = since x and y are function of θ
dθ dθ
d
ie., tan ψ = dθ =
sy
(r sin θ ) r cos θ + r′ sin θ
where r′ =
dr
d
(r cos θ ) - r sin θ + r′ cos θ dθ
ea

Dividing both the numerator and denominator by r′ cos θ we have,
r cos θ r′ sin θ
y

+
tan ψ = r′ cos θ r′ cos θ
ud

− r sin θ r′ cos θ
+
r′ cos θ r′ cos θ
st

r
+ tan θ
Or tan ψ = r ′ …(2)
r
1 - . tan θ
r′
Comparing equations (1) and (2) we get
r r ⎛ dθ ⎞
tan φ = = or tanφ = r⎜ ⎟
r ′ ⎛ dr ⎞ ⎝ dr ⎠
⎜ ⎟
⎝ dθ ⎠

Compiled by www.studyeasy.in
Compiled by www.studyeasy.in

2
1 1 1 ⎛ dr ⎞
Prove with usual notations = 2+ 4⎜ ⎟ or
p 2
r r ⎝ dθ ⎠
2
1 ⎛ du ⎞ 1
= u 2 + ⎜ ⎟ where u =
⎝ dθ ⎠
2
p r

.in
Proof : sy
ea
Let O be the pole and OL be the initial line. Let P (r, θ ) be any point on the curve and hence

we have OP = r and L O P = θ
y
ud

Draw ON = p (say) perpendicular from the pole on the tangent at P and let φ be the angle
made by the radius vector with the tangent.
st

∧ ∧
From the figure O N P = 90 L O P = θ
Now from the right angled triangle ONP
ON
sin φ =
OP
P
ie., sin φ = or p = r sin φ
r
we have p = r sin φ …(1)
1 dr
and cot φ = …(2)
r dθ
Squaring equation (1) and taking the reciprocal we get,

Compiled by www.studyeasy.in
Compiled by www.studyeasy.in

2011
1 1 1 1 1
= 2 . ie., = 2 cosec 2 φ
p 2
r sin 2 φ p 2
r

Or
1
p 2
=
1
r 2
(
1 + cot 2 φ )
Now using (2) we get,

1 1 ⎡ 1 ⎛ dr ⎞
2⎤
= 2 ⎢1 + 2 ⎜ ⎟ ⎥
⎝ dθ ⎠
2
p r ⎢⎣ r ⎥⎦
2
1 1 1 ⎛ dr ⎞
Or = 2 + 4 ⎜ ⎟ …(3)
⎝ dθ ⎠
2
p r r
1
Further, let =u
r

.in
Differentiating w.r.t. θ we get,
2 2
1 ⎛ dr ⎞ du 1 ⎛ dr ⎞ ⎛ du ⎞
− 2 ⎜ ⎟=
r ⎝ dθ ⎠ dθ
⇒ 4 ⎜ ⎟ = ⎜ ⎟ , by squaring
r ⎝ dθ ⎠ ⎝ dθ ⎠
sy
Thus (3) now becomes
ea
2
1 ⎛ du ⎞
= u2 + ⎜ ⎟ …(4)
⎝ dθ ⎠
2
p
y
ud

1. Find the angle of intersection of the curves:


r = a (1 + cos θ ) & r = b (1 - cos θ )
st

Solution : r = a (1 + cos θ) : r = b(1 – cos θ)


⇒ log r = log a + log (1 + cos θ) : log r = log b + log (1 – cos θ)

Differentiating these w.r.t. θ we get


1 dr - sin θ 1 dr sin θ
= 0 + : = 0 +
r dθ 1 + cos θ r dθ 1 − cos θ
− 2 sin (θ/2 ) cos (θ/2 ) 2 sin (θ/2 ) cos (θ/2 )
cot φ1 = : cot φ 2 =
2 cos 2 (θ/2 ) 2 sin 2 (θ/2 )
ie., cot φ1 = - tan (θ /2 ) = cot (π/2 + θ /2 ) : cot φ 2 = cot (θ /2 )

⇒ φ 1 = π /2 + θ /2 : φ 2 = θ/2

Compiled by www.studyeasy.in
Compiled by www.studyeasy.in

2011
∴ angle of intersection = φ1 - φ 2 = π/2 + θ/2 - θ/2 = π/2

Hence the curves intersect orthogonally.


2. S.T. the curves r = a (1 + sin θ ) & r = a (1 - sin θ ) cut each other orthogonally

Solution : log r = log a + log (1 + sin θ) : log r = log a + log (1 – sin θ)


Differentiating these w.r.t θ we get
1 dr cos θ 1 dr - cos θ
= : =
r dθ 1 + sin θ r dθ 1 − sin θ
cos θ - cos θ
ie., cot φ1 = : cot φ 2 =
1 + sin θ 1 − sin θ
1 + sin θ 1 - sin θ
We have tan φ1 = and tan φ 2 =

.in
cos θ - cos θ

1 - sin 2 θ cos 2 θ
∴ tan φ1 . tan φ 2 = = = -1
sy
− cos 2 θ
Hence the curves intersect orthogonally.
- cos 2 θ
ea

3. Find the angle of intersection of the curves:


y

r = sin θ + cos θ, r = 2 sin θ


ud

Solution : log r = log (sin θ + cos θ) : r = 2 sin θ


⇒ log r = log (sin θ + cos θ ) : log r = log 2 + log (sin θ)
st

Differentiating these w.r.t θ we get


1 dr cos θ - sin θ 1 dr cos θ
= : =
r dθ sin θ + cos θ r dθ sin θ
cos θ (1 - tan θ )
ie., cot φ1 = : cot φ 2 = cot θ ⇒ φ 2 = θ
cos θ (1 + tan θ )
ie., cot φ1 = cot (π/4 + θ ) ⇒ φ1 = π/4 + θ

∴ φ1 - φ 2 = π/4 + θ - θ = π/4

The angle of intersection is π / 4

Compiled by www.studyeasy.in
Compiled by www.studyeasy.in

2011

4. Find the angle of the curves: r = a log θ and r = a/ log θ


Solution : r = a log θ : r = a/ log θ
⇒ log r = log a + log (log θ ) : log r = log a - log (log θ )
Differentiating these w.r.t q, we get,
1 dr 1 1 dr 1
= : =−
r dθ log θ . θ r dθ log θ . θ
1 1
ie., cot φ1 = : cot φ 2 = -
θ log θ θ log θ
Note : We can not find φ1 and φ 2 explicitly.
∴ tan φ1 = θ log θ : tan φ 2 = θ log θ

.in
Now consider, tan φ1 = θ log θ : tan φ 2 = - θ log θ

2 θ log θ 2θ log θ
Now consider, tan (φ1 - φ 2 ) = sy = ......…(1)
1 + tan φ1 tan φ 2 1 − (θ log θ)2

We have to find θ by solving the given pair of equations :


ea
r = a log θ and r = a/log θ
a
Equating the R.H.S we have a log θ =
y

log θ
ud

ie., (log θ = 1 )2 = 1 or log θ ⇒ θ = e


Substituting θ = e in (1) we get
st

tan (φ1 − φ 2 ) =
2e
(∵ log e = 1)
1 - e2
⎛ 2e ⎞
∴ angle of intersection φ1 - φ 2 = tan -1 ⎜ 2 ⎟
= 2 tan -1 e
⎝1- e ⎠
5. Find the angle of intersection of the curves:
r = a (1 – cos θ ) and r = 2a cos θ
Solution : r = a (1 – cos θ) : r = 2a cos θ
Taking logarithms we have,
Log r = log a + log (1 – cos θ) : log r = log 2a + log (cos θ)
Differentiating these w.r.t θ, we get,

Compiled by www.studyeasy.in
Compiled by www.studyeasy.in

2011
1 dr sin θ 1 dr - sin θ
= : =
r dθ 1 - cos θ r dθ cos θ
2 sin (θ/2 ) cos (θ/2 )
ie., cot φ1 = : cot φ 2 = - tan θ
2 sin 2 (θ/2 )
ie., cot φ1 = cot (θ /2 ) : cot φ 2 = cot (π/2 + θ )

⇒ φ1 = θ /2 : φ 2 = π/2 + θ

∴ φ1 − φ 2 = θ/2 - π/2 - θ = π / 2 +θ 2 …(1)

Now consider r = a (1 - cos θ ) = 2a cos θ

Or 3 cos θ = 1 or θ = cos -1 (1/3)


Substituting this value in (1) we get,

.in
The angle of intersection π / 2 + 1/2 . cos -1 (1/3)

sy
6. Find the angle of intersection of the curves :
r = aθ and r = a / θ
ea
Solution : r = aθ : r=a/θ
⇒ log r = log a + log θ : log r = log a - log θ
y

Differentiating these w.r.t θ, we get,


ud

1 dr 1 1 dr 1
= : =-
r dθ θ r dθ θ
1 1
ie., cot φ1 = : cot φ 2 = -
st

θ θ
or tan φ1 = θ : tan φ 2 = - θ

Also by equating the R.H.S of the given equations we have


aθ = a/ θ or θ 2 = 1 ⇒ θ = ± 1
When θ = 1, tan φ1 = 1, tan φ 2 = - 1 and
When θ = - 1, tan φ1 = - 1, tan φ 2 = 1 .
∴ tan φ1 . tan φ 2 = - 1 ⇒ φ1 - φ 2 = π /2

The curves intersect at right angles.

Compiled by www.studyeasy.in
Compiled by www.studyeasy.in

2011

7. Find the pedal equation of the curve: r (1 - cos θ ) = 2a


Solution : r (1 - cos θ ) = 2a
⇒ log r + log (1 - cos θ ) = log 2a

Differentiating w.r.t θ, we get


1 dr sin θ 1 dr - sin θ
+ = 0 or =
r dθ 1 - cos θ r dθ 1 - cos θ
- 2 sin (θ/2 ) cos (θ/2 )
∴ cot φ = = - cot (θ/2 )
2 sin 2 (θ/2 )
ie., ∴ cot φ = cot (- θ /2 ) ⇒ φ = - (θ /2 )
Consider p = r sin φ

.in
∴ p = r sin (- θ /2 ) or p = - r sin (θ /2 )

Now we have, r (1 - cos θ ) = 2a …(1)


sy
p = - r sin (θ /2 ) …(2)
We have to eliminate θ from (1) and (2)
ea
(1) can be put in form r . 2 sin 2 (θ / 2 ) = 2a

ie., r sin 2 (θ/2 ) = a


y
ud

But p/-r = sin (θ/2), from (2)


⎛ p2 ⎞
∴ r ⎜⎜ 2 ⎟⎟ = a or p 2 = ar
st

⎝r ⎠
Thus p2 = ar is the required pedal equation.

8. Find the pedal equation of the curve: r2 = a2 sec 2θ


Solution : r2 = a2 sec 2θ
⇒ 2 log r = 2 log a + log (sec 2θ)
Differentiating w.r.t θ , we get,
2 dr 2 sec 2θ tan 2θ 1 dr
= ie., = tan 2θ
r dθ sec 2θ r dθ
ie., cot φ = cot (π /2 - 2θ ) ⇒ φ = π/2 - 2θ

Compiled by www.studyeasy.in
Compiled by www.studyeasy.in

2011
Consider p = r sin φ ∴ p = r sin (π /2 - 2θ ) ie., p = r cos 2 θ

Now we have, r 2 = a 2 sec 2θ …(1)


p = r cos 2θ …(2)
From (2) p/r = cos 2θ or r/p = sec 2 θ
Substituting in (1) we get, r 2 = a 2 (r/p ) or pr = a2
Thus pr = a2 is the required pedal equation.

9. Find the pedal equation of the curve: rn = an cos nθ


Solution : rn = an cos nθ
⇒ n log r = n log a + log (cos n θ )

.in
Differentiating w.r.t q we get
n dr - n sin nθ 1 dr
= ie., = - tan nθ
r dθ cos nθ r dθ sy
∴ cot φ = cot (π /2 + n θ ) ⇒ φ = π /2 + n θ

Consider p = r sin φ
ea
∴ p = r sin (π /2 = nθ ) ie., p = r cos nθ

Now we have, r n = a n cos nθ …(1)


y

p = r cos nθ …(2)
ud

∴ (1) as a consequenc e of (2) is r n = a n (p/r )


st

Thus rn + 1 = pan is the required pedal equation.

10. Find the pedal equation of the curve: rm = am (cos mθ + sin mθ)
Solution : rm = am (cos mθ + sin mθ)
Differentiating w.r.t θ, we get,
m dr - m sin mθ + m cos mθ
=
r dθ cos mθ + sin mθ
1 dr cos mθ - sin mθ cos mθ (1 - tan mθ )
ie., = =
r dθ cos mθ + sin mθ cos mθ (1 + tan mθ )
∴ cot φ = cot (π /4 + m θ ) ⇒ φ = π /4 + m θ

Compiled by www.studyeasy.in
Compiled by www.studyeasy.in

2011
Consider p = r sin φ
∴ p = r sin (π /4 + m θ )

ie., p = r [sin (π /4 ) cos m θ + cos (π/4 ) sin m θ ]

ie., p =
r
(cos mθ + sin mθ )
2
(we have used the formula of sin (A + B) and also the values sin (π/4 ) = cos (π/4 ) = 1/ 2 )
Now we have, r m = a m (cos m θ + sin m θ ) …(1)

p=
r
(cos m θ + sin mθ ) …(2)
2
Using (2) in (1) we get,

.in
p 2
rm = am . or r m +1 = 2 a m p
r
Thus r m + 1 = 2 a m p is the required pedal equation.
sy
11. Establish the pedal equation of the curve:
ea
( )
r n = a n sin n θ + b n cos nθ in the form p 2 a 2n + b 2 n = r 2 n + 2

Solution : We have r n = a n sin nθ + b n cos nθ


y

(
⇒ n log r = log a n sin nθ + b n cos nθ )
ud

Differentiating w.r.t θ we get


st

n dr na n cos nθ - nb n sin nθ
= n
r dθ a sin nθ + b n cos nθ
a n cos nθ - b n sin nθ
Dividing by n, cot φ = n
a sin nθ + b n cos nθ
Consider p = r sin φ
Since φ cannot be found, squaring and taking the reciprocal we get,
1
p 2
1 1 1
(
= 2 cosec 2 φ or 2 = 2 1 + cot 2 φ
r p r
)
1 1
∴ 2= 2
⎧⎪ (
⎨1 + n
2
)
a n cos nθ - b n sin nθ ⎫⎪
2⎬
p r ⎪⎩ (a sin nθ + b n cos nθ ⎪⎭ )

Compiled by www.studyeasy.in
Compiled by www.studyeasy.in

2011

1 1
ie., 2 = 2 ⎨
( ) (
⎧⎪ a n sin nθ + b n cos nθ 2 + a n cos nθ - b n sin nθ ) ⎫⎪⎬
2

p r ⎪⎩ (
a n sin nθ + b n cos nθ
2
) ⎪⎭

1 1
ie., 2 = 2 ⎨
( ) (
⎧⎪ a 2 n sin 2 nθ + cos 2 nθ + b 2 n cos 2 nθ + sin 2 nθ ⎫⎪

)
p r ⎪⎩ (
a n sin nθ + b n cos nθ
2
) ⎪⎭

(product terms cancels out in the numerator)


1 1 a 2 n + b2 n
=
( )
ie., .
p 2 r 2 a n sin nθ + b n cos nθ 2

1 1 a 2 n + b 2n
= , by using the given equation.
( )
or .
p2 r 2 rn
2

(
∴ p2 a 2 n + b2 n )= r 2 n+2
is the required pedal equation.

.in
12. Define Curvature and Radius of curvature sy
Solution: A Curve Cuts at every point on it. Which is determined by the tangent drawn.
ea

Y y = f (x) Tangent
y
ud

s P (x,y)
A
st

O ψ X
Let P be a point on the curve y= f (x) at the length ‘s’ from a fixed point A on it. Let the tangent
at ‘P’ makes are angle ψ with positive direction of x – axis. As the point ‘P’ moves along curve, both s
and ψ vary.

The rate of change ψ w.r.t s, i.e., as called the Curvature of the curve at ‘P’.
ds
The reciprocal of the Curvature at P is called the radius of curvature at P and is denoted by ρ.

1 ds
∴ρ = =
dψ dψ
ds

Compiled by www.studyeasy.in
Compiled by www.studyeasy.in

2011
ds 1 dψ
∴ρ = (or) =
dψ ρ ds
1 dψ
Also denoted ρ = ∴K =
K ds
K is read it as Kappa.

13. Derive an expression for radius of curvature in Cartesian form.


Solution :(1) Cartesian Form:

Y = f (x)

P T
A S ψ
dy
ψ

.in
ψ P dx R
X
O
sy
Let y = f (x) be the curve in Cartesian form.
ea
dy
We know that, tan ψ = (From Figure) ----------- (1)
dx
Where ψ is the angle made by the tangent at P with x – axis. Differentiating (1) W.r.t x, we get
y


ud

d2y
Sec2ψ . =
dx dx 2
d2y dψ
i.e., = Sec2 ψ
st

2
dx dx
d ψ
= (1 + tan2 ψ)
dx
d ψ ds
= (1 + tan2 ψ) .
dx dx
1

1 ⎧⎪ ⎛ dy ⎞ ⎫⎪ 2
2

= (1 + tan ψ)
2
⎨1 + ⎜ ⎟ ⎬
ρ ⎪⎩ ⎝ dx ⎠ ⎪⎭
1
⎧⎪ ⎛ dy ⎞ 2 ⎫⎪ 1 ⎧⎪ ⎛ dy ⎞ 2 ⎫⎪ 2
= ⎨1 + ⎜ ⎟ ⎬ . . ⎨1 + ⎜ ⎟ ⎬ from eqn (1)
⎪⎩ ⎝ dx ⎠ ⎪⎭ ρ ⎪⎩ ⎝ dx ⎠ ⎪⎭

Compiled by www.studyeasy.in
Compiled by www.studyeasy.in

2011
3

1 ⎧⎪ ⎛ dy ⎞ 2 ⎫⎪ 2
= ⎨1 + ⎜ ⎟ ⎬
ρ ⎪⎩ ⎝ dx ⎠ ⎪⎭
3
⎡ ⎛ dy ⎞ 2 ⎤ 2
⎢1 + ⎜ ⎟ ⎥
⎢ ⎝ dx ⎠ ⎦⎥
∴ρ= ⎣ 2
d y
dx 2
∴ ρ = ( 1+ y1 2 )3/2 ……………(1)
y2
dy d2y
Where y1 = , y2 =
dx dx 2
This is the formula for Radius of Curvature in Cartesian Form.
14. Show that the Curvature of a Circle at any point on it, is a Constant

.in
Tangent

O
sy r PP P
ea
A ψ X
T
y
ud

Solution: Consider a Circle of radius r. Let A be a fixed point and ‘P’ be a given point on the circle
such that arc AP = S.
st

Let the angle between the tangent to the Circle at A and P be ψ. Then clearly AOP = ψ.

∴ AP = rψ
i.e., S = rψ
This is the intrinsic equation of the circle.
Differentiating w.r.t S we get

dψ dψ 1
1=r Or K = =
ds ds r
1
∴ K = which is Constant
r

Hence the Curvature of the Circle at any point on it is constant.

Compiled by www.studyeasy.in
Compiled by www.studyeasy.in

2011

15. Derive an expression for radius of curvature in parametric form.

Solution: We have ρ = (1 + y 12 )3/2

y2
Let x =f (t), y = g (t) be the curve in Parametric Form.
dy •
dy dt Y
Then y1 = = = •
dx dx
dt X
⎛ • ⎞ ⎛ • ⎞
d2y d ⎛ dy ⎞ d ⎜ Y ⎟ d ⎜Y ⎟ dt
Y2 = = ⎜ ⎟ =
dx 2 dx ⎝ dx ⎠ dx ⎜⎜ X• ⎟⎟ dt ⎜⎜ X• ⎟⎟ dx
⎝ ⎠ ⎝ ⎠
⎛ ⎞
⎛ • ⎞ ⎛ • ⎞
d ⎜ Y ⎟ ⎜⎜ 1 ⎟⎟ d ⎜ Y ⎟ 1

.in
= . .
dt ⎜⎜ X• ⎟⎟ ⎜ dx ⎟ dt ⎜⎜ X• ⎟⎟ X•
⎝ ⎠ ⎜ ⎟ ⎝ ⎠
⎝ dt ⎠
=
xy − yx 1
(x )2 ( X• )
sy

ea
X y − yx
∴ Y2 = 3
⎛•⎞
⎜ x⎟
⎝ ⎠
y

Substituting Y1 and Y2 in equation 1.


ud

ρ=
(1 + Y ) 1
2
3
2
, we get
y2
st

( ( )
3
⎡1+ Y 2⎤ 2

⎢ x ⎥⎦
ρ= ⎣
xy − yx
(x )3
ρ=
[(x) + ( y ) ]
2 2
3
2
-------------------- (2)
xy − yx
Equation (2) is called the Radius of Curvature in Parametric Form.

Compiled by www.studyeasy.in
Compiled by www.studyeasy.in

2011

16. Derive an expression for radius of curvature in polar form.


Solution: Let r = f (θ) be the curve in the Polar Form. We know that, Angle between the
tangent and radius vector,


Tan φ = r
dr
1
= r.
dr

r
i.e., tan φ =
(dr dθ )
Differentiate w.r.t ‘θ’ we get

dr dr d 2r
. −r

.in

= dθ dθ 2 dθ
2
Sec2 φ .
dθ ⎛ dr ⎞
⎜ ⎟
⎝ dθ ⎠



=
⎧ dr
1 ⎪ dθ r
2
(
d 2r 2 ⎫
dθ ⎪
)
sy
2 ⎨ ⎬
dθ Sec φ ⎪ 2
( )
ea
dr ⎪
⎩ dθ ⎭

(
⎧ dr 2
)
− d 2r 2 ⎫
y

⎪ dθ r
=
1 dθ ⎪
⎨ ⎬
1 + tan φ ⎪
2
( 2
)
ud

dr ⎪
⎩ dθ ⎭
⎧ ⎛ dr ⎞ 2
d 2r ⎫
⎪⎜ ⎟ −r ⎪
⎪ ⎝ dθ ⎠ dθ 2 ⎪
st

1
= ⎨ ⎬
r2 ⎪ ⎛ dr ⎞
2

1+ ⎜ ⎟
⎛ dr ⎞ ⎩
2
⎪ ⎝ dθ ⎠ ⎪⎭
⎜ ⎟
⎝ dθ ⎠
2
⎛ dr ⎞ d 2r
⎜ ⎟ −r 2
dφ dθ ⎠ dθ
= ⎝
dθ ⎛ dr ⎞
2

⎜ ⎟ +r
2

⎝ d θ ⎠

From figure ψ = θ + φ

Differentiating w.r.t θ, we get

Compiled by www.studyeasy.in
Compiled by www.studyeasy.in

2011
dψ dφ
=1+
dθ dθ

2
⎛ dr ⎞ d 2r
⎜ ⎟ −r
dψ dθ ⎠ dθ 2
∴ =1+ ⎝
dθ ⎛ dr ⎞
2

⎜ ⎟ +r
2

⎝ dθ ⎠
2
⎛ dr ⎞ d 2r
r 2 + 2⎜ ⎟ −r
dψ ⎝ dθ ⎠ dθ 2
=
dθ ⎛ dr ⎞
2

⎜ ⎟ +r
2

⎝ d θ ⎠
1

ds ⎧⎪ ⎛ dr ⎞ ⎫⎪ 2
2

Also we know that = ⎨r 2 + ⎜ ⎟ ⎬


dθ ⎪⎩ ⎝ dθ ⎠ ⎪⎭

.in
ds ds dθ
Now, ρ= = .
dψ dθ dψ
1⎛ dr ⎞
⎜ ⎟ +r
2
2
sy
⎧⎪ 2 ⎛ dr ⎞ ⎫⎪
2 2
⎝ d θ ⎠
ea
= ⎨r + ⎜ ⎟ ⎬ .
⎪⎩ ⎝ dθ ⎠ ⎪⎭
2
⎛ dr ⎞ d 2r
r + 2⎜
2
⎟ −r
⎝ dθ ⎠ dθ 2
y

(r
+ r12 ) 2
3
ud

2
ρ = 2 ----------- (3)
r + 2r12 − rr2
st

dr d 2r
Where r1 = , r2 =
dθ dθ 2

Equation (3) as called the radius of curvature in Polar form.

17. Derive an expression for radius of curvature in pedal form.


Solution: Let p = r Sin φ be the curve in Polar Form.

We have p = r Sin φ

Differentiating p W.r.t r, we get

dp dφ
= Sin φ + r Cos φ
dr dr

Compiled by www.studyeasy.in
Compiled by www.studyeasy.in

2011

dθ dr
But Sin φ = r , Cos φ=
ds ds

dp dθ dr dφ
∴ =r +r
dr ds ds dr

dθ dφ dr
=r + .
ds dr ds

dθ dφ
=r +r
ds ds

d
=r (θ + φ)
ds

.in

=r
ds


dp
dr
=r

ds
sy
ea
dp 1 dψ 1
= r. ∵ =
dr ρ ds ρ
y
ud

dr
∴ ρ= r --------------- (4)
dp

Equation (4) is called Radius of Curvature of the Curve in Polar Form.


st

18. Find the radius of curvature at (x, y) for the curve ay2 = x3.

Solution: Given ay2 = x3 -------- (1) is in Cartesian form.

We have, Radius of curvature in Cartesian form.

ρ=
(1 + y )2
1
3
2
------------ (2)
y2

Differentiating (1) w.r.t x we get,

Compiled by www.studyeasy.in
Compiled by www.studyeasy.in

2011

dy dy 3 x2 3x 2 3 x
a 2y = 3x 2 ⇒ = y1 = = 1
=
dx dx 2 ay 2 a
⎛ x3 ⎞ 2
2a⎜⎜ ⎟⎟
⎝ a ⎠
3 x
∴ y1 =
2 a

Differentiating y1 w.r.t ‘x’ we get

d2y 3 1 3
y2 = 2
= =
dx 2 a 2 x 4 a x

Substitute Y1 and Y2 in (2), we get.


x (4a + 9 x )
3
2
ρ=

.in
6a

⎛ x⎞
19. Find the radius of curvature at (x,y) for the curve y = c log Sec ⎜ ⎟

Solution: Given y = c log Sec ⎜ ⎟


⎛ x⎞
-------- (1)
sy ⎝c⎠

⎝c⎠
ea
Differentiating (1) w.r.t x, we get

1 ⎛ x⎞ ⎛ x⎞ ⎛ x⎞ 1 ⎛ x⎞
y

y1 = c. .Sec⎜ ⎟ tan⎜ ⎟ = c tan ⎜ ⎟ . = tan ⎜ ⎟


⎛ x⎞ ⎝c⎠ ⎝c⎠ ⎝c⎠ c ⎝c⎠
Sec⎜ ⎟
ud

⎝c⎠
Differentiating y1 W.r.t x we get
st

⎛ x⎞ 1 1 ⎛ x⎞
y 2 = Sec 2 ⎜ ⎟ = Sec 2 ⎜ ⎟
⎝c⎠ c c ⎝c⎠

Substitute y1 and y2 in δ =
(1 + y ) 2
1
3
2

y2

∴δ =
[1 + tan (x c )] = [Sec (x c )]
2
3
2 2
3
2

Sec (x ) Sec (x )
1 2 1 2

c c c c

( c)
ρ = cSec x

Compiled by www.studyeasy.in
Compiled by www.studyeasy.in

2011

20. Find the radius of curvature at the point ‘t’ on the curve x = a (t + Sint), y = a (1 – Cost).

Solution: Given Curves are in Parametric Form

∴ Radius of Curvature, ρ =
((x) + (y) 2
2 2

------ (1)
)
3

xy − yx
Differentiating the given Curves W.r.t t, we get

dx dy
x= = a (1 + Cost) y = = a Sint
dt dt
Differentiating W.r.t ‘t’ we get

x = - a Sint, y = a Cost

Substitute x , y , y and x in (1), we get

.in
ρ=
(a (1 + Cost )
2
+ a 2 S int 22
) 3

a(1 + Cost )aCost − aS int( − aS int)

=
a 3 {1 + 2Cost + Cos 2 t + Sin 2 t } 2
3
sy
a 2 {Cost + Cos 2 t + Sin 2 t}
ea
{ }= { } 3

a 2(1 + Cost )
3
2 a 2.2Cos 2 t 2
8a.Cos 3 t
= 2 = 2
(1 + Cost ) 2Cos t
2
2Cos t 2
y

2 2
ρ = 4aCos t
ud

2
21. Find the Radius of Curvature to x = a Cost + log tan t { ( 2 )}, y = a Sint at t.
st

{
Solution: Here x = a Cost + log tan t ( 2 )}, y = a Sin t
dx ⎧
⎪ 1 1 ⎫⎪
= a ⎨− S int + 2 t
2 2⎬
.Sec .
dt ⎪⎩ tan t ⎪⎭
2
1
=a { -sint + }
2 sin t / 2 cos t / 2
1
=a { -sint + }
2 sin t / 2
= a { (1-sin 2t) / sint }

dx
= a Cos2t / sint
dt

Compiled by www.studyeasy.in
Compiled by www.studyeasy.in

2011
dy
and = aCost
dx

dy dy / dt aCost
∴ = = = tant
dx dx / dt a cos t cos t / sin t

dy
∴ = tan t
dx
Differentiating W.r.t ‘x’ we get

d2y dt 1 1
2
= Sec 2 t = Sec 2 t = Sec 2 t.
dx dx dx aCos t
2
S int
dt
d2y 1
2
= = Sec 4 tS int
dx a

.in
Substitute
dy
&
d2y
in ρ =
(1 + y12 ) 3
2
, we get
dx dx 2 y2 sy
(1 + tan t ) 2
3
2
aSec3t Cost
ea
i.e., ρ= = 4
=a = aCott
1 Sec tS int S int
Sec 4tS int
a
∴ρ = a Cot t.
y
ud

⎛a a⎞
22. Find the Radius of Curvature to x + y = a at ⎜ , ⎟
⎝ 4 4⎠
Solution: Given x+ y= a -------------- (1)
st

Differentiating (1) w.r.t to ‘x’ we get

1 1 dy
+ =0
2 x 2 y dx

i.e., y1 =
dy
=−
y
=−
( a− x ) (From (1))
dx x x

a
y1 = 1 − ----------- (2)
x

Compiled by www.studyeasy.in
Compiled by www.studyeasy.in

2011

d2y ⎛ 1 ⎞ −3 a
Also y 2 = = − a ⎜ − ⎟x 2 = 3 -------------- (3)
⎝ 2⎠
2
dx 2x 2

⎛a a⎞
At the given point ⎜ , ⎟
⎝ 4 4⎠
a a 4
Then y1 = 1 − = - 1 & y2 = 1 − =
a
2 2a 2
4
3
a
( )
(1 + y )
3
2 2
∴ Substitute y1 and y2 in ρ= 1

y2

ρ=
(1 + (−1) ) 2
3
2
=
3
2 2 a 2 2a
= =
a
4 4 4 2
a

.in
a
ρ=
2
sy
23. Show that for the Cardioids r = a (1 + Cosθ), ρ2 / r 2 is a constant
ea
Solution: r = a (1 + Cosθ)

dr
= - a Sinθ
y


ud

We have,

2
st

1 1 1 ⎛ dr ⎞
= 2 + 4⎜ ⎟ , is Pedal Equation.
P 2
r r ⎝ dθ ⎠

1 1
= 2
+ 4 a2Sin2θ
r r

r 2 + a 2 Sin 2θ a 2 (1 + Cosθ ) 2 + a 2 Sin 2θ


= =
r4 r4

2a 2 (1 + Cosθ )
=
r4

Compiled by www.studyeasy.in
Compiled by www.studyeasy.in

2011
1 2a 2 ⎛r⎞
∴ = ⎜ ⎟ (∵ r = a (1 + Cosθ)
P2 r4 ⎝a⎠
r
∴ = 1 + Cosθ
a
1 2a
2
= 3
P r

r3
∴ P2 =
2a

Differentiating w.r.t ‘P’ we get


1 2 dr
2P = 3r
2a dp

.in
dr 4ap
=
dp 3r 2

Now, ρ=r
dr 4ap
dp
=
3r
sy
ea
ρ2 1 ⎡16a 2 p 2 ⎤ 16a 2 r 3 8a
And = ⎢ ⎥= . =
r2 r ⎣ 9r 2 ⎦ 9r 3 2a 9
y

1 1 1 r2
Find the Radius of Curvature of the Curve = + −
ud

24.
P 2 a 2 b 2 a 2b 2
1 1 1 r2
Solution: Given = + −
P 2 a 2 b 2 a 2b 2
st

Differentiating w.r.t to P, we get

−2 −1 dr
3
= 2 2 2r
P a b dp

dr a 2 b 2
∴ = 3
dp p r

dr a 2b 2 a 2b 2
∴ ρ = r. = r. 3 = 3
dp pr p

Compiled by www.studyeasy.in
Compiled by www.studyeasy.in

2011

a
25. Find the Radius of Curvature at (r,θ) on r =
θ
a
Solution: Given r =
θ

Differentiating w.r.t to ‘θ’ we get

dr − a − a 1 − r
= = . =
dθ θ 2 θ θ θ

dr − r
=
dθ θ
2
1 1 1 ⎛ dr ⎞
We have 2 = 2 + 4 ⎜ ⎟

.in
P r r ⎝ dθ ⎠
1 1 r2 1 1
= 2 + 4 2 = 2 + 2 2
r r θ r rθ sy
1 1 ⎛ 1 ⎞ 1 ⎛ θ 2 + 1⎞
= ⎜1 + ⎟ = ⎜ ⎟
P 2 r 2 ⎝ θ 2 ⎠ r 2 ⎜⎝ θ 2 ⎟⎠
ea


∴P =
y

θ 2 +1
ud

rθ r. a a.r
P= = r =
θ 2 +1 a2 a2 + r 2
+ 1
r2
st

Differentiating above result w.r.t to‘P’ we get

dr 1 dr
a 2 + r 2 .a − r.a 2/ r
dp 2/ a 2 + r 2 dp
1=
(
a +r
2 2
)
⎛ 2 r 2a ⎞ dr
2

a +r =⎜ a +r .a−
2 2

⎟ dp
⎝ a2 + r 2 ⎠

(a 2 + r 2 ). a − r 2 a dr
a2 + r 2 =
a2 + r 2 dp

Compiled by www.studyeasy.in
Compiled by www.studyeasy.in

(a 2
+ r 2 ) a2 + r 2 = a3
dr
dp
(a 2
+ r2 )
3
2
=
dr
a3 dp

Thus, ρ = r.
dr r. a 2 + r 2
=
( )
3
2

dp a3

∴ ρ = 3 (a 2 + r 2 ) 2
r 3

Exercises:
⎛π ψ ⎞
(1) Find the Radius of the Curvature at the point (s, ψ) on S = a log tan ⎜ + ⎟

.in
⎝4 2⎠

(2) Find the Radius of the Curvature of xy = C2 at (x,y)


sy
(3) Find the Radius of the Curvature of xy3 = a4 at (a,a)
ea
(4) Find the Radius of Curvature at the point θ on x = C Sin 2θ (1 + Cos 2θ),
y = C Cos 2θ (1 – Cos 2θ)
y

(5) If ρ1 and ρ2 are the radii of curvature at the extremities of any chord of the cardiode
16a 2
ud

r = a (1 + Cosθ) which posses through the Pole prove that ρ1 + ρ 2 =


2 2

9
st

Compiled by www.studyeasy.in
Compiled by www.studyeasy.in

2011

DIFFERENTIAL CALCULUS – III

PARTIAL DIFFERENTIATION
Introduction :-
Partial differential equations abound in all branches of science and engineering and
many areas of business. The number of applications is endless.
Partial derivatives have many important uses in math and science. We shall see
that a partial derivative is not much more or less than a particular sort of directional
derivative. The only trick is to have a reliable way of specifying directions ... so most of
this note is concerned with formalizing the idea of direction

.in
So far, we had been dealing with functions of a single independent variable. We will now
consider functions which depend on more than one independent variable; Such
sy
functions are called functions of several variables.
ea
Geometrical Meaning
Suppose the graph of z = f (x,y) is the
y

surface shown. Consider the partial


ud

derivative of f with respect to x at a point


(x0, y0). Holding y constant and varying x,
st

we trace out a curve that is the intersection


of the surface with the vertical plane y = y0.
The partial derivative fx(x0,y0). measures
the change in z per unit increase in x along
this curve. That is, fx(x0, y0) is just the slope
of the curve at (x0, y0). The geometrical
interpretation of fy(x0, y0). is analogous.

Compiled by www.studyeasy.in
Compiled by www.studyeasy.in

2011
Real-World Applications:
Rates of Change:
In the Java applet we saw how the concept of partial derivative could be applied
geometrically to find the slope of the surface in the x and y directions. In the following
two examples we present partial derivatives as rates of change. Specifically we explore
an application to a temperature function ( this example does have a geometric aspect in
terms of the physical model itself) and a second application to electrical circuits, where
no geometry is involved.
I. Temperature on a Metal Plate
The screen capture below shows a current website illustrating thermal flow for chemical

.in
engineering. Our first application will deal with a similar flat plate where temperature
varies with position. sy
* The example following the picture below is taken from the current text in SM221,223:
ea
Multivariable Calculus by James Stewart.
y
ud
st

Compiled by www.studyeasy.in
Compiled by www.studyeasy.in

2011
Suppose we have a flat metal plate where the temperature at a point (x,y) varies
according to position. In particular, let the temperature at a point (x,y) be given by,

T ( x, y ) = 60 /1 + x 2 + y 2
where T is measured in oC and x and y in meters.
Question: what is the rate of change of temperature with respect to distance at the point
(2,1) in (a) the x-direction? and (b) in the y-direction ?
Let's take (a) first.
What is the rate of change of temperature with respect to distance at the point (2,1) in
(a) the x-direction?
What observations and translations can we make here?

.in
Rate of change of temperature indicates that we will be computing a type of derivative.
Since the temperature function is defined on two variables we will be computing a partial
sy
derivative. Since the question asks for the rate of change in the x-direction, we will be
ea
holding y constant. Thus, our question now becomes:
What is dT dxat the point (2,1)?
y

T ( x, y ) = 60 /1 + x 2 + y 2 = 60(1 + x 2 + y 2 ) −1
ud

∂T = −60(2 x)(1 + x 2 + y 2 ) −2
∂x
∂T (2,1) = −60(4)(1 + 4 + 1) −2 = −20
∂x 3
st

Conclusion :
The rate of change of temperature in the x-direction at (2,1) is −20 3 degrees
per meter;
note this means that the temperature is decreasing !

Compiled by www.studyeasy.in
Compiled by www.studyeasy.in

2011

Part (b):
The rate of change of temperature in the y-direction at (2,1) is computed in a similar
manner. T ( x, y ) = 60 /1 + x 2 + y 2 = 60(1 + x 2 + y 2 ) −1
∂T = −60(2 y )(1 + x 2 + y 2 ) −2
∂x
∂T (2,1) = −60(2)(1 + 4 + 1) −2 = −10
∂x 3
Conclusion :
The rate of change of temperature in the y-direction at (2,1) is −10 degrees
3
per meter;
note this means that the temperature is decreasing !

.in
II. Electrical Circuits: Changes in Current sy
The following is adapted from an example in a former text for SM221,223 Multivariable
ea
Calculus by Bradley and Smith.
* In an electrical circuit with electromotive force (EMF) of E volts and resistance R ohms,
y

the current, I, is
ud

I=E/R amperes.
Question: (a) At the instant when E=120 and R=15 , what is the rate of change of
st

current with respect to voltage.


(b) What is the rate of change of current with respect to resistance?

(a) Even though no geometry is involved in this example, the rate of change questions
can be answered with partial derivatives.
we first note that I is a function of E and R, namely,
I(E,R) = ER-1

Compiled by www.studyeasy.in
Compiled by www.studyeasy.in

The rate of change of current with respect to voltage =


the partial derivative of I with respect to voltage, holding resistance constant is
∂I = R −1
∂E

when E=120 and R=15 , we have ∂ I ∂ E = 15 −1 ≈ 0.0667


verbal conclusion : If the resistance is fixed at 15 ohms, the current is increasing with
respect to voltage at the rate of 0.0667 amperes per volt when the EMF is 120 volts.
Part (b):
What is the rate of change of current with respect to resistance?

.in
Using similar observations to part (a) we conclude:
sy
The partial derivative of I with respect to resistance, holding voltage constant = ∂I ∂E = ER −1
ea
when E=120 and R=15 , we have ∂I ∂E (120,15) = −120(15)−1 ≈ −0.5333
y

Conclusion : If the EMF is fixed at 120 volts, the current is decreasing with respect to
ud

resistance at the rate of 0.5333 amperes per ohm when the resistance is 15 ohms.
st

Key Words :-
Then the partial derivative of z w.r.t x is given by
∂z f ( x + δ x, y ) − f ( x, y )
zx = = lim
∂x δ x→0 δx
The partial derivative of z w.r.t y is given by
∂z f ( x, y + δ y ) − f ( x, y )
zy = = lim
∂y δ y →0 δy

Compiled by www.studyeasy.in
Compiled by www.studyeasy.in

2011
∂u ∂u
1. If u = e ax -by sin (ax + by ) show that b -a = 2 ab u
∂x ∂y

Solution : u = e ax -by sin (ax + by )


∂u
∴ = e ax - by cos (ax + by ) . a + a.e ax -by sin (ax + by )
∂x
∂u
ie., = a e ax - by cos (ax + by ) + au …(1)
∂x
∂u
Also = e ax - by cos (ax + by ) . b + (- b ) e ax -by sin (ax + by )
∂x
∂u
ie., = b e ax - by cos (ax + by ) − bu …(2)
∂y
∂u ∂u

.in
Now b =a by using (1) and (2) becomes
∂x ∂y

= abe ax − by cos (ax + by ) + abu - abe ax -by cos (ax + by ) + abu


= 2 abu
sy
∂u ∂u
ea
Thus b - a = 2 abu
∂x ∂y

2. If u = eax + by f (ax - by ), prove that


y

∂u ∂u
=a = 2abu
ud

b
∂x ∂y

Solution : u = e ax+ by f (ax - by ) , by data


st

∂u
= e ax + by . f ′ (ax - by ) a + ae ax + by f (ax - by )
∂x
∂u
Or = a e ax + by . f ′ (ax - by ) + a u …(1)
∂x
∂u
Next, = e ax + by f ′ (ax - by ) . (- b ) + b e ax + by f (ax - by )
∂y
∂u
Or = − b e ax + by f ′ (ax - by ) + ba …(2)
∂y
∂u ∂u
Now consider L.H.S = b + a
∂x ∂y

Compiled by www.studyeasy.in
Compiled by www.studyeasy.in

2011
{ } {
= b ae ax + by f ′ (ax - by ) + au + a - be ax + by f ′ (ax - by ) + bu }
= ab e ax + by f ′ (ax - by ) + abu - ab e ax + by f ′ (ax - by ) + abu
= 2abu = R.H.S
∂u ∂u
Thus b +a = 2a bu
∂x ∂y

⎛ ∂ 2u ∂ 2u ∂ 2u ⎞
3. ( )
If u = log x 2 + y 2 + z 2 , show that x 2 + y 2 + z 2 ⎜⎜ 2 + 2 + 2 ⎟⎟ = 1
⎝ ∂x ∂y ∂z ⎠

Solution : By data u = log x 2 + y 2 + z 2 = log (x 2 + y 2 + z 2 )


1
2
The given u is a symmetric function of x, y, z,
(It is enough if we compute only one of the required partial derivative)

.in
∂u 1 1 x
= . 2 . 2x = 2
∂x 2 x + y + z
2 2
x + y2 + z 2

∂ 2 u ∂ ⎛ ∂u ⎞ ∂ ⎛
∂x 2
= ⎜ ⎟= ⎜⎜ 2
x ⎞

2 ⎟
∂x ⎝ ∂x ⎠ ∂x ⎝ x + y + z ⎠
2
sy
ea
ie.,
(x 2
)
+ y2 + z2 1 - x . 2 x
=
y2 + z2 − x2
(x 2
+ y2 + z2 )
2
(x 2
+ y2 + z2 )
2
y

∂ 2u y2 + z2 − x2
ud

∴ = …(1)
∂x 2 (
x2 + y2 + z2
2
)
∂ 2u z2 + x 2 − y2
st

Similarly ∴ = …(2)
∂y 2 (
x2 + y2 + z2
2
)
∂ 2u x 2 + y2 − z 2
= …(3)
∂z 2 (
x2 + y2 + z2
2
)
Adding (1), (2) and (3) we get,
∂ 2u ∂ 2u ∂ 2u x 2 + y2 + z2 1
+ + = = 2
∂x 2
∂y 2
∂z 2
x2 + y2 + z2 (
2
x + y2 + z 2 )
( )
⎛ ∂ 2u ∂ 2u ∂ 2u ⎞
Thus x 2 + y 2 + z 2 ⎜⎜ 2 + 2 + 2 ⎟⎟ = 1
⎝ ∂x ∂y ∂z ⎠

Compiled by www.studyeasy.in
Compiled by www.studyeasy.in

2011
4. If u = log (tan x + tan y + tan z), show that,
sin 2x ux + sin 2y uy + sin 2z uz = 2
Solution : u = log (tan x + tan y + tan z) is a symmetric function.
sec 2 x
ux =
tan x + tan y + tan z

(2 sin x cos x ) sec2 . x


sin 2 x u x =
tan x + tan y + tan z
2 tan x
Or sin 2 x u x = …(1)
tan x + tan y + tan z
2 tan y
Similarly sin 2 y u y = …(2)
tan x + tan y + tan z

.in
2 tan z
sin 2 z u y = …(3)
tan x + tan y + tan z
Adding (1), (2) and (3) we get, sy 2 (tan x + tan y + tan z )
sin 2 x u x + sin 2y u y + sin 2z u z = =2
(tan x + tan y + tan z )
ea
Thus sin 2x u x + sin 2y u y + sin 2z u z = 2
y

∂u ∂u ∂u
ud

3
5. If u = log (x3 + y3 + z3 – 3xyz) then prove that + + = and hence show
∂x ∂y ∂ z x + y + z
2
⎛∂ ∂ ∂⎞
st

-9
that ⎜⎜ + + ⎟⎟ u =
⎝ ∂x ∂y ∂z ⎠ (x + y + z )2
Solution : u = log (x3 + y3 + z3 – 3xyz) is a symmetric function
∂u 3x 2 - 3yz
= 3 …(1)
∂x x + y3 + z 3 - 3xyz

∂u 3 y 2 - 3zx
= 3 …(2)
∂y x + y3 + z 3 - 3xyz

∂u 3z 2 - 3xy
= 3 …(3)
∂z x + y3 + z 3 - 3xyz
Adding (1), (2) and (3) we get,

Compiled by www.studyeasy.in
Compiled by www.studyeasy.in

2011

+ + =
(
∂u ∂u ∂u 3 x 2 + y 2 + z 2 − xy − yz − zx )
∂x ∂y ∂z (
x 3 + y3 + z 3 − 3xyz )
Recalling a standard elementary result,
(
a 3 + b 3 + c 3 − 3abc = (a + b + c ) a 2 + b 2 + c 2 − ab − bc − ca )
We have,
∂ u ∂ u ∂u
+ + =
(
3 x 2 + y 2 + z 2 − xy − yz − zx )
(
∂x ∂y ∂z ( x + y + z ) x 2 + y 2 + z 2 − xy − yz − zx )
∂u ∂u ∂ u 3
Thus + + =
∂ x ∂y ∂z x + y + z
2
⎛ ∂ ∂ ∂ ⎞
Further ⎜⎜ + + ⎟⎟ u
⎝ ∂x ∂y ∂z ⎠

.in
⎛ ∂ ∂ ∂ ⎞ ⎛ ∂ ∂ ∂ ⎞
= ⎜⎜ + + ⎟⎟ ⎜⎜ + + ⎟⎟ u
⎝ ∂x ∂y ∂z ⎠ ⎝ ∂x ∂y ∂z ⎠
sy
⎛ ∂ ∂ ∂ ⎞ ⎛ ∂u ∂u ∂u ⎞
= ⎜⎜ + + ⎟⎟ ⎜⎜ + + ⎟⎟
⎝ ∂x ∂ y ∂ z ⎠ ⎝ ∂x ∂y ∂z ⎠
ea
⎛ ∂ ∂ ∂ ⎞ ⎛ 3 ⎞
= ⎜⎜ + + ⎟⎟ ⎜⎜ ⎟⎟ , by using the earlier result.
⎝ ∂x ∂ y ∂ z ⎠ ⎝ x+ y+ z⎠
y

∂ ⎛ ⎞ ∂ ⎛ ⎞ ∂ ⎛ ⎞
ud

3 3 3
= ⎜⎜ ⎟⎟ + ⎜⎜ ⎟⎟ + ⎜⎜ ⎟
∂x ⎝ x + y + z ⎠ ∂y ⎝ x + y + z ⎠ ∂z ⎝ x + y + z ⎟⎠
−3 −3 −3 −9
st

= + + =
(x + y + z) 2
(x + y + z)
2
(x + y + z) 2
( x + y + z )2
2
⎛ ∂ ∂ ∂ ⎞ -9
Thus ⎜⎜ + + ⎟⎟ u =
⎝ ∂x ∂y ∂z ⎠ ( x + y + z )2

Compiled by www.studyeasy.in
Compiled by www.studyeasy.in

2011

6. If u = f (r ) and x = r cos θ, y = r sin θ ,

∂ 2u ∂ 2u
prove that 2 + 2 = f ′′ (r ) + f ′ (r )
1
∂x ∂y r
Solution :Observing the required partial derivative we conclude that u must be a function of x, y. But
u = f( r) by data and hence we need to have r as a function of x, y. Since x = r cos θ, y = r
sin θ we have x2 + y2 = r2.

∴ we have u = f (r ) where r = x 2 + y2

∂ 2 u f ′ (r ) 2 2 f ′′ (r )
∂x 2
r
(
= 3 r - x + 2 . x 2 and
r
)

.in
∂ 2 u f ′(r ) 2 2 f ′′ (r ) 2
∂y 2
=
r3
r - y (
+
r2
.y)
Adding these results we get,
∂ 2 u ∂ 2 u f ′ (r )
{ f ′′(r )
(
sy
)} ( )
+ = 3 2x2 - x2 + y2 + 2 x2 + y2
∂x ∂y
ea
2 2
r r
f ′(r ) 2 f ′′(r ) 2 1
= . r + 2 . r = f ′ (r ) + f ′′ (r )
r3 r r
y

∂ 2u ∂ 2u
+ 2 = f ′′ (r ) + f ′ (r )
1
ud

Thus
∂x 2
∂y r
∂u ∂u
7. Prove that x +y = nu
st

∂x ∂y

Proof : Since u = f (x, y) is a homogeneous function of degree n we have by the definition,


u = x n g ( y/x ) …(1)
Let us differentiate this w.r.t x and also w.r.t.y
∂u ⎛ y ⎞
∴ = x n . g′ ( y/x ) . ⎜ - 2 ⎟ + nx n - 1 g ( y/x )
∂x ⎝ x ⎠
∂u
ie., = x n - 2 y g′ ( y/x ) + nx n - 1 g ( y / x ) …(2)
∂x
∂u ⎛1⎞
Also = x n . g′ ( y/x ) . ⎜ ⎟
∂y ⎝ x⎠

Compiled by www.studyeasy.in
Compiled by www.studyeasy.in

2011
∂u
ie., = x n - 1 . g′ ( y/x ) …(3)
∂y
∂u ∂u
Now consider x +y as a consequence of (2) and (3)
∂x ∂y

[ ] [ ]
= x - x n - 2 y g′ ( y/x ) + n x n - 1 g ( y/x ) + y x n - 1 g′ ( y / x )

= − x n −1 y g′ ( y/x ) + n x n g ( y/x ) + x n - 1 y g′ ( y/x )

= n . x n g ( y/x )
= n u, by using (1)
Thus we have proved Euler’s theorem
∂u ∂u
x +y = n u ; x u x + yu y = n u
∂x ∂y

.in
∂ 2u ∂ 2u
8. Prove that x 2 + 2 x y = n (n - 1) u
∂x 2 ∂x ∂y
sy
Proof : Since u = f (x, y) is a homogeneous function of degree n, we have Euler’s theorem
∂u ∂u
+y =nu
ea
x …(1)
∂x ∂y
Differentiating (1) partially w.r.t. x and also w.r.t y we get,
y

⎛ ∂ 2u ∂u ⎞ ∂ 2u ∂u
⎜ x 2 + 1. ⎟+ y = n …(2)
ud

⎜ ∂x ∂x ⎟⎠ ∂x ∂ y ∂x

∂ 2u ⎛ ∂ 2u ∂u ⎞ ∂u
Also, x + ⎜⎜ y 2 + 1 . ⎟=n
⎟ …(3)
st

∂y ∂x ⎝ ∂y ∂y ⎠ ∂y

We shall now multiply (2) by x and (3) by y.


∂ 2u ∂u ∂ 2u ∂u
x2 + x + x y =nx and
∂x 2
∂x ∂x∂y ∂x

∂ 2u ∂ 2u ∂u ∂u
xy + y2 2 + y = ny
∂y ∂x ∂y ∂y ∂y

∂ 2u ∂ 2u
Adding these using the fact that = we get,
∂y ∂x ∂y ∂x

Compiled by www.studyeasy.in
Compiled by www.studyeasy.in

2011
⎛ 2 ∂ 2u ∂ 2u 2 ∂ u ⎞ ⎛ ∂u
2
∂u ⎞ ⎛ ∂u ∂u ⎞
⎜x + 2 xy + y ⎟ + ⎜⎜ x + y ⎟ = n ⎜ x + y ⎟
⎜ ∂x

2
∂x∂y ∂y ⎟⎠ ⎝ ∂x
2 ⎟
∂y ⎠ ⎜
⎝ ∂x ∂y ⎟⎠

∂ 2u ∂ 2u 2 ∂ u
2
ie., x 2 + 2 x y + y + n u = n (n u ), by using (1)
∂x 2 ∂x∂y ∂y 2

∂ 2u ∂ 2u 2 ∂ u
2
or x 2
+2xy +y + n (nu ) - nu = n (n - 1) u
∂x 2 ∂x∂y ∂y 2

∂ 2u ∂ 2u 2 ∂ u
2
Thus x 2
+2xy +y + n (n - 1) u
∂x 2 ∂x∂y ∂y 2

ie., x 2 u xx + 2 x y u xy + y 2 u yy = n (n - 1) u

x y z ∂u ∂u ∂u
9. If u = + + show that x +y +z =0

.in
y+z z+ x x+y ∂x ∂y ∂z
Solution : (Observe that the degree is 0 in every term)

u=
x
+
y
+
z
y+ z z+ x x+ y
sy
ea
We shall divide both numerator and denominator of every term by x.

= x {g ( y/x, z/x )}
1 y/ x z
u= + +
y/x + z/x z / x + 1 1 + y / x
y

⇒ u is homogeneous of degree 0. ∴ n = 0
ud

∂u ∂u ∂u
We have Euler’s theorem, x +y +z =nu
∂x ∂y ∂z
st

∂u ∂u ∂u
Putting n = 0 we get, x +y +z =0
∂x ∂y ∂z

⎛ x4 + y4 ⎞ ∂u ∂u
10. If u = log ⎜⎜ ⎟⎟ show that x +y =3
⎝ x+ y ⎠ ∂x ∂y

Solution : we cannot put the given u in the form xn g (y/x)

∴e =
(
x4 + y4 x4 1 + y4 / x4
u
= =x ⎨
)
3 ⎪1 + ( y/x ) ⎪
⎧ 4⎫

x+ y x (1 + y/x ) ⎪⎩ 1 + ( y / x ) ⎪⎭

ie., eu = x3 g (y/x) ⇒ eu is homogeneous of degree 3 ∴ n = 3


Now applying Euler’s theorem for the homogeneous function eu

Compiled by www.studyeasy.in
Compiled by www.studyeasy.in

2011

We have x
( )
∂ eu
+y
∂ eu( )
= n eu
∂x ∂y
∂u ∂u
ie., x e u + y eu = 3e u
∂x ∂y
∂u ∂u
Dividing by eu we get x +y =3
∂x ∂y

⎛ x3 + y3 ⎞
11. If u = tan -1 ⎜⎜ ⎟⎟ show that
⎝ x− y ⎠
(i) x ux + y uy = sin 2 u
(ii) x2uxx + 2 x y uxy + y2uyy = sin 4 u – sin 2 u
⎛ x 3 + y3 ⎞
Solution : (i) u = tan ⎜⎜ ⎟ by data

.in
-1

⎝ x - y ⎠

⇒ tan u = =
(
x3 + y3 x3 1 + y3 / x3 )
sy ⎧⎪1 + ( y/x )3 ⎫⎪
= x2 ⎨ ⎬
x-y x (1 - y/x ) ⎪⎩1 − ( y / x ) ⎪⎭

ie., tan u = x2/g (y/x) ⇒ tan u is homogeneous of degree 2.


ea
Applying Euler’s theorem for the function tan u we have,
∂ (tan u ) ∂ (tan u )
+y = n . tan u ; n = 2
y

x
∂x ∂y
ud

∂u ∂u
ie., x sec 2 u + y sec 2 u = 2 tan u
∂x ∂y
st

∂u ∂u 2 tan u sin u
or x +y = = 2 cos 2 u = 2 cos u sin u = sin 2 u
∂x ∂y 2
sec u cos u
∴ xu x + yu y = sin 2u

(ii) We have xux + y uy = sin 2 u …(1)


Differentiating (1) w.r.t x and also w.r.t y partially we get
x u xx + 1 . u x + yu xy = 2 cos 2u . u x …(2)

And x uyx + yuyy + 1 . uy = 2 cos 2u . uy …(3)


Multiplying (2) by x and (3) by y we get,
x 2 u xx + x u x + xy u xy = 2 cos 2 u. xu x

Compiled by www.studyeasy.in
Compiled by www.studyeasy.in

2011
xy u yx + y 2 u yy + y u y = 2 cos 2 u. yu y

Adding these by using the fact that uyx = uxy, we get


( ) (
x 2 u xx + 2 x y u xy + y 2 u yy + xy x + yu y = 2 cos 2u xu x + y u y )
By using (1) we have,
x 2 u xx + 2 x y u xy + y 2 u yy = 2 cos 2u sin 2 u - sin 2u

(since sin 2θ = 2 cos θ sin θ, first term in the R.H.S becomes sin 4u)
Thus x2uxx + 2 x y uxy + y2 uyy = sin 4 u – sin 2u
⎛x y z⎞ ∂u ∂u ∂u
12. If u = f ⎜⎜ , , ⎟⎟ Prove that x +y +z =0
⎝y z x⎠ ∂x ∂y ∂z
>> here we need to convert the given function u into a composite function.

.in
Let u = f (p, q, r ) where p =
x y z
,q= ,r =
y z x

∂u ∂ u ∂p ∂ u ∂ q ∂u ∂ r
sy
ie., {u → (p, q, r ) → (x, y, z )} ⇒ u → x, y, z

∴ = + +
ea
∂x ∂ p ∂x ∂q ∂x ∂r ∂x
∂u ∂u 1 ∂u ∂u ⎛ z ⎞
ie., = . + . 0+ .⎜- ⎟
∂x ∂p y ∂q ∂r ⎝ x 2 ⎠
y

∂u x ∂u z ∂u
ud

∴ x = - …(1)
∂x y ∂p x ∂r
Similarly by symmetry we can write,
st

∂u y ∂u x ∂u
y = - …(2)
∂ y z ∂ q y ∂p
∂ u z ∂ u y ∂u
z = - …(3)
∂z x ∂r z ∂q
∂u ∂u ∂u
Adding (1), (2) and (3) we get x +y +z =0
∂x ∂y ∂z

Compiled by www.studyeasy.in
Compiled by www.studyeasy.in

2011
∂u ∂u ∂u
13. If u = f(x – y, y – z, z – x) show that + + =0
∂x ∂y ∂z
>> Let u = f(p, q, r) where p = x – y, q = y – z, r = z – x
∂ u ∂ u ∂p ∂u ∂ q ∂u ∂r
∴ = + +
∂ x ∂ p ∂x ∂ q ∂x ∂ r ∂ x
∂u ∂u ∂u ∂u
ie., = .1+ .0 + (- 1)
∂x ∂p ∂q ∂r
∂u ∂u ∂ u
∴ = - …(1)
∂ x ∂p ∂ r
Similarly we have by symmetry
∂u ∂u ∂u
= - …(2)
∂ y ∂ q ∂p

.in
∂ u ∂u ∂u
= - …(3)
∂z ∂r ∂ q

Adding (1), (2) and (3) we get,


∂ u ∂u
+ +
sy
∂u
=0
∂ x ∂y ∂z
ea

14. If z = f(x, y) where x = r cos θ and y = r sin θ


y

2
⎛ ∂z ⎞ ⎛ ∂z ⎞ ⎛ ∂z ⎞
2 2 2
1 ⎛ ∂z ⎞
Show that ⎜ ⎟ + ⎜⎜ ⎟⎟ = ⎜ ⎟ + 2 ⎜ ⎟
ud

⎝ ∂ x ⎠ ⎝ ∂y ⎠ ⎝ ∂ r ⎠ r ⎝ ∂θ ⎠
Solution : {z → ( x, y ) → (r, θ ) } ⇒ z → (r, θ )
st

∂z ∂z ∂ x ∂z ∂y ∂ z ∂x ∂z ∂ y
∴ = + ; +
∂ r ∂x ∂r ∂ y ∂ r ∂ x ∂θ ∂ y ∂θ
∂z ∂ z ∂z
ie., = cos θ + sin θ …(1)
∂r ∂x ∂y

∂z ∂ z ⎡ ⎤
and = ( - r sin θ) + ∂z (r cos θ) = r ⎢ - ∂z sin θ + ∂z cos θ⎥
∂θ ∂ x ∂y ⎣ ∂x ∂y ⎦
1 ∂z − ∂z ∂z
or = = sin θ + cos θ
r ∂θ ∂x ∂y
squaring and adding (1), (2) and collecting suitable terms have,

Compiled by www.studyeasy.in
Compiled by www.studyeasy.in

2011

[ ]
2 2 2
⎛ ∂z ⎞ 1 ⎛ ∂z ⎞ ⎛ ∂ z ⎞
⎜ ⎟ + 2 ⎜ ⎟ = ⎜ ⎟ cos θ + sin θ
2 2
⎝ ∂r ⎠ r ⎝ ∂θ ⎠ ⎝ ∂x ⎠

[ ]
2
⎛ ∂z ⎞ ∂z ∂z ∂z ∂z
+ ⎜⎜ ⎟⎟ sin 2 θ + cos 2 θ + 2 cos θ sin θ - 2 sin θ cos θ
⎝ ∂y ⎠ ∂x ∂ y ∂x ∂y
2
1 ⎛ ∂z ⎞ ⎛ ∂z ⎞ ⎛ ∂z ⎞
2 2 2
⎛ ∂z ⎞
∴ ⎜ ⎟ ⎜ ⎟ = ⎜ ⎟ + ⎜ ⎟ ie., R.H.S = L.H.S
⎝ ∂r ⎠ r 2 ⎝ ∂θ ⎠ ⎝ ∂x ⎠ ⎜⎝ ∂y ⎟⎠

15. If z = f (x, y ) where x = e u + e − v , y = e -u − e v


∂z ∂z ∂z ∂z
Prove that x −y = −
∂x ∂y ∂u ∂v
Solution : {z → (x, y ) → (u, v )} ⇒ z → (u, v )

.in
∂ z ∂ z ∂ x ∂ z ∂ y ∂ z ∂ z ∂x ∂ z ∂ y
∴ = + ; = +
∂ u ∂ x ∂ u ∂y ∂u ∂ v ∂ x ∂ v ∂ y ∂ v
∂z ∂z ∂z
ie., =
∂u ∂x
. eu +
∂y
- e -u ( ) sy …(1)

( )
∂ z ∂z - v ∂ z
( )
ea
= -e + - e-v …(2)
∂ v ∂x ∂y
∂ z ∂z
Consider R.H.S = − and (1) – (2) yields
y

∂ u ∂v
ud

∂z u
∂x
(
e + e-v -
∂z - u
∂y
)
e − ev =
∂z
∂x
.x-(∂z
∂y
. y )
∂z ∂z ∂z ∂z
st

Thus - =x −y ie., R.H.S = L.H.S


∂u ∂y ∂x ∂y
∂ (u , v, w )
16. Find where u = x2 + y2 + z2, v = xy+yz+zx, w=x+y+z
∂ (x , y, z )

∂u ∂u ∂u
∂x ∂y ∂z
∂ (u , v, w ) ∂v ∂v ∂v
Solution : The definition of J = =
∂ (x , y, z ) ∂x ∂y ∂z
∂w ∂w ∂w
∂x ∂y ∂z

But u = x2 + y2 + z2, v = xy+yz+zx, w=x+y+z

Compiled by www.studyeasy.in
Compiled by www.studyeasy.in

2011

Substituting for the partial derivatives we get

2x 2y 2z
J= y+z x+z y+x
1 1 1

Expanding by the first row,


J = 2x {(x + z) – (y + x)} -2y {(y + z) – (y + x)}
+2z {(y + z) – (x + z)}
= 2x (z-y) – 2y(z-x) + 2z(y-x)
= 2(xz – xy – yz + xy + yz – xz) = 0 Thus J = 0

.in
yz zx xy ∂ (u , v, w )
17. If u = ,v= ,w= , show that =4
x y z ∂ (x , y, z )

Solution : by data u =
yz
x
,v=
zx
y
sy
,w=
xy
z
ea
∂u ∂u ∂u − yz z y
∂x ∂y ∂z x2 x x
∂ (u , v, w ) ∂v ∂v ∂v z − zx x
= =
y

∂ (x , y, z ) ∂x ∂y ∂z y y2 y
∂w ∂w ∂w − xy
ud

y x
∂x ∂y ∂z z z z2

− yz ⎧⎛ − zx ⎞ ⎛ − xy ⎞ ⎛ x ⎞ ⎛ x ⎞⎫
st

= ⎨⎜⎜ 2 ⎟⎟ ⎜ 2 ⎟ − ⎜ ⎟ ⎜⎜ ⎟⎟⎬
x2 ⎩⎝ y ⎠ ⎝ z ⎠ ⎝ z ⎠ ⎝ y ⎠⎭

z ⎧ z ⎛ − zx ⎞ y x ⎫ y ⎧ z x y ⎛ − zx ⎞⎫
- ⎨ ⎜ ⎟− ⋅ ⎬+ ⎨ ⋅ − ⎜ ⎟⎬
x ⎩ y ⎜⎝ y 2 ⎟⎠ z y ⎭ x ⎩ y z z ⎜⎝ y 2 ⎟⎠⎭

− yz ⎧ x 2 x 2 ⎫ z ⎧ − x x ⎫ y ⎧ x x⎫
= 2 ⎨ − ⎬− ⎨ − ⎬+ ⎨ + ⎬
x ⎩ yz yz ⎭ x ⎩ z z ⎭ x ⎩ y y⎭

= 0+1+1+1+1=4

∂ (u , v, w )
Thus =4
∂ (x , y, z )

Compiled by www.studyeasy.in
Compiled by www.studyeasy.in

2011
18. If u + v = ex cos y and u – v = ex sin y find the jacobian of the functions u and v w.r.t x
and y.

∂u ∂u
∂ (u , v ) ∂x ∂y
Solution : we have to find =
∂ (x , y ) ∂v ∂v
∂x ∂y

Using the given data we have to solve for u and v in terms of x and y.

By data u + v = ex cos y ……(1)


u – v = ex sin y ……(2)

.in
(1) + (2) gives : 2 u = ex (cos y + sin y)
(2) – (2) gives : 2 v = ex (cos y – sin y)

Ie., u =
ex
(cos y + sin y) ; v =
ex
sy
(cos y – sin y)
2 2
ea
∂u ex ∂v e x
∴ = (cos y + sin y), = (- sin y - cos y)
∂x 2 ∂x 2
y

ex ex
∂ (u , v ) (cos y + sin y) ( − sin y + cos y)
ud

Now = 2x 2
∂ ( x, y ) e − ex
(cos y − sin y) (sin y + cos y)
2 2
st

ex ex
= . { - ( cos y + sin y)2 – (cos y – sin y)2}
2 2
− e2x − e2x
= {1+sin 2y) + (1 – sin 2y)} =
4 2
∂ (u , v ) − e 2 x
Thus =
∂ (x , y ) 2

Compiled by www.studyeasy.in
Compiled by www.studyeasy.in

2011

∂ (r, θ)
19. (a) If x = r cos θ , y = r sin θ find the value of
∂ (x , y )

∂ (x , y ) ∂ (r, θ)
(b) Further verify that . =1
∂ (r, θ) ∂ (x , y )

(a) Solution : We shall first express r, θ in terms of x and y.


We have x = r cos θ , y = r sin θ by data.

y
∴ x2 + y2 = r2 and = tan θ or θ = tan-1 (y/x)
x

.in
Consider r2 = x2 + y2
Differentiating partially w.r.t x and also w.r.t y we get,

2r
∂r
∂x
= 2x and 2r
sy
∂r
∂y
= 2y
ea
∂r x ∂r y
∴ = and =
∂x r ∂y r

Also consider θ = tan −1 ( y / x )


y

∂θ −y ∂θ
ud

1 1 1
∴ = . 2 and = .
∂ x 1 + (y / x) 2
x ∂ y 1 + (y / x) 2
x

∂θ −y ∂θ x
i.e., = 2 and = 2
st

∂ x x + y2 ∂ y x + y2

∂r ∂r x y
∂ (r ,θ ) ∂r dy r r
Now = =
∂ ( x, y ) ∂θ ∂θ −y x
dx dy x + y2
2
x + y2
2

x2 y2 (x2 + y 2 ) 1
i.e., = + = =
r(x 2 + y 2 ) r(x 2 + y 2 ) r(x 2 + y 2 ) r
∂ (r , θ ) 1
∴ =
∂ ( x, y ) r

Compiled by www.studyeasy.in
Compiled by www.studyeasy.in

Solution (b) : Consider x = r cos θ , y = r sin θ


∂x ∂x
∂ ( x, y ) ∂r ∂θ cosθ − r sin θ
= = = r (cos 2 θ + sin 2 θ ) = r
∂(r , θ ) ∂y ∂y sin θ r cosθ
∂r dθ
∂ ( x, y )
∴ =r
∂(r , θ )

∂ ( x, y ) ∂ (r ,θ ) 1
From (1) and (2) : ⋅ = r ⋅ =1
∂ ( x,θ ) ∂ ( x, y ) r

.in
20. If x = u (1 − v ), y = uv then show that JJ / = 1

Solution : x = u (1 − v); y = uv
sy
ea
∂x ∂y ∂x ∂y
= (1 − v), =v = −u , =u
∂u ∂u ∂v ∂v
y

∂x ∂x
ud

∂ ( x, y ) ∂u ∂v (1 − v) − u
J= = =
∂(u, v) ∂y ∂y v u
st

∂u ∂v
= (1 − v)u + uv = u ∴ J =u
Next we shall express u and v in terms of x and y.
By data x = u - uv and y = uv

y y
Hence x + y = u. Also v = =
u x+ y
y ∂u ∂u
Now we have, u = x + y; v = ∴ = 1, = 1,
x+ y ∂x ∂y

Compiled by www.studyeasy.in
Compiled by www.studyeasy.in

2011
∂v ( x + y ) ⋅ 0 − y ⋅ 1 x
= =
∂x ( x + y)2 ( x + y) 2

∂u ∂u
1 1
∂ (u , v) ∂x ∂y
∴J/ = = = −y x
∂ ( x, y ) ∂v ∂v
( x + y) 2 ( x + y) 2
∂x ∂y
x y x+ y 1 1
= + = = =
( x + y) 2
( x + y) 2
( x + y) 2
( x + y) u
1 1
Thus J / = Hence J ⋅ J / = u ⋅ Thus JJ / = 1
u u

21. State Taylor’s Theorem for Functions of Two Variables.

.in
Statement: Considering f (x + h, y + k) as a function of a single variable x, we have by
Taylor’s Theorem

f (x + h, y + k) = f (x, y + k) + h
sy
∂f ( x, y + k ) h 2 ∂ 2 f ( x, y + k )
+ + ---------(1)
∂x 2! ∂x 2
ea
Now expanding f (x, y + k) as function of y only,

∂f ( x, y ) k 2 ∂ 2 f ( x, y )
y

f (x, y + k) = f (x, y) + k + + --------


∂y 2! ∂y 2
ud

∂f ( x, y ) k 2 ∂ 2 f ( x, y )
∴ (i) takes the form f (x + h, y + k) = f (x,y) + k + +-------------------------------- +
∂y ∂y 2
st

2!
∂ ⎧ ∂f ( x, y ) k 2 ∂ 2 f ( x, y ) ⎫
h ⎨ f ( x, y ) + k + + − − − − −⎬
∂x ⎩ ∂y 2! ∂y 2

h2 ∂2 ⎧ ∂ ( x, y ) ⎫
+ ⎨ f ( x , y ) + k + − − − − − − − − ⎬
2! ∂x 2 ⎩ ∂y ⎭

∂f ∂f 1 ⎛ 2 ∂ 2 f ∂2 f 2 ∂ f
2

Hence f (x + h, y + k) = f (x , y) +h +k + ⎜⎜ h + 2 hk + k ⎟⎟ + − − (1)
∂x ∂y 2! ⎝ ∂x 2 ∂x∂y ∂y 2 ⎠

In symbol we write it as

Compiled by www.studyeasy.in
Compiled by www.studyeasy.in

2
⎛ ∂ ∂⎞ 1⎛ ∂ ∂⎞
F (x + h, y + k) = f (x,y) + ⎜⎜ h + k ⎟⎟ f + ⎜⎜ h + k ⎟⎟ f + − − − −
⎝ ∂x ∂y ⎠ 2! ⎝ ∂x ∂y ⎠

Taking x = a and y = b, (1) becomes

1
f (a + h, b + k) = f (a,b) + [h f x (a,b) + kf y (a,b)] + [h 2 f xx (a,b)+ 2hkf xy (a,b)
2!

+ k 2 f yy (a,b)] + ---------

Putting a + h = x and b + k = y so that h = x – a, k = y – b, we get

F (x,y) = f (a,b) + [(x – a) f x (a,b) + (y – b) f y (a,b)]

1
= [(x – a)2 f xx (a,b) + 2 (x – a) (y – b) f xy (a,b) + (y – b)2 f yy (a,b)] +------ (2)

.in
2!

This is Taylor’s expansion of f (x,y) in powers of (x – a) and (y – b). It is used to expand f (x,y)
in the neighborhood of (a,b)

corollary, putting a = 0, b = 0 in (2), we get


sy
ea
1
f (x,y) = f (0,0) + [x f x (0,0) + y f y (0,0)] + [ x 2 f xx (0,0) + 2xy f xy (0,0)
2!
+ y 2 f yy (0,0) ] + --------- (3)
y

This is Maclaurin’s Expansion of f (x,y)


ud

22. Expand e x log (1 + y) in powers of x and y up to terms of third degree.

Solution: Here
st

f (x,y) = e x log (1 + y) ∴ f (0,0) = 0

f x (x,y) = e x log (1 + y) ∴ f x (0,0) = 0

1
f y (x,y) = e x ∴ f y (0,0) = 1
1+ y

f xx (x,y) = e x log (1 + y) ∴ f xx (0,0) = 0

1
f xy (x,y) = e x ∴ f xy (0,0) = 1
1+ y

Compiled by www.studyeasy.in
Compiled by www.studyeasy.in

2011
f yy (x,y) = - e x (1 + y) -2 ∴ f yy (0,0) = -1

f xxx (x,y) = e x log (1 + y) ∴ f xxx (0,0) = 0

1
f xxy (x,y) = e x ∴ f xxy (0,0) = 1
1+ y

f xyy (x,y) = -e x (1 + y) 2 ∴ f xyy (0,0) = -1

f yyy (x,y) = 2e x (1 + y)-3 ∴ f yyy (0,0) = 2

Now, Maclaurin’s expansion of f (x,y) gives


1
f (x,y) = f (0,0) + x (f x (0,0) + y fy (0,0) + {x2 fxx (0,0) + 2xy fxy (0,0) +
2!
1
y2 fyy (0,0)} + {x3 fxxx (0,0) + 3x2 y fxxy (0,0) + 3xy2 fxyy (0,0) + y3 fyyy (0,0)} + -----------

.in
2!

1
∴ex log (1 + y) = 0 + x.0 + y (1) + {x2.0 + 2xy (1) + y2 (-1)}
1 3
2!
+
2! sy
{x .0 + 3x2y (1) + 3xy2 (-1) + y3(2)}+---------
ea
1 1 1
= y + xy - y2 + (x2y = xy2) + y3 + ----------
2 2 2
y

⎛ π⎞
23. Expand f (x,y) = ex Cosy by Taylor’s Theorem about the point ⎜1, ⎟ up to the Second
⎝ 4⎠
ud

degree terms.
π ⎛ π⎞
Solution: f (x,y) = ex Cosy and a = 1, b = ∴ f = ⎜1, ⎟ = e
⎝ 4⎠
st

4 2
⎛ π⎞ e
fx (x,y) = ex Cos y ∴ f ⎜1, ⎟ =
⎝ 4⎠ 2
⎛ π⎞ e
fy (x,y) = -ex Sin y ∴ fy ⎜1, ⎟ = -
⎝ 4⎠ 2
⎛ π⎞ e
fxx(x,y) = ex Cos y ∴ fxx ⎜1, ⎟ =
⎝ 4⎠ 2
⎛ π⎞ e
fxy (x,y) = -ex Sin y ∴ fxy ⎜1, ⎟ = -
⎝ 4⎠ 2
⎛ π⎞ e
fyy (x,y) = - ex Cos y ∴ fyy ⎜1, ⎟ = -
⎝ 4⎠ 2
Hence by Taylor’s Theorem, we obtain

Compiled by www.studyeasy.in
Compiled by www.studyeasy.in

2011

⎛ π⎞ ⎡ ⎛ π⎞ ⎤
f (x,y) = f ⎜1, ⎟ + ⎢( x − 1) f x + ⎜ y − ⎟ fy ⎥ +
⎝ 4⎠ ⎣ ⎝ 4⎠ ⎦
1 ⎡ ⎛ π⎞
2

⎢ x− f x + x − ⎜ y − ⎟ f yy ⎥ + -------------
2
( 1) 2( 1 )
2! ⎣⎢ ⎝ 4⎠ ⎦⎥
e ⎡ e ⎛ π ⎞⎛ e ⎞⎤ 1
i.e., ex Cosy = + ⎢( x − 1) + ⎜ y − ⎟⎜ − ⎟⎥ +
2 ⎣ 2 ⎝ 4 ⎠⎝ 2 ⎠⎦ 2!
⎡ π ⎞⎛ e ⎞ ⎛ π ⎞ ⎛ − e ⎞⎤
2
e ⎛
⎢( x − 1) + 2( x − 1)⎜ y − ⎟⎜ − ⎟+⎜y− ⎟ ⎜ ⎟⎥ + ------------
2

⎣⎢ 2 ⎝ 4 ⎠⎝ 2⎠ ⎝ 4 ⎠ ⎝ 2 ⎠⎦⎥

e ⎡ π ⎞⎤ 1 ⎡ π⎞ ⎛ π⎞ ⎤
2
⎛ ⎛
ex Cosy = ⎢1 + ( x − 1) − ⎜ y − 4 ⎟⎥ + 2! ⎢( x − 1) − 2( x − 1)⎜ y − 4 ⎟ − ⎜ y − 4 ⎟ ⎥ + -------}
2

2 ⎣ ⎝ ⎠⎦ ⎢⎣ ⎝ ⎠ ⎝ ⎠ ⎥⎦

.in
Exercise:

1) Expand exy up to Second degree terms by using Maclaurin’s theorem


sy
2) Expand Log (1 – x – y ) up to Third degree terms by using Maclaurin’s theorem
ea
3) Expand x2y about the point (1,-2) by Taylor’s expansion

4) Obtain the Taylor’s expansion of ex Siny about the point 0, π ( 2


) up to Second degree terms
y

5) Expand esinx up to the term containing x4


ud
st

Compiled by www.studyeasy.in
Compiled by www.studyeasy.in

2011

Maxima and Minima:-


In mathematics, the maximum and minimum (plural: maxima and minima) of a
function, known collectively as extrema (singular: extremum), are the largest and
smallest value that the function takes at a point within a given neighborhood.

A function f (x, y) is said to have a Maximum value at (a,b) if their exists a


neighborhood point of (a,b) (say (a+h, b+k)) such that f (a, b) > f (a+h, b+k).
Similarly,
Minimum value at (a,b) if there exists a neighborhood point of (a,b) (say (a+h,
b+k)) such that f (a, b) < f (a+h, b+k).

A Minimum point on the graph (in red) f ( x, y ) = x 2 + y 2 (1 − x)3

.in
sy
y ea
ud

A Maximum point on the graph is at the top (in red)


st

Compiled by www.studyeasy.in
Compiled by www.studyeasy.in

2011

ddle point on
A sad n the graph of z=x2−y2 (in red)

.in
Saddle point betw
ween two hillss.
sy
y ea
ud
st

Neccessary an
nd Sufficcient Con
ndition:-

• If fx =0
= and fy =0
= (Necesssary Conddition)
• Functiion will be minimumm if AC-B2 > 0 and A > 0
• Functio
on will be maximum
m if AC-B2 > 0 and A < 0
• Functio
on will be neither
n maaxima nor minima
m if AC-B2 < 0
• If AC C-B2 = 0 we
w cannot make
m any conclusionn without any
a
further anaalysis
w
where A = f xx , B = f xy , C = f yy
A

Compiled by www.studyeasy.in
Compiled by www.studyeasy.in

2011

Working Procedure:-

• First we find Stationary points by considering


fx =0 and fy =0 .
• Function will be minimum if AC-B2 > 0 and A > 0 at that
stationary point
• Function will be maximum if AC-B2 > 0 and A < 0 at that
stationary point
• Function will be neither maximum nor minimum if AC-B2 < 0
at that stationary point and it is called as SADDLE POINT.

.in
25. Explain Maxima & Minima for Functions of Two Variables& hence obtain the Necessary
Conditions for Maxima, Minima.
sy
Solution: Let Z = f (x,y) be a given function of two independent variables x & y. The above equation
represents a surface in 3D.
Z P(a,b) f (a,b)
ea

Z = f (x,y)
y

A = f (a,b)
ud

Y
a O
N
st

b M(a,b)
X

A given points (a,b) on the surface has Co-ordinates [a,b, f (a,b)]

Compiled by www.studyeasy.in
Compiled by www.studyeasy.in

2011

Definition:
The function Z = f (x,y) is said to be a maximum at the point (a,b) if f (x,y) < f (a,b)
in the neighborhood of the point (a,b)

Cap
Z

Cup

O Y

(a,b)

.in
X
Definition:
sy
The function Z = f (x,y) is said to posses a minimum at the point (a,b) if f (x,y) > f
(a,b) in the neighborhood of the point (a,b)
ea
Necessary Condition for Maxima, Minima:
y

If Z = f (x,y) has a max or min at (a,b) then f x (a,b) = 0, fy (a,b) = 0


ud

Sufficient Conditions for Maxima, Minima:

Put R = fxx (a,b), S = fxy (a,b), T = fyy (a,b)


st

(1) Suppose S2 – RT > 0

There is no maxima or minima at (a,b)

(2) Suppose S2 – RT < 0

Thus there is maxima or minima at (a,b) according as R < 0 Or R > 0

(3) Suppose S2 – RT = 0, Then there is a saddle point at (a,b)

Compiled by www.studyeasy.in
Compiled by www.studyeasy.in

26. Find the maxima and minima of the functions f (x,y) = x3 + y3 – 3axy, a > 0 is constant.

Solution: Given f (x,y) = x3 + y3 – 3axy

fx = 3x2 – 3ay, fy = 3y2 – 3ax

fxx = 6x fyy = 6y.

Put fx = 0, fy = 0 and solve

i.e., 3x2 – 3ay = 0 & 3y2 – 3ax = 0

i.e., x2 = ay & y2 = ax
2
x2 ⎛ x2 ⎞
⇒y= ∴ ⇒ ⎜⎜ ⎟⎟ = ax (∵x2 = ay)

.in
a ⎝ a ⎠

x4 sy
∴ = ax
a2

∴ x4 = a3x
ea
i.e., x (x3 – a3) = 0
y

∴ x = 0, x = a
ud

⇒ y = 0, y = ± a

∴ The critical Or stationary points are (0,0), (a,a) and (a,-a)


st

(1) At (0,0)

R = fxx (0,0) = 0

S = fxy (0,0) = -3a

T = fyy (0,0) = 0

∴ S2 – RT = 9a2 – 0 = 9a2 > 0

∴ There is neither a maximum or a minimum at (0,0)

Compiled by www.studyeasy.in
Compiled by www.studyeasy.in

27. Examine the following functions for extreme values f = x4 + y4 – 2x2 + 4xy – 2y2

Solution:
fx = 4x3 – 4x + 4y

fy = 4y3 – 4x – 4y

fxy = 4, fxx = 12 x2 – 4, fyy = 12y2 – 4

Put fx = 0, fy = 0 and solve

i.e., 4x3 – 4x + 4y = 0 → (1)

4y3 + 4x – 4y = 0 → (2)

.in
Adding (1) & (2), we get

4 (x3 + y3) = 0 sy
i.e., x3 + y3 = 0
i.e., y = - x
ea
Substitute y = -x in (1), we get
y

4x3 – 4x – 4x = 0
ud

i.e., 4x3 – 8x = 0

i.e., x3 – 2x = 0 ⇒ x (x2 – 2) = 0
st

i.e., x = 0 & x2 – 2 =0

i.e., x = 0 & x = ± 2

x= 2,- 2

∴x = 0, 2 ,- 2 and corresponding values of y are y = 0, - 2, 2

∴ The critical points are (0,0), 2 ,- 2 , - 2, 2

(1) at (0,0)

R = fxx (0,0) = - 4

Compiled by www.studyeasy.in
Compiled by www.studyeasy.in

S = fxy (0,0) = 4

T = fyy (0,0) = -4

∴ S2 – RT = 16 – (-4) (-4) = 16 – 16 = 0

i.e., S2 – RT = 0, These is a saddle point at (0,0)

(2) at
2,- 2

R = fxx 2,- 2 =24– 4 = 20

S = fxy 2 ,- 2 =4

.in
T = fyy 2 ,- 2 = 20

∴ S2 – RT = 16 – (20) (20) = 16 – 400 = -384 < 0


sy
ea
Thus these is neither maximum nor minimum according to R < 0 or R > 0 at 2,- 2

Hence R = 20 > 0
y

∴ There is a minimum at 2 ,- 2
ud

∴ f min = ( 2 ) + (− 2 )
4 4
( )
−2 2
2
( ) (
+4 2 − 2 −2− 2 )
2
st

=4+4–4–8–4

=-8

(3) at - 2 , 2

(
R = fxx − 2 , 2 = 20 > 0 )
(
S = fxy − 2 , 2 = 4 )
(
T = fyy − 2 , 2 = 20 )
∴ S2 – RT = 16 – 400 = -384 < 0

Compiled by www.studyeasy.in
Compiled by www.studyeasy.in

(
Since R > 0, ∴ There is minima at − 2 , 2 )
(
∴ fmin = - 8 at − 2 , 2 )
( ) (
∴ Extreme Value = - 8 at − 2 , 2 & − 2 , 2 )
Exercise:

1) Find the extreme values of f = x3 y2 (1 – x – y)


2) Determine the maxima or minima of the function Sin x + Sin y + Sin (x + y)
3) Examine the function f(x,y) = 1+ sin( x2 + y2) for extremum.

28. If PV2 = K and if the relative errors in P is 0.05 and in V is 0.025 show that the error in K
is 10%.

.in
δP δV
Solution : PV 2 = K by data. Also = 0.05 and = 0.025
P V
⇒ log P + 2 log V = log K sy
ea
⇒ δ (log P ) + 2δ (log V ) = δ (log K )
y

1 1 1
i.e., δP + 2 ⋅ δV = δK
ud

p V K
δK δK
i.e., 0.05 + 2(0.025) = or = 0.1
K K
st

δK
∴ × 100 = (0.1) × 100 = 10
K

Thus the error in K is 10%.

Compiled by www.studyeasy.in
Compiled by www.studyeasy.in

2011
29. The time T of a complete oscillation of a simple pendulum is given by the formula
T = 2π l / g ⋅

(i) If g is a constant find the error in the calculated value of T due to an error of 3%
in the value of l.
(ii) Find the maximum error in T due to possible errors upto 1% in l
and 3% in g.

Solution :

δl
(i ) T = 2π l / g , g = Constant, × 100 = 3
l

.in
1
⇒ log T = log 2π + (log l − log g )
2
1
⇒ δ (log T ) = δ (log 2π ) + δ (log g ) sy
2
δT 1 δl
i.e., = 0+ −0
ea
T 2 l
δT 1 ⎛ δl ⎞ 1
or ×100 = ⎜ × 100 ⎟ = (3) = 1.5
T 2⎝ l ⎠ 2
y
ud

∴ the error in T = 1.5%.


st

(ii) If g is not a constant we have,


δT 1 ⎛ δl ⎞ 1 ⎛ δg ⎞
× 100 = ⎜ × 100 ⎟ − ⎜⎜ × 100 ⎟⎟
T 2⎝ l ⎠ 2⎝ g ⎠

The error in T will be maximum if the error in l is positive and the error in g is negative (or vice-
versa) as the difference in errors converts in to a sum.

⎛ δT ⎞ 1 1
∴ max ⎜ × 100 ⎟ = (+1) − (−3) = 2
⎝T ⎠ 2 2
∴ the maximum error in T is 2%.

Compiled by www.studyeasy.in
Compiled by www.studyeasy.in

2011

30. The current measured by a tangent galvanometer is given by the relation


c = k tan θ where θ is the angle of deflection. Show that the relative error in c due to a
given error in θ is minimum when θ = 45 0.

Solution : Consider c = k tan θ . K is taken as a constant.


⇒ log c = log k + log (tan θ )
⇒ δ (log c) = δ (log k ) + δ log (tan θ )

1 sec 2 θ
i.e., δc = 0 + δθ
c tan θ

.in
δc cos θ 1 δθ δθ
i.e., = ⋅ δθ or sy =
c sin θ cos 2 θ c sin θ cos θ
ea
δc 2
i.e., = δθ
c sin 2θ
y

The relative error in c being δc / c minimum when the denominator of the R.H.S. is maximum
ud

and the maximum value of a sine function is1.


st

∴ sin 2θ = 1 ⇒ 2θ = 90 0 or θ = 45 0 ⋅

Thus the relative error in c is minimum when θ = 450

1
31. If T = mv 2 is the kinetic energy, find approximately the change in T as m changes
2
from 49 to 49.5 and v changes from 1600 to 1590. 6 Marks
1
Solution : We have by data T = mv 2 and
2
m = 49, m + δm = 49.5 ∴ δm = 0.5
v = 1600, v + δv = 1590 ∴ δv = −10

Compiled by www.studyeasy.in
Compiled by www.studyeasy.in

2011
We have to find δT . (logarithm is not required)
1
∴ δT = δ ( mv 2 )
2
1
2
{
m( 2vδv) + δm.v 2 }
i.e, =
1
2
{
( 49) ( 2) (1600 ) ( −10) + (0.5) (1600 ) 2 = 1,44,000 }
Thus the change in T = δT = −1,44,000

32. The pressure p and the volume v of a gas are concentrated by the relation
pv 1.4 = cons tan t. Find the percentage increase in pressure corresponding to a

.in
diminution of ½% in volume.
Solution :
pv1.4 = Constant = c( say ), by data.
⇒ log p + 1.4 log v = log c
sy
ea
⇒ δ (log p) + 1.4δ (log v) = δ (log c)
δp ⎛ δv ⎞ δv 1
i.e, + 1.4⎜ ⎟ = 0; But × 100 = − , by data.
⎝ v⎠
y

p v 2
ud

δp ⎛ δv ⎞ δp
∴ × 100 + 1.4⎜ × 100 ⎟ = 0 or × 100 = +0.7 .
p ⎝ v ⎠ p
Thus the percentage increase in pressure = 0.7
st

33. Find the percentage error in the area of an ellipse when an error of +1% is made in
measuring the major and minor axis.

Solution : For the ellipse x 2 / a 2 + y 2 / b 2 = 1 the area (A) is given by π ab where 2a and 2b are
the lengths of the major and minor axis.
Let 2a = x and 2b = y.
δx δy
By data × 100 = 1, × 100 = 1.
x y

Compiled by www.studyeasy.in
Compiled by www.studyeasy.in

x y π
A = πab = π ⋅ ⋅ = xy
2 2 4
∴ log A = log (π / 4) + log x + log y
⇒ δ (log A) = δ log (π / 4) + δ (log x) + δ (log y )
δA δx δy δA δx δy
i.e., =0+ + or × 100 = × 100 + × 100
A x y A x y
δA
∴ × 100 = 1 + 1 = 2
A
Thus error in the area = 2%

27. If the sides and angles of a triangle ABC vary in such way that the circum radius

.in
remains constant, prove that
δa δb δc
+ + =0
cos A cos B cos c
sy
Solution : If the triangle ABC is inscribed in a circle of radius r and if a,b,c respectively
denotes the sides opposite to the angles A,B,C we have the sine rule (formula) given by
ea
a b c
= = = 2r
sin A sin B sin C
y

or a = 2r sin A, b = 2r sin B, c = 2r sin C


ud

⇒ δa = 2rδ (sin A), δb = 2rδ (sin B), δc = 2rδ (sin C )


i.e., δa = 2r cos AδA, δb = 2r cos BδB, δc = 2r cos CδC
st

δa δb δc
or = 2rδA, = 2 rδ B , = 2 rδC
cos A cos B cos C
Adding all these results we get,
δa δb δc
+ + = 2r (δA + δB + δC ) = 2rδ ( A + B + C )
cos A cos B cos C

But A+B+C = 180 = π radians = constant.


⇒ δ ( A + B + C ) = δ (constant) = 0
δa δb δc
Thus + + =0
cos A cos B cos C

Compiled by www.studyeasy.in
Compiled by www.studyeasy.in

2011

Multiple choice Questions:


DIFFERENTIAL CALCULUS

1) The radius of curvature at any point of catenary S=C tan Φ is


a) c sec2 Φ b) c cos2 Φ
c) c tan Φ
-2 d)none

2) Stationary points of f(x,y) = sin x+ sin y+ sin ( x + y ) is


a) (п/3, п/3) b) (п/6, п/6)
c) (п/4, п/4) d) none

3) If the curvature is zero, that point is known as--------------------


a) Point of inflection b) Stationary point
c) (a) or (b) d) none

.in
4) The radius of curvature of the curve y = 4 sin x- sin 2x at x=п/2 is
a) 5√5/4 b) -5√5/4
c)5√5/2 d) none sy
5) The function x2+2xy+ 2y2+2x+2y has a minimum value at
ea
a) (-3/2,1/2) b)(3/2, 1/2 ) c) (3/2,-1/2) d)none

6) The stationary point of f(x,y,z)=x2+y2+z2 where x,y,z are connected by x+y+z=a is


y

a) (a,a) b) (-a,-a) c) (2a, 0) d) none


ud

7) The radius of curvature of the curve √x+√y =1 at the point (1/4,1/4) is ------------
a) ρ =1/√2 b) ρ=√2 c) ρ= -1/√2 d) none )
st

8) The expression for the derivative of arc length in Cartesian form is given by
a) ds/d = r 2 + ( dr/dθ ) 2 b) ds/dx= 1 + (dy/dx) 2
c) ds/dr= 1 + ( d?/dr) 2 d) ds/dt= 1+ (dy/dx)2

9)The formulae for radius of curvature in Cartesian form is

a) ρ = (x1) 2 + (y1) 2)3/2 /x1y11+x11y1

b) ρ =
(1+ y ) 2
1
3/ 2

y2

c) ρ = [(x1) 2 + (y1) 2)3/2 ]

Compiled by www.studyeasy.in
Compiled by www.studyeasy.in

2011
d) ρ = (1+(y1) 2 ) 3/2

10) The function for which Rolle’s theorem is true is:


a) f(x)=log x in the interval [1/2,2]
b) f(x)=|x+1| in the interval [-2,2]
c) f(x)=| x |in the interval [-1,1]
d) Non of the above

11) The value of ‘c’in Rolle’s theorem ,where − π / 2 < c < π / 2 and f(x)= cosx is equal to:
aπ /4 b) π / 3 c) π d)0

12).The expansion of tanx in powers of x by Maclaurin’s theorem is valid in the interval :


a) (− ∞, ∞ ) b) (− 3π / 2,3π / 2) c) (− π , π ) d) (− π / 2, π / 2)

13) The value of ‘c’in Lagrange,s mean value theorem ,where [1,2]
and f(x)= x(x-1) is :

.in
a)5/4 b)3/2 c)7/4 d) 11/6

14) The value of ‘c’in Rolle’s theorem, where [0, π ]


and f(x)= sinx is equal to:
a) π / 6 b) π / 3 c) π / 2
sy
d) Non of these
ea
15) The maximum value of logx/x is:
a) 1 b)e c)2/e d) 1/e
y

16) The maximum value of (l/x ) x is equal to:


a)e b)1 c) e 1/e d) (1/e)e
ud

17) The difference bwtween the maximum and minimum values of the function
a sinx+bcosx is:
st

a) 2 a 2 + b 2 b) 2(a 2 + b 2 ) c) a 2 + b 2 d) − a 2 + b 2

18) Which one of the following statement is correct for the function f(x)=x3
a) f(x) has a maximum value at x=0
b) f(x) has a manimum value at x=0
c) f(x) has a neither a maximum nor a manimum value at x=0
d) f(x) has no point of inflexion

19) which one of the following is not an indeterminate form


a) ∞ + ∞ b) ∞ − ∞ c) ∞ / ∞ d) 0 X∞

20) In Lagrange,s mean value theorem , f1(c)=


a) f(a) – f(b)/ b -a b) f(b) – f(a) / a -b
c) f(b) – f(a) / b -a d) none

Compiled by www.studyeasy.in
Compiled by www.studyeasy.in

2011

21) In Cauchy’s mean value theorem , f1(c) / g1(c) =


a) f(a) – f(b)/ g(b) – g(a) b) f(b) – f(a) / g(b) – g(a)
.c) f(b) +f(a) / g(b) +g(a) d) none
x x
a - b
22) lim is equal to :
x →0 x

a) 0 b) ∞ c) log (a/b) d) log (a-b)

log x
23) The value of lim is equal to:
x→0 x −1

a) -1 b) ∞ c) 1 d) 0

24) The value of lim (1 – cos x / 3 x2 )is equal to:

.in
x →0

a) 3 b) 1/3 c) 1/6 d) 1/9

25) lim ( ex + e-x – 20) / x2 is equal to :


x→0
sy
ea
a) 1 b) -1 c) 1/2 d) -1/2

26) The value of lim ( 1 + x )1/x is :


x→0
y

a) 1 b) -1 c) 1/e d) e
ud

sin x
27) The value of lim =….
x →0 x
st

a) 1 b) 2 c) 3 d)0
28) The formulae for radius of curvature in polar form is

a) ρ = rdr/dp b) ρ = [ (r 2 + (r1) 2)3/2] / [r2+2 ( r1) 2- r r2]

c) ρ =(r 2 + (r1) 2)3/2 d) None

29) The value of lim 1 1/x =……


x →0

a) 1 b) ∞ c) -∞ d) 0

Compiled by www.studyeasy.in
Compiled by www.studyeasy.in

2011

KEY ANSWERS:

1-a 2-a 3-a 4-a 5-a 6-a 7-a 8-b 9-b 10-a

11-d 12-d 13-b 14-c 15-d 16-c 17-a 18-c 19-a 20-c

21-b 22-c 23-c 24-c 25-a 26-c 27-a 28-b 29-b --

.in
sy
y ea
ud
st

Compiled by www.studyeasy.in
Compiled by www.studyeasy.in
INTEGRAL CALCULUS
DIFFERENTIATION UNDER THE INTEGRAL SIGN:

Consider an integral involving one parameter  and denote it as



   , 


where a and b may be constants or functions of . To find the derivative of


 when it exists it is not possible to first evaluate this integral and then to
find the derivative, such problems are solved by using the following rules.

(A) Leibnitz’s Rule for Constant limits of Integration:

Let ,  
,  be continuous functions of x and  then



 , 
   , 
,
 
 
where a, b are constants and independent of parameter 

.in
(B)
If in the integral     , 

sy
Leibnitz’s Rule for Variable Limits of Integration:


,  satisfies the same conditions, and are functions of the


ea

, 
    , 
  ,   , 
parameter, then

     
    
y
ud

***********************************************************

Example 1: Evaluate 
 and hence show that 
 
**
       
st

 
!
"
by using differentiation under integral sign.
Solution: Let #  

    

Differentiate w.r.t  by Leibnitz’s Rule under integral sign.


' ( )  sin 
$ % #&  


# '# 

  ( )  sin 


  -( )   5 then
).  )/01
.2 34
#  6)4 #  7 … … … … … . . 1
;< =
6>( ?<@66 7 , AB6 #  ∞ = =

Compiled by www.studyeasy.in
D D
Compiled by www.studyeasy.in

$ ∞  6 ∞  7   2  7 $72


$ #   6)4 # AB6 #  0 G( H(6 
 
!    !
"  "

*****************************************************


 , # L 0
Example 2: using differentiation under the integral sign, evaluate

4   )4
IJK 

Solution: Let #  


 … … … … . . 1
4   )4
IJK 

6>( % #&  
  
   
 
 4    )4 4   IJK  4
.  IJK  IJK 
4
43.
Integrating both sides w.r.t #
$ #  log1  #  Y … … … … … . . 2
From 1 when #  0 0  0
From 2 when I0log 1C $ Y  0
The solution is _`  abcd  `

.in
sy
*****************************************************

Example 3: Evaluate 
 using the method of differentiation
 j kl 
ea
43 2 



under integral sign.
Solution: Let I = 
 j k l 
y

43 2 
Differentiate w.r.t a by Leibnitz’s rule under integral sign
ud



' 1
$ % &   6)4 


 ' 1   " 
st


 1
 .

1  "  "  1   " 

Let  43 2  43 2  by partial fractions


4 s3t u3v
43 2  2 43 2   2

Solving, z  0, {  , Y  0, | 
2 )4
2 )4 2 )4


1 " 1 D
$ % &  " }  ~
 

  1 1     1   
" " " 2  1

Integrating w.r.t  $     €


  log  1  C
! 4 !
" 34 "
Then C 0 by putting 0

Compiled by www.studyeasy.in
Compiled by www.studyeasy.in

Example 4: Evaluate  ‚<H1  # ?<@


 using the method of
!
*****************************************************

differentiation under integral sign

****************************************************************
Reduction fformulae
ormulae:
ormulae:
I  Sin θ d θ
II  cos  θ d †
III  sin‡ θcos  θ †

And to evaluate

I
‰
 2 sin θ d †

.in
II
‰
 cos  θ d †
2

III  sin‡ θcos  θ d †


Š/"
sy
With † = x
ea
∫ sin x dx = ∫ sin n −1 x. sin x dx
n
(a )

= sin n −1 x.(− cos x) − ∫ (n − 1) sin n− 2 x. cos x(− cos x) dx


y
ud

= − sin n −1 x. cos x + ( n − 1) ∫ sin n − 2 x.(1 − sin 2 x)dx

= − sin n−1 x. cos x + ( n − 1) ∫ sin n − 2 xdx − (n − 1) ∫ sin n x dx


st

n ∫ sin n x dx = − sin n −1 x. cos x + (n − 1) ∫ sin n− 2 x dx

Or sin n −1 x. cos x n − 1
∫ sin x dx = − ∫ sin n − 2 x dx ........... (1)
n
+
n n

Similarly, (b) sin x. cos n−1 x n − 1


∫ cos x dx = − n ∫
cos n − 2 x dx ........... (2)
n
+
n
Thus (1) and (2) are the required reduction formulae

Compiled by www.studyeasy.in
Compiled by www.studyeasy.in

.in
TO Evaluate
sy
ea
Then etc
y
ud
st

i) =

ii) ( put x = a sinθ, so that dx = a cosθ dθ


θ
Also when x = 0, θ = 0, when x = a, θ = π/2)

= =

Compiled by www.studyeasy.in
Evaluate ‘   
Compiled by www.studyeasy.in
%Ž 3  &
iii) ( put x = a tan , so that dx = a sec d
Also when x = 0, θ = 0, when x = ∞, θ =
π/2)

–/ Ž ’“” •
Ž ’“” •
=
‰
 cos ")" θ dθ
d 2
Žd
=
. )Ž)™………™. . .
d )—)˜………—.d –
Žd 
=

Evaluate  sinš x cos " x dx


 sin" x sinx cosx" dx
iv)
=
4)›J"œ "œ "
  € dx
" "
=
sin" 2x  sin" 2x cos2x dx
4

=

- dx   sin" 2x  cos2x. 2dx5


=
4 4)›Jšœ 4

.in
 " "

- dx   cos 4x dx  dx5
4 Ÿ "œ
4ž  
=
sy
-¢  £¤¥™¢  £¤¥— ¢5
d d d
ea
d¡ ™ —
=
Evaluate  cos š θ sin  6θ d †
Š/ž
v)
 cos š θ 2sin3θcos3θ  d †
Š/ž
y

§ sin  θ cos ¨ 3θ d †
Š/ž
ud

 sin  x cos ¨ x d x
§ Š/"
—
st

. 
§ ©¡.™. d
— d‘.§.¡.™. d˜
=

(Put 3• = x ; so that 3d• = dx. Also when • = 0, x = 0

When • = –/6, x = –/2

Evaluate   š 1   "

4  /"
vi)
 @=š 6?<@ "  /" ?<@ 6
6 =
!/"
=

 @=š 6 ?<@ š 6
6
!/"
=

Compiled by www.studyeasy.in
—
Compiled by www.studyeasy.in
§ ¡. ™.   ˜¡
=

(put x = sint so that dx = cost dt, when x = 0, t = 0 when x = 1, t = –/2 )

************************************************************************
Tracing of Curves:
For the evaluation of Mathematical quantities such as Area, Length,
Volume and Surface area we need the rough graph of the equation in either
Cartesian or parametric or polar form depending on the statement of the
problem. We use the following theoretical steps to draw the rough graph.

A) Cartesian Curves: y = f (x)

a) Symmetry:
i) If the power of y in the equation is even, the curve is symmetric

.in
about x- axis
ii) If the power of x in the equation is even, the curve is symmetric
about y- axis
iii)
sy
If both the powers x and y are even then the curve is symmetric
about both the axis.
ea
iv) If the interchange of x and y leaves the equation unaltered then the
curve is symmetric about the line y = x
v) Replacing x by – x and y by – y leaves the equation unchanged the
y

curve has a symmetry in opposite quadrants.


ud

b) Curve through the origin:


The curve passes through the origin, if the equation does not contain
constant term.
st

c) Find the origin, is on the curve. If it is, find the tangents at 0, by equating
the lowest degree terms to zero.
i) Find the points of intersections with the coordinate axes and the tangents
at these points.

.
For, put x = 0 find y; and put y = 0,


find x. At these points, find
  ∞, then the tangent is parallel to y axis.


 0, then the tangent is parallel to x axis.


If

d) Asymptotes: express the equation of the curve in the form y = f (x). Equate
the denominator to zero. If the denominator contains x, then there is an
asymptote.

Compiled by www.studyeasy.in
Compiled
e) Find the region in which thebycurve
www.studyeasy.in
lies.
f) Find the interval in which the curve is increasing or decreasing.

B) Parametric Form: x=f(t), y=g(t)

In this case we try to convert the parametric form into Cartesian form by
eliminating the parameter (if possible). Otherwise we observe the following
I) Find dy/dt and dx/dt and hence dy/dx.
II) Assign a few values for t and find the corresponding value for x, y ,y’.
III) Mark the corresponding points, observing the slope at these points.

C) Polar curves: r = f(•)

a) Symmetry: 1. If the substitution of - • for • in the equation , leaves the

.in
equation unaltered, the curve symmetrical about the initial line.

2. If the power of r are even, the curve is symmetrical


about the pole.
sy
b) Form the table, the value of r, for both positive and negative values of •
and hence note how r varies with • . Find in particular the value of • which gives
ea
r = 0 and r = ∞ .

c) Find tan ¯ . This will indicate the direction of the tangent.


y
ud

• that are contained between certain limits.


d) Sometimes by the nature of the equation it is possible to ascertain the value of r and
st

e) Transform into Cartesian, if necessary and adopt the method given before.

f) Sketch the figure.

PROBLEMS FOR TRACING THE CURVES

1. Astroid :   Ž ”°’— ± , ²  Ž ’³— ± °´ —  ²—  Ž— )


  

It is symmetrical about the x-axis

Limits || ¶ 

and |·| ¶ 

The curve lies entirely within the square bounded by the lines   ¸ , ·  ¸ 

Compiled by www.studyeasy.in
Compiled by www.studyeasy.in
Points: we have

when t = 0 or

, when t 

As t increases x Y Portion traced


+ve and +ve and
From 0 to
decreases from increases from From 0 to ∞ A to B
a to 0 0 to a
-ve and
+ ve and
From to π increases
decreases From ∞to 0 B to C
numerically
from a to 0
from 0 to -a

.in
As t increases from π to 2π,
π,, we get the reflection of the curve ABC in the x - axis.
The values of t > 2π give no new points.
sy
Hence the shape of the curve is as shown in the fig.
y ea

Y
ud
st

-X O X

-Y

Here ox = oy = a

Compiled by www.studyeasy.in
Compiled
It is symmetrical about the y axis. byAswww.studyeasy.in
such we may consider the curve
rve only for
positive values of x or .

Limits: The greatest value of y is 2a and the least value is zero. Therefore the curve
lies entirely between the lines y = 0 and y = 2a.

Points: We have

As increases x Y Portion traced


From 0 to π increases from increases from
From ∞ to 0 0 to A
0 to aπ 0 to 2π
From π to 2π increases from decreases
From 0 to ∞ A to B
a π to 2aπ from 2a to 0

.in
As decreases from 0 to - 2π,, we get the reflection of the arch OAB in the y-
axis. Hence the shape of the curve is as in the fig.
sy
Y
y ea

X
ud

2. Cardioid: r
st

Fig.,

Initial Line

A cardioids is symmetrical about the initial line and lies entirely within the circle r
= 2a. Its name has been derived from the Latin word ‘ Kardia’- meaning heart.
Because it is a heart shaped curve.

*********************************************************************
***

Compiled by www.studyeasy.in
Compiled by www.studyeasy.in

I) Length
II) Area
III) Volume
IV) Surface area

Table to find the values: Area, Length ,VolumeThe surface area


Quantity Area (A) Length (S) By revolving about the axis of
rotation to form solid
Coordinate Volume (V) Surface area (SA)


@
system

 
  ¹1  ·4"
  D· "
  2D·



Cartesian

form y = f (x)

 º1  ·4"
1
Where







@
 
6  º 4 "  · 4 "
6

.in
 D· "
6  2D·
6

6
6
6
Parametric

form x= x(t)
y= y(t) Where
sy 1
 º 4 "  · 4 "
j
1



@
r = f († )  » "
†  ¹» "  »4"
†  D·
†  2D·
†
ea
"
2
†
†
Polar form

Where 
1

º» "  »4"
y
ud

3. Find the entire length of the cardioid »   1  ?<@ †, Also show that the upper
st

half is bisected by †  The cardioid is symmetrical about the initial line and for its
!
 
upper half, increases from 0 to π

Also z‚@<,  -a sin θ.



¾ "
$ Length of the curve  2  ¹À» "  - 5 Á d†
!

= 2  ºÂ 1  ?<@ †"   @=†" Ã


†
!

= 2  º21  ?<@†
†
!

Compiled by www.studyeasy.in
! by www.studyeasy.in
Compiled
= 4a
Å/" Š
= 4a Ä Ä
4/"

= 8a (sin π/2 - sin 0) = 8a

$ Length of upper half of the curve of the cruve is 4a. Also length of the arch AP
from 0 to π/3
Æ Æ
= 2  Ÿ º21  ?<@†
† = 2  Ÿ cos .
†
¼
"
= 4|sin †/2|
!/ 

= 4 -  05
4
"
= 2a ( half the length of upper half of the cardioids )

Fig.

.in
sy
—  ²— =Ž—
  
ea
1. Find the entire length of the curve

Solution:
The equation to the curve is  Ÿ  · Ÿ = Ÿ ), the curve is symmetrical about the axis and
2 2 2
y
ud

it meets the ‘x’ – axis at x = a

Fig.
st

If S1 = the length of the curve AB


Then required length is 4S1
"
S = 4S1 =  ¹1   €



Now,
® 4/ 
 Ÿ  · Ÿ = Ÿ $   €
2 2 2 ®
 

S = 4  ¹1   €
"/ 


®


4 ¹

 2/Ÿ 3 ® 2/Ÿ
 2/Ÿ
=

Compiled by www.studyeasy.in
4 ¹

2/Ÿ
Compiled by www.studyeasy.in
 2/Ÿ
=

4  4/   )4/ 
  4 4/  Ç È
 2/Ÿ
"/ 
=
s = 6a units

Fig.

2. Find the perimeter of cardioid r = a (1+cosθ).

Solution:
The equation to the curve is symmetrical about the initial line.
Fig.

$ The required length of the curve is twice the length of the curve OPA

.in
$  @=†
At O, θ = π and at A θ = 0


Now, r = a(1+cosθ)
"
  ¹» "   €
†
! ¾ sy

s=

  ºa" 1  cosθ"   " @=" †


†
!
ea
s=
2  2acos dθ
Š Å
"
s=
• –
™Ž É Ê
’³
y


s= d = 8a units
 ‘
ud

Find the area of the  Ÿ  · Ÿ = Ÿ


2 2 2
3.
Solution: The parametric equation to the curve  Ÿ  · Ÿ = Ÿ is given by :
2 2 2
st

x  ?<@   † , ·   @=  †

Area = 4  ·
 x  ?<@   † , ·   @=  †

Put

$ dx = -3a cos2θ sinθ dθ when x = 0, θ = π/2 ; when x = a, θ = 0

4 @=  †-3a cos2θ sinθ dθ


!/"
A =
12 "  @=š † cos2θ dθ
!/"
=
12 "  . € €
  4 4 !
š3" "3" " "
=

Sq. units
 ! 2

=

Compiled by www.studyeasy.in
Compiled by www.studyeasy.in

4. Find the area of the cardioid r = a (1+cosθ).

$ Total area = 2 © area above the line θ = 0


Solution: The curve is symmetrical about the initial line.

= 2 »
† is the formula
!4 "
"
A for area in polar curves
2  a 1  cosθ"
†
!4 "
"
A =
"
=   2?<@ €
†
! " "¼
"
A
= 4 "  ?<@ š
†
! ¼
"
Put θ/2 = t $
†  2
6
A

4 "  ?<@ š 6 2
6
!/"
=
§Ž © ©
— d –
™  
=

Sq. units
 ! 2
"
=

.in
x  6  @=6, ·   1  ?<@6 , 0 ¶ 6 ¶ 2D and its base.
5. Find the area bounded by an arch of the cycloid

Solution: x  6  @=6, ·   1  ?<@6  for this arch ‘t’ varies from 0 to 2π.
sy
$ z»(  jË ·

"!

= jË  1  ?<@6   1  ?<@6 


6 since dx =  1  ?<@6 
6
"!
ea

=   jË 1  ?<@6 "


6
"!
y

=4 " jË @= š


6
"! j
ud

"

= 8 " jË @=š 6


6
!

= 16 " jË @=š 6


6
!/"
st

= d¡Ž . . .
— d –
™  
= 3D " ÌÍ. B=6@

6. Find the volume generated by revolving the cardiod r = a (1+cosθ) about the
initial line.

Solution: For the curve, varies from 0 to

____________________________________________________________________

Find the volume of the solid obtained by revolving the Astroid x2/3 + y2/3 = a2/3

Compiled by www.studyeasy.in
Compiled by www.studyeasy.in

Volume is obtained by revolving the curve from x = 0 to x = a about x-axis and taking
two times the result.

Î  2  D· "


 2  D@=   †" 3 ?<@2†@=†


†
!
"

 ”Ïг” ϐ³±’
—–Ž—
d‘˜

_____________________________________________________________________
___

Problems for practice:

2. Find the volume of the solid obtained by revolving the cissoid · " 2    
1. Find the surface area of r = a (1 - cosθ)

   about its asymptote.

.in
3. Find the length between [0, 2D ] of the curve   †  sin †, ·  1 
cos †. sy
____________________________________________________________________
ea

Find the surface area of solid generated by revolving the astroid  Ÿ  · Ÿ = Ÿ


2 2 2
y

about the axis.


ud

Solution: The required surface area is equal to twice the surface area generated by
revolving the part of the astroid in the first quadrant about the axis.

Taking x  ?<@   6 , ·   @=  6 we have,


st

Surface area = 2  2D·


  4  ·
6
!/" !/" 1
j

· ¹ €   €
6
® "
=4 
!/" 
j j

= 4π  asin  t Ñ 3acos " t sint"  3acos " t sint" Ò1/2dt


Š/"

= 12a "  sinš t cos t dt


Š/"
Put z = sint
D " ÌÍ. B=6@
4"
Ó
=

7. Find the surface area of the solid generated when the cardioid r = a (1+cosθ)
revolves about the initial line.

Compiled by www.studyeasy.in
Solution: The equation Compiled by www.studyeasy.in
to the curve is r = a (1+cosθ). For the upper part of the
curve, θ varies from 0 to π

Put x = r cosθ, y = rsinθ

$ Surface area =  2D ·
@
!

= 2D  » @=† 
†
! 1

"
= 2D  » @=†¹» "   €
†
! ¾

= 2D  1  ?<@†. @=† º " 1  cos †"   " @=" †


†
!

Ôa2
= 16Ô

.in
UNIT IV: VECTOR CALCULUS
sy
ea
Scalar and Vector point functions:
y

(I) If to each point p(R) of a region E in space there corresponds a definite


ud

scalar denoted by f(R), then f (R) is called a ‘scalar point function’ in E. The
region E so defined is called a scalar field.
st

Ex: a) The temperature at any instant


b)The density of a body and potential due to gravitational matter.
(II) If to each point p(R) of a region E in space there corresponds a definite
vector denoted by F(R), then it is called the vector point function in E. the
region E so defined is called a vector field.
Ex: a) The velocity of a moving fluid at any instant
b) The gravitational intensity of force.
Note: Differentiation of vector point functions follows the same rules as those
of ordinary calculus.
If F (x,y,z) be a vector point function then

Compiled by www.studyeasy.in

Ø 'Ø ' 'Ø '· 'Ø 'Ù
  
Compiled by www.studyeasy.in

6 ' '6 '· '6 'Ù '6

Ø 'Ø
 'Ø
· 'Ø
Ù
  

6 '
6 '·
6 'Ù
6
'Ø 'Ø 'Ø

Ø 
 
· 
Ù
' '· 'Ù
' ' '

Ø  Ú
 
· 
ÙÛ Ø
' '· 'Ù

( 1)

Vector operator del ( ∇ )

.in
The operator is of the form

∇  = Ý 7
   sy
 ® Þ

GRADIENT, DIVERGENCE, CURL (G D C)


ea
Gradient of the scalar point function:
y

It is the vector point function f defined as the gradient of the scalar point
ud

function f and is written as grad f, then

grad f = ∇f = = Ý 7 €
  
 ® Þ

' ' '


st

= Ý 7
' '· 'Ù

DIVERGENCE OF A VECTOR POINT FUNCTION

The divergence of a continuously differentiable vector point function F(div F)is


defined by the equation
'Ø 'Ø 'Ø

=ß Ø  ∇. Ø  = Ý 7
' '· 'Ù

 Ø   =  à Ý  ¯ 7 6>(

Compiled by www.studyeasy.in
' ' '
Compiled by www.studyeasy.in


=ß Ø  ∇. Ø  Ú= Ý  7 Û ·  =  à Ý  ¯ 7
' '· 'Ù
' 'à '¯
∇·Ø   
' '· 'Ù

CURL OF A VECTOR POINT FUNCTION

The curl of a continuously differentiable vector point function F is defined by


the equation
'Ø 'Ø 'Ø
curl F  ∇ © Ø  = © Ý© 7©
' '· 'Ù
' ' '
curl F  ∇ © Ø  Ú= Ý  7 Û © =  àÝ  ¯7
' '· 'Ù

.in
= Ý 7
' ' '
curl F  ∇ © Ø  â â
sy ' '· 'Ù
 à ¯
ea
'¯ 'à '¯ ' '¯ '
∇©Ø  =Ú  ÛÝÚ  Û7Ú  Û
'· 'Ù ' 'Ù ' 'Ù
y

∇ 
∇ © 
ud

∇·Ø
st

DEL APPLIED TWICE TO POINT FUNCTIONS

Le being vector point functions, we can form their divergence and curl,
whereas being a scalar point function, we can have its gradient only. Then we
have Five formulas:

'" '" '" 


div grad f  ∇   "  "  "
"
' '· 'Ù

?B»‚ H»
  ∇ © ∇  0

3.
=ß ?B»‚ Ø  ∇ · ∇ © Ø  0
=ß ?B»‚ Ø  ∇ · ∇ © Ø  0

?B»‚ ?B»‚ Ø  H»



=ß Ø  ∇" F

Compiled by www.studyeasy.in
Compiled by www.studyeasy.in

H»

=ß Ø  ?B»‚ ?B»‚ Ø  ∇" F
∇∇ · Ø  ∇ © ∇ © Ø  ∇" F

PROOF:

㠏 ä  ã · ãä  ã · ³ æ  ç €ä
å å å
å å² åè
(I) To prove that

' ' '


 ∇ · Ú= Ý 7 Û
' '· 'Ù
' ' ' ' ' '
 curl grad f  ∇ © ∇f  Ú= Ý  7 Û © Ú= Ý 7 Û
' '· 'Ù ' '· 'Ù

'" '" '" 

.in
 " " "
' '· 'Ù

'" '" '"


sy
 é "  "  "ê 
' '· 'Ù

 ∇" f
ea

'" '" '"


G>(»( ∇"    =@ ?‚‚(
ëA‚?= <A»6<»
' " '· " 'Ù "
y
ud


∇" f  0 =@ ?‚‚(
ëA‚?( ′ @ (ÍB6=<

(II) To prove that ”Ï´ì í´ŽŒ î  ‘


st

' ' ' ' ' '


curl grad f  Ü © Üf  Ú= Ý  7 Û © Ú= Ý 7 Û
' '· 'Ù ' '· 'Ù

= Ý 7
' ' '
â â
 ' '· 'Ù
â' ' 'â
' '· 'Ù

Compiled by www.studyeasy.in
'  ' Compiled
 ' www.studyeasy.in
 '  '  ' 
 ié  ê jé  ê  ké  ê
by
'· 'Ù '· 'Ù ' 'Ù ' 'Ù ' '· ' '·

Ü © Üf  0

(III) To prove thatã · ã © ñ  ∑ ³ € · ³ © å  æ © å²  ç © åè €


å åñ åñ åñ
å

 ∑ = · = © Ý© 7© €
2 ó 2 ó 2 ó
 2 ®  Þ 

 ∑ = © = · =©Ý· =©7· €
2 ó 2 ó 2 ó
 2  ®  Þ

 ∑ 7 · Ý· €
2 ó 2 ó
 ®  Þ

0

(IV) To prove that ã © ã © ñ  ∑ ³ € © ³ © å  æ © å²  ç © åè €


å åñ åñ åñ

.in
å

'"Ø '"Ø '"Ø


 ô = © é= ©  Ý ©  7 © ê
' "
sy ' '· ' 'Ù

'"Ø '"Ø '" Ø '" Ø


 ô õé= · " ê =  = · = " ö  õé= · ê Ý  = · Ý ö
ea
' ' ' '· ' '·
'"Ø '"Ø
 õé= · ê 7  = · 7 ö
' 'Ù ' 'Ù
y
ud

'"Ø '"Ø '"Ø '"Ø


 ô= · " =  = · Ý =· 7ô "
' ' '· ' 'Ù '
st

' 'Ø 'Ø 'Ø '"Ø


 ô = Ú= · Ý· 7· Ûô "
' ' '· 'Ù '

 ÜÜ · F  Ü" F

curl curl F  ÜÜ · F  Ü" F

(V) To prove that


We have by (IV) which implies ÜÜ · F  ?B»‚ ?B»‚ Ø  Ü" F
ÜÜ · F  Ü © Ü © Ø  Ü" F

Compiled by www.studyeasy.in
Compiled by www.studyeasy.in

=3Ý27

.
ø´°Ðì“ù: Ì><G 6>6 ã " r   nn  1r )"
úbaûü¤b¥: Ü" r    Ü · Ür  
R
 Ü · Ún r )4 Û  nÜ · r )" R  nÂÜr )" . R  r )" Ü · RÃ
r
R
 n Àn  2r )  · R  r )" 3Á
r
 nÂn  2r )š r "   3r )" Ã
 nn  1r )"

.in
Otherwise: Ü" r     
ý2 þ  ý2 þ  ý2 þ 
ýœ2 ý2 ý2

Now  nr )4  nr )4  nr )" x


ýþ  ýþ œ sy
ýœ ýœ þ

∂^2 r^n /∂x^2   nÂr^n  2  n  2 r^n  3 ∂r/


∂x xà  nÂr^n  2  n  2 r^n  3 x/r xÃ
ea

 nÂr )"  n  2r )š x " à … … … … … . 1


y

SImilarly,  nÂr )"  n  2r )š y " à … … … … . . 2


ý2 þ 
ý2
ud

 nÂr )"  n  2r )š z " à … … … … … 3


ý2 þ 
ý2

Adding equations 1, 2and 3, gives


st

Ü" r    nÂ3r )"  n  2r )š x "  y "  z " Ã


 nÂ3r )"  n  2r )š r " à  nn  1r )"

In particular Ü"  €  0
4
þ

Compiled by www.studyeasy.in
Compiled by www.studyeasy.in

components of velocity and acceleration at t=2 in the direction of =  3 Ý  2 7


Solution: ;>( A<@=6=< ß(?6<» < 6>( A»6=?‚( 6 6=( 6 =@,

.in
sy
y ea
ud
st

Compiled by www.studyeasy.in
Compiled by www.studyeasy.in

  6    1=  6 " Ý  6  5 7

;>( ß(‚<?=6· Î
  =@ H=ß( ·



j

Î
  
 36 " =  2 6 Ý  7

j

;>( ??(‚(»6=<    =@ H=ß( ·



 2


j j 2

  66=2Ý
2


j 2

;>( 6 6=( 6  2, Î
 
 »( H=ß( ·

Î
  
 12 =  4 Ý  7

j

   12 =  2 Ý
2


j 2

Î(‚<?=6· ß(?6<» = 6>(


=»(?6=< < =  3Ý  27 =@

.in
 ·  3  3" €  12 =  4 Ý  7 ·  3  3" € 
 Î

√4š √4š √4š

z??(‚(»6=< = 6>(
=»(?6=< < =  3Ý  27 =@
sy
  ·  €  12 =  2 Ý ·  €
3  3"  3  3"  4
√4š √4š √4š
ea

ø´°Ðì“ù: Ø=
6>( B=6 ß(?6<» <»‚ 6< 6>( @B»?( ·   Ù " 
y

4 6 6>( A<=6 1, 1,2


ud

°ìϱ³°: Î(?6<» <»‚ 6< 6>( H=ß( @B»?( =@ Ü·   Ù " 

Ü·   Ù "   = Ý 7 € ·   Ù " 


  
st

 ® Þ

 ·   Ù " =  3· " Ù " Ý  2·   Ù 7


z6 1, 1,2
 4 =  12 Ý  4 7
(?( 6>(
(@=»(
B=6 <»‚ 6< 6>( @B»?( =@

  4=  12Ý  47


š )4" 3š 4
√4ž34šš34ž √4¨ž

Compiled by www.studyeasy.in
Compiled by www.studyeasy.in

·Ù   6 6>( A<=6 2, 1,1 = 6>(


=»(?6=< < =  2Ý  27
°ìϱ³°: Î(?6<» <»‚ 6< 6>( @B»?( , ·, Ù =@ Ü

i. e. Ü  = Ý 7 € · "  ·Ù   
  
 ® Þ

Ü  · " =  2·  Ù    Ý  3·Ù "  7


z6 2, 1,1
Ü  =  3 Ý  3 7
|=»(?6=<‚
(»=ß6=ß( = 6>(
=»(?6=< < =  2Ý  27 =@

 Ü",)4,4 ·  =  3Ý  37 ·
3" 3" 3" 3"
√43š3š √

 
4)ž)ž )44
   

Ü" r    nn  1r )4


Ü" r    ÜÜr  

.in

ø´°Ðì“ù: Ø=
6>( ?<@66@ , , ? @B?> 6>6 6>( ß(?6<»
sy
Ø    ·  Ù̂    ? ·  2Ù7    2·  Ù̂ =@ =»»<66=<‚
ea
°ìϱ³°:
=ß( Ø    ·  Ù̂    ? ·  2Ù7    2·  Ù̂
y

=@ =»»<66=<‚
ud

Ì=?( 6>( ß(?6<» =(‚


=@ =»»<66=<‚
;>(»(<»( Ü © Ø  0
st

̂ ̂ 7
YB»‚ Ø  â â
  
 ® Þ
  ·  Ù   2·  Ù   ?·  2Ù
 ?  1̂  1  ̂    17
=. (. , ?  1̂  1  ̂    17
0
;>=@ =@ A<@@=‚( <‚· G>(, ?  1  0, 1    0,   1  0
G>=?> =A‚=(@   1,   1, ?  1

Orthogonal curvilinear co-ordinates

Compiled by www.studyeasy.in
Compiled by www.studyeasy.in
Let the rectangular co-ordinates (x,y,z) of Any point be expressed as
function of (u,v,w),
So that x = x(u,v,w),y = y(u,v,w),z = z(u,v,w) …..(1)
Suppose that (1) can be solved for u,v,w in terms of x,y,z
i,e u = (x,y,z), v = v(x,y,z),w = w(x,y,z) ….(2)
We assume that the functions in (1) and (2) are single valued functions
and have continuous partial derivatives so that the correspondence between
curvilinear co-ordinates
(x,y,z) and (u,v,w) is unique. Then (u,v,w) are called curvilinear
of (x,y,z). Each of u,v,w has a level of surface through an arbitrary point .

The surface are called co-ordinate

surface through

.in
Each pair of these co-ordinate surface intersects In curves called the co-
sy
ordinate curves. The curve of intersection of will be called the w-curve, for
only w changes along this curve. Similarly we define u and v-curves.
ea
In vector notation, (1) can be written as
R = x(u,v,w)I + y(u,v,w)J + z(u,v,w)K
y
ud
st

Compiled by www.studyeasy.in
Compiled by www.studyeasy.in

.in
sy
y ea
ud
st

Compiled by www.studyeasy.in
Compiled by www.studyeasy.in

.in
sy
y ea
ud
st

Compiled by www.studyeasy.in
Compiled by www.studyeasy.in

.in
sy
y ea
ud
st

Compiled by www.studyeasy.in
Compiled by www.studyeasy.in

.in
sy
y ea
ud
st

The co-ordinate curves for ρ are rays perpendicular to the Z-axis; for ф
horizontal circles with centers on the Z-axis; for z lines parallels to the Z-axis.
x = ρ cos ф,

Compiled by www.studyeasy.in
Compiledyby
= ρwww.studyeasy.in
sin ф, z=z

So that scale factors are h1=1, h2 = ρ, h3= 1. Also the volume element
dV=ρ dρ dф dz.
2) Spherical polar co-ordinates:
Let p(x,y,z) be any point whose projection on the xy-plane is Q(x,y) .
Then the Spherical polar co-ordinates of p are such that r = op,

The level surfaces are respectively spheres


about O, cones about the Z-axis with vertex at O and planes through the Z-axis.

.in
The co-ordinate curves for r are rays from the origin; for θ, vertical circles with
centre at O (called meridians); for ф, horizontal circles with centres on the Z-
sy
axis.
ea
x = OQ cosф
= OP cos(90-θ)cosф
= r sinθ cosф,
y
ud

y = OQ sinф = r sinθ sinф


z = r cosθ
st

So that the scale factors are

Also the volume element

Compiled by www.studyeasy.in
Compiled by www.studyeasy.in

UNIT - 6
DIFFERENTIAL EQUATIONS

#######################################################################
Text Book:
 Higher Engineering Mathematics B S Grewal
Reference:

.in
 Higher Engineering Mathematics John Bird
 Advanced Engineering Mathematics. Zill and Cullion
 Internet

Unit –VI:
Goal:
sy
ea
 One of our goal in this course is to solve or find solutions of differential equations.
 Sketch the family of curves given by the Differential equation
 A particular curve of a family may be determined when a point on the curve is specified
y

Notation:
ud

dy d 2 y d 3 y
. .
Leibnitz notation dx dx 2 dx 3
y′, y′′, y′′′........ y n
st

Prime notation:

Newton’s dot notation: d 2s ..


= −32 becomes s = −32
dt 2
Learning outcomes
Upon successful completion of ordinary differential equation, it is expected that a student will be able to
do the following.
 Identify an ordinary differential equation & its order
 Verify whether a given function is a solution of a given ordinary differential equation
 Classify ordinary differential equations as variable separable, homogenous, linear, exact and
reducible to these forms.
 Find solutions of separable differential equations
 To be able to derive an ordinary differential equations as the mathematical model for a physical
phenomenon.
 Solve first order homogeneous differential equations.

Compiled by www.studyeasy.in
Compiled by www.studyeasy.in

Introduction to Differential Equations:


 In Mathematics, a differential equation is an equation
dy in which the derivative of a function
appear as variables. dx
 Many of the fundamental laws of Physics, Chemistry, Biology and economics can be formulated
as differential equations
 Definition: The derivative of a function y = Φ(x) is itself another function Φ’(x)
found by an appropriate rule.

For example: The function y = e0 . 1x 2 is differentiable on the interval(-∞, ∞),


and its derivative is dy =0.2 x e0.1x2
dx dy = 0 . 2 xy
If we replace e 0 .1x 2 by the symbol y, we obtain
dx

Definition:

.in
A differential equation is an equation which involves differential coefficients or the
differentials.
Definition:
sy
A differential equation is simply an equation involving an unknown function and its
derivatives
ea
1 e x dx + e y dy = 0
d 2x
+ n2 x = 0
y

2 2
dt
ud

dx x
3 y=x +
dy dx
dy
st

5
d 4 y d 2 x  dx 
4. 4
+ 2 +   = et
dt dt  dt 
3/2
d 2 y   dy  
2

5. k 2 = 1+   
dx   dx  
∂2u ∂2u ∂2u
6. 2 + 2 + 2 = 0
∂x ∂y ∂z

Definition ODE:
A differential equation involving derivatives with respect to a single independent variable is
called an ordinary differential equation.
Partial Differential Equations:
A differential equation involving partial derivatives is called a partial differential equation.
Order of a Differential Equation:

Compiled by www.studyeasy.in
Compiled by www.studyeasy.in

The order of a differential equation is the order of the highest order derivative occurring in it.
Degree of a Differential Equation:
The degree of a differential equation is the degree of the highest order derivative occurring in it,
when the derivatives are free from radicals and fractions.

Examples 1, 2, 3, 4 and 6 are of first degree Ex: 5 is of second degree because it can be written as

3


 2 

   2
k d 2 y
   

1 +





dy 




 = 





 







dx 










dx 2 



 

dy
Family of Curves: Integrating both sides of the derivative =3 gives y = 3 x + c,
dx
Where c is an arbitrary constant y = 3 x + c, represents a family of curves, each of the curves in the
family depending on the value of c. Examples include y = 3 x + 8, y = 3 x + 3, y = 3 x and y = 3x - 8

.in
and these are shown in the following figure.

sy
y ea
ud

A particular curve of a family may be determined when a point on the curve is specified. Thus, if
y = 3x + c passes through the point (1, 2) , from which C = -1. The equation of the curve
st

passing through (1,2) is therefore y = 3 x -1

Example: Sketch the family of curves given by the equation dy


= 4 x and determine the
dx
equation of one of these curves which passes through the point (2,3).

Solution: Integrating both sides of d y


= 4 x
with respect to x gives
d x
dy
∫ dx
dx = ∫ 4 x d x i .e y = 2 x 2 + c

Compiled by www.studyeasy.in
Compiled by www.studyeasy.in

To determine the equation of the curve passing through the point ( 2, 3), from which c = -5

Hence equation of the Curve passing through the point (2,3) is y = 2x2-5.

.in
Concept of a Solution of an ODE:
sy
Any function Φ , defined on an interval I and possessing at n derivatives that are continuous on I
ea
which when substituted into an nth order ordinary differential equation reduces the equation to an
identity, is said to be a solution of the equation on the interval
y

How do you Solve a Differential Equation for Unknown Function: y = Φ ( x ) ?.


ud

The problem is loosely equivalent to the familiar reverse problem of differential calculus: Given a
derivative, find an antiderivative
st

2
For example: In our initial discussion we have already seen that y = e0.1x is a solution of
dy
= 0.2 x y on the interval ( − ∞, ∞ )
dx

Solution Curve Or Integral Curve: A solution curve is the graph of a differentiable function

Solution of a Differential Equation:


A solution or Integral of a differential equation is a relation between the independent and the
dependent variables which satisfies the given differential equation.

General Solution or Complete Integral:


A solution containing the number of arbitrary constants equal to the order of the equation is called
the general solution or complete integral.

Compiled by www.studyeasy.in
Compiled by www.studyeasy.in

Particular Solution:
Any solution obtained from the general solution by giving specific values to one or more of the
arbitrary constants is called a particular solution.
For example: y 2 = a ( x + b ) is the general solution of the second order
2
differential equation d 2 y  dy  , as it contains two arbitrary constants. If we put a =
y 2
+  = 0
dx  dx 
1, b = 0 then y2 = x particular solution

The Solution of Equations of The Form :


dy
= f ( x)
dx

.in
A differential equation of the form dy = f ( x ) is solved by direct integration, i.e.
dx
y = ∫ f ( x)dx + c
sy
dy
ea
Example: Determine the general solution of x = 2 − 4 x3
dx
Solution: Rearranging dy gives:
y

3
x = 2 − 4 x
dx
ud

dy 2 − 4 x3 2 4 x3 2
= = − = − 4x2
dx x x x x
st

Integrating both sides gives: 4 x3


y = 2log x − +c
3
Problem: Find the particular solution of the differential equation dy given
5 + 2 x = 3,
dx
the boundary conditions 2 when x = 2.
y = 1
5
Solution: dy then dy 3 − 2 x 3 2 x
5 + 2 x = 3, = = −
dx dx 5 5 5
On integration we get 3 x x 2
y = − + c
5 5

Compiled by www.studyeasy.in
Compiled by www.studyeasy.in

Substituting the boundary conditions


2 to evaluate c, gives c Hence the
y =1 and x = 2
5
particular solution is 3x x2
y = − +1
5 5

Example: The bending moment M of the beam is given by dM where w and l


= − w (l − x ) ,
dx
are constants.

Determine M in terms of x given: 1 2 when x =0.


M = wt
2

Solution: d M = − w l − x = − w l + w x :
( )
dx

.in
Integrating with respect to x gives
wx2 When
M = − w lx + + c
2 sy
1 2
M = wl , x = 0
2
ea
2
Thus 1
w l 2 = − w l (0
w (0 ) + c From which, c 1
)+ = w l 2
y

2 2 2
ud

2
Hence the particular solution is w (x) 1
M = − w lx + + wl2
2 2
st

1
i.e M = w ( l 2 − 2 lx + x 2 )
2
The Solution of Equations of the Form dy
= f ( y):
dx
Or dy and then the solution is obtained by direct integration i.e
dx =
f (y )
dy
∫ dx = ∫ f (y)

Compiled by www.studyeasy.in
Compiled by www.studyeasy.in

Example: Find the general solution of d y


= 3+ 2y
dx

Solution:,
dy
dx = ,
3+ 2y
Integrating both sides gives dy
∫ dx = ∫ 3 + 2 y

1
x = log ( 3 + 2 y ) + c, General Solution
2

Example: Determine the particular solution of (y 2


−1 ) dy = 3y

.in
dx
Given that y = 1 when x = 2
1
.

 y
6
2
− 1 
sy
Solution: Rearranging d x =   d y
ea
 3 y 
 y 1 
=  −  d y
y

 3 3 y 
ud

2
 y 1  i.e. x =
y 1
lo g y + c ,
∫ dx = ∫
Integrating gives: −
 3 −  d y
 3y  6 3
st

which is the General Solution.

1 1 1 1
When y = 1, x = 2 , thus 2 = − log1 + c, from which, C=2.
6 6 6 3
Hence the particular solution is y2 1
x= − log y + 2.
6 3

Compiled by www.studyeasy.in
Compiled by www.studyeasy.in

o
Example: The variation of resistance, R ohms, of an aluminium conductor with temperature θ c is

given by d R
= α R , where α is the temperature coefficient of resistance of aluminium.
d θ
If R = Ro when θ = 0o c. Solve the equation for R .

Solution: d R of the form dy Rearranging gives: dR


= α R = f ( y ), dθ =
dθ dx αR
dR
Integrating both sides gives: ∫ dθ = ∫αR
1
i .e . θ = lo g R + c
α

.in
Which is the general solution.

Substituting the boundary conditions R = Ro when θ = 0 gives:


0=
α
1
lo g R o + c for which c = −
sy1
α
log R o
ea
Hence the particular solution is
1 1
θ = lo g R − lo g R o
α α
y

1
= (l o g R − lo g R o )
α
ud

1 R
θ = lo g
α R o
R
α θ = lo g .
st

R o

Hence R
eαθ = ⇒ R = Ro eαθ
Ro

The Solution of Equation of The Form dy


= f ( x ). f ( y ) :
dx
A differential equation of the form d y where f ( x ) is a function of
= f ( x ). f (y ):
dx
x only and f ( y ) is a function of y only, may be rearranged as dy
= f ( x ) dx ,
f (y)

Compiled by www.studyeasy.in
Compiled by www.studyeasy.in

And then the solution is obtained by direct integration i.e. dy


∫ f (y)
= ∫ f ( x ) d x.
dy
Example: Solve the equation 4 xy = y2 −1
dx
Solution: Separating the variables gives  4 y  1
 y2 − 1  dy = x dx
 

Integrating both sides gives:  4y  1


∫  y 2 − 1 
dy = ∫ x
dx

On integration, the general solution is 2log( y2 −1) = log x + c

.in
2
or log ( y 2 − 1 ) − log x = c sy 2
 (y 2
− 1)
2
 ( y 2 − 1)
ea
or   or log = ec .
lo g   = c
x x
 
y


ud

Example: Determine the particular solution of = 2 e 3t − 2θ , given that


dt
t = 0 when θ = 0.
st


Solution: = 2 e 3t− 2θ
dt

=2( e3t )( e−2θ )


Separating the variables gives:

dθ 3t
−2θ
= 2 e dt
e

Compiled by www.studyeasy.in
Compiled by www.studyeasy.in

i.e. e2θ dθ = 2e3t dt,


Integrating both sides gives
∫ e2θ dθ = ∫ 2e3t dt ,

e 2θ 2 3t
Thus the general solution is = e +c
2 3
When t = 0, θ = 0, thus: 1
e θ
=
2
e θ
+ c from which
2 3
1
c = − .
6

.in
1 2θ 2 1
Hence the particular solution is : e = e 3t −
2 3 6

Or 3e2θ = 4 e3t −1.


sy
ea
dy
Example: Find the curve which satisfies the equation xy = (1 + x 2 ) and passes through the
dx
y

point ( 0 ,1 ) .
ud

Solution: Separating the variables gives: x dy


d x =
st

(1 + x 2 ) y

1
Integrating both sides gives log (1 + x 2 ) = log y + c
2
when 1
x = 0, y = 1 thus log1 = log1 + c
2

1
from which c = 0 .Hence, the particular solution is log (1 + x 2 ) = log y i.e.
2
1 1
2 2
log (1 + x ) 2 = log y from which y = (1 + x ) 2

Compiled by www.studyeasy.in
Compiled by www.studyeasy.in

Hence the equation of the curve is y = 1− x2

Example: When a cake is removed from an oven, its temperature is measured at 300 o F . Three
minutes later its temperature is 200 o F . How long will it take for the cake to cool off to a room
temperature of 70 o F ?.

.in
Solution: Given T = 70,
m

By Newton’s Law of cooling, we have dT


dt
sy
= k(T − Tm ) , T ( 0 ) = 300

dT
ea
i.e. =k(T − 70) −−−(1) and determine the value of k so that T ( 3 ) = 200,
dt
separating the variables gives: dT Integrating both sides gives
y

= k dt
T − 70
ud

log(T − 70) = kt + c1
st

⇒ T= 70 + c2 ekt −−−−−( 2)
Using initial conditions:

when t = 0, T = 300

T ( 0 ) = 70 + c2 e k .0
300 = 70 + c 2 ∴ c 2 = 230,

equation (2) becomes T = 70 + 230 e kt ------- (3)

Compiled by www.studyeasy.in
Compiled by www.studyeasy.in

At t = 3: T(3)=200, equation (3) becomes

T = 70 + 230e3k
200 = 70 + 230e3k
⇒ 130 = 230e3k
130 3k 1 13
⇒ = e or k = log = −0.19018
230 3 23
thus T ( t ) = 70 + 230 e − 0.19018 t ---------(4)

We expect the cake to reach the room temperature after a reasonably long period of time

.in
T(t) t(time)
750 20.1
740 21.3
730
720
sy 22.8
24.9
710 28.6
ea
70.50 32.3
y
ud
st

Example:

Solve the initial value problem dy x


= − , y (4 ) = −3 .
dx y

Solution: By rewriting the equation as y d y = − x d x w e g e t


y2 x2
∫ y dy = − ∫ x dx and
2
= −
2
+ c1

⇒ x 2 + y 2 = c 2 , c 2 = 2c1

Compiled by www.studyeasy.in
Compiled by www.studyeasy.in

This solution of the differential equation represents a family of concentric circles centered at the
origin.When x = 4, y = -3 so that c2 = 25. Thus the initial value problem determines the circle x2 +
y2 = 25. with radius 5. Particular solution is y = − 2 5 − x 2 , − 5 < x < 5 The
solution curve is the lower semicircle, shown in green color, that contains the point (4,-3)

Example:Weight Reduction Model:

.in
 Weight of person = x kg
 Tries to reduce weight
 Weight loss per month= 10% of weight
 Starting weight = 100kg
dx
= − 0 . 1 x
sy
dt
ea
Initial conditions x =100 at t = 0.Determine x(t) as a function of t.

Analytical Solution of Simple model:


y
ud

dx
Recall the model: = − 0 .1 x x (t = 0 )= 100
dt

dx
st

Cross multiplying, = − 0 . 1 dt
x

dx
Integrating both sides from o to t ∫ x
= − 0 .1 ∫ dt

C+log x(t) = -0.1t. Using initial conditionsC= - log 100. Thus the final solution is

x (t )
log = − 0 . 1t or x (t ) = 100 e − 0 .1t
100

Compiled by www.studyeasy.in
Compiled by www.studyeasy.in

Example: Solve the differential equation 2 dy 2


(x + y ) = a
dx
2
 x + y  dx
⇒   = − − − − − (1)
 a  d y
x + y
L et = t − − − − − (2)
a

Differentiating equation (2) with respect to y we get 1  dx  dt


 1 +  =
a  dy  dy

dx dt
⇒ =a −1
dy dy
dt
⇒ t2 = a−1

.in
dy
dt sy
⇒ t 2 +1= a
dy
ea
dy dt
⇒ = 2 ( var iable separable)
a t +1
y

on integration
ud

y
= t a n − 1 (t ) + c
a
st

 x + y 
⇒ a ta n −1   = y + c ′.
 a 
Homogeneous Equation:

Definition: A function is said to be homogeneous of the nth degree in x and y if it can be put in the
form xn f ( y / x ).
These equations can be put in the form d y
=
f1 (x , y ) ---------- (1)
dx f2 (x , y )
where f 1 and f 2 are expressions homogeneous of the same degree in x and y .

Compiled by www.studyeasy.in
Compiled by www.studyeasy.in

The solution is obtained as follows

Put y = vx in (1) we get

dy f ( x, y ) from (1)
= 1
dx f2 ( x, y )

= F (v ), a function of v

dv
v+ x = F (v )
dx
dv
x = F (v ) − v
dx

.in
dv dx
F (v ) − v
=
x
.
(v.s.) By direct integration we get the solution of the given equation

Example: sy
Solve ( x 2 + y 2 ) dx − 2 xy dy = 0.
ea
The given equation can be put in the form
y

dy x2 + y2 -------(1).
=
ud

dx 2 xy
This is homogeneous in x & y
st

Solution: Put y = vx in equation (1) we get

d v 1 + v 2
v + x =
d x 2 v
d v 1 − v 2
⇒ x =
d x 2 v
2 v d v d x
= ( v .s .)
1 − v 2 x
2
on integration we get − lo g (1 − v ) − lo g x = lo g c
⇒ lo g (1 − v 2 ) + lo g x = − lo g c
lo g (1 − v 2 ) x = K ′

Compiled by www.studyeasy.in
Compiled by www.studyeasy.in

⇒ (1 − v 2
). x = C
2
y
⇒ (1 − ).x = C
x2
⇒ x 2
− y 2 = xC

Example: Solve y 2 d x + ( x y + x 2 ) d y = 0 -----(1)

Equation (1) can be put in the form

dy y2 Which is homogeneous in x and y


=− 2
,
dx xy + x

Solution: Put y = vx and we obtain dv v2


v + x = −
dx v + 1

.in
dv −2v2 + v
⇒ x. =
dx v + 1
(v + 1) dx
⇒ = 0 − − − − − (3 )
v ( 2 v + 1)
+
x sy
Now ( v + 1) v 1
ea
= + − − − − − − (4)
v ( 2 v + 1) v ( 2 v + 1) v ( 2 v + 1)
1 A B
y

Again, = +
v (2 v + 1) v 2v + 1
ud

1 = A ( 2 v + 1 ) + Bv
st

A =1, B=-2

1 1 2
= −
v ( 2 v + 1) v 2v + 1

Equation (4) becomes

v + 1 1 1 2
= + −
v ( 2 v + 1) ( 2 v + 1) v 2v + 1
1 1
= −
v 2v + 1

Equation (3) becomes 1 1 d x


− + = 0
v 2 v + 1 x

Compiled by www.studyeasy.in
Compiled by www.studyeasy.in

on integration we get 1
lo g v − lp g ( 2 v + 1) + lo g x = c
2
1
2
log v − log( 2 v + 1 ) + log x = c
 vx 
log  1  = c
 (2 v + 1 ) 2 

 vx 
 1  = k
 (2 v + 1 ) 2 

Non-homogeneous equation of the first degree in x and y:

These equations of the form

.in
dy ax + by + c
= − − − −− −(1)
dx a′x + b′y + c′

put x = x ′ + h and y = y ′ + k ,
sy where h and k are some constants
to be determined.
ea
Then

dx = dx′ and dy = dy′


y

dy dy′ a (x ′ + h )+ b ( y ′ + k ) + c
ud

∴ = =
dx dx′ a ′ ( x ′ + h ) + b ′( y ′ + k ) + c ′
ax′ + by′ + ah + bk + c
st

=
a ′x ′ + b ′ y ′ + a ′h + b ′ k + c ′

If h and k are determined so that ah + bk + c = 0 & a ′h + b′k + c ′ = 0

Then d y ′
=
a x ′ + b y ′ , which is homogeneous in x ′ and y′, which can be
dx′ a ′x ′ + b ′ y ′

solved by putting y′ =vx′.


This method fails when a : b = a ′ : b ′

Compiled by www.studyeasy.in
Compiled by www.studyeasy.in

a b
i .e . w h e n =
a′ b′

Because the h and k become indeterminate

Suppose a b 1 .
= =
a ′ b ′ m

Then Equation (1) can be written as

dy ax + by + c put a x + b y = v th e n
=
dx m (ax + by ) + c′ a + b
d y
=
d v
d x d x

v + c dv
a + b. =

.in
m v + c′ dx
Then by separating the variables and integrating we get the solution.

Example: Solve
sy
(2 x − 4 y + 5) dy = ( x − 2 y + 3) dx.
ea
Solution: The given equation can be written as

dy x− 2y +3  a b 
=  H ere ′ = ′ 
y

dx 2x − 4y + 5  a b 
ud

(x − 2 y) + 3
= − − − − − − − (1)
2(x − 2 y) + 5
st

Put v=x-2y

dv dy
= 1 − 2
dx dx

dy 1  dv 
= 1 − 
dx 2  dx 

v + 3 1  dv 
From equation (1) , = 1 − 
2v + 5 2  dx 
2v + 6 dv
⇒ = 1 −
2v + 5 dx

Compiled by www.studyeasy.in
Compiled by www.studyeasy.in

dv 2v + 6
=1−
dx 2v + 5
2v + 5 − 2v − 6
=
2v + 5
1
= −
2v + 5
On integration we get

v 2 + 5v = − x + c
2
i.e. ( x − 2 y ) + 5 ( x − 2 y ) = − x + c.
Example: Solve d y x + 2y −3  a b 
=  H ere ′ ≠ ′ 
dx 2x + y − 3  a b 

.in
Solution: Given equation is non-homogeneous in x and y
Put
x = x ′ + h, y = y ′ + k
dy x′ + 2 y ′ + h + 2 k − 3
sy
∴ = − − − − − − (1)
ea
dx 2 x′ + y ′ + 2 h + k − 3

We find h, k such that h + 2k − 3 = 0


 solve by cross multiplication
y

2h + k − 3 = 0
ud

h k 1
= = ∴h =1, k =1
−6 + 3 −6 + 3 1− 4
st

Equation (1) becomes dy′ x′ + 2 y′


=
dx′ 2 x′ + y′
This is homogeneous in x′ and y′
Put
y ′ = vx ′
d y ′ d v
∴ = v + x ′
d x ′ d x ′
1 + 2 v
=
2 + v

Compiled by www.studyeasy.in
Compiled by www.studyeasy.in

On simplification we get

2+v dx ′
2
dv = (var iables are separable )
1− v x′
2+v dx ′
∫ 1 − v2 dv = ∫ x′
2+v
∫ (1 + v)(1 − v) dv = log x′ + c − − − − − ( 2 )
2+v A B
now = +
(1 + v )(1 − v ) (1 + v ) (1 − v)
A = 1/ 2, B = 3/ 2

.in
Equation (2) becomes

 1 3 
∫  2(1 + v ) 2(1 − v ) dv = log x′ + c
+ sy
1 3
⇒ log(1 + v ) − log(1 − v ) = log x ′ + c ,
ea
2 2
On simplification we get
y

1 + v
ud

3
= lo g x ′2 c .
(1 − v )
st

Compiled by www.studyeasy.in
Compiled by www.studyeasy.in

.in
sy
y ea
ud
st

Compiled by www.studyeasy.in
Compiled by www.studyeasy.in

Differential Equations

Unit-7

Exact Differential Equations: M dx + N dy = 0


Verify the condition
∂ M ∂ N
=
∂ y ∂ x

Then integrate M dx with respect to x as if y were constants,

then integrate the terms in N d y which do not contain terms in x and equate sum of
these two integrals to a constant.

Example: Solve

.in
( a 2 − 2 xy − y 2 ) dx − ( x + y ) 2 dy = 0
M = a 2 − 2 xy − y 2 ; N = − ( x + y ) 2 = − x 2 − y 2 − 2 xy


∂M
∂y
= −2 x − 2 y;
∂N
∂x
= − 2x − 2 y ∴
∂M
∂y
=
∂N
∂x
sy
∴ The given equation is an exact differential equation.∴ Solution is given by
ea
∫ (a ∫ (− y
2
− 2 xy − y 2 )dx + 2
)dy = 0
⇒ 3a 2 x − 3 x 2 y − 3 y 2 x − y 3 = 3c
y
ud

Example:

y y
Solve (e + 1) co s d x + e s in x d y = 0
st

∂M ∂N
= e y co s x ; = e y co s x
∂y ∂x

∫ (e
y
Solution is : + 1 ) cos x dx = c

(e +1)sinx = c
y

Integrating Factors:

Sometimes a differential equations which is not exact may become exact on multiplication by a
suitable function known as an Integrating Factor.

Compiled by www.studyeasy.in
Compiled by www.studyeasy.in

For example the differential equation ydx − xdy = 0


is not exact, but when multiplied by 1 1 1
or or by
y2 x 2
xy
it becomes on exact equation. The number of integrating factor is infinite for a given equation.

Rules For Finding The Integrating Factors::

1
Rule I: when Mx + N y ≠ 0 and the equation is homogeneous then is an Integrating
Mx + Ny
factor of Mdx + Ndy = 0

Example: Solve: (
x 2 ydx − x 3 + y 3 dy = 0 )

.in
Solution: Mx+Ny=-y4 I.F.=-1/y4

x2  x3 1 
sy
Multiply Given equation By I.F.
− 3 dx +  4 +  dy = 0
y y y
ea
On integration we have − x3
+ log y = c
3y3
y

Rule II: When Mx - Ny ≠ 0 and the equation has the form f1(xy)ydx+f2(xy)xdy=0
ud

1
then Integrating factor is
Mx − Ny
st

Example: Solve (1 + xy) ydx + (1 − xy)xdy = 0


Mx - Ny = 2x2y2 I.F.=1/2x2y2

dx dy ydx + xdy
Multiply given equation by I.F. we get − + on intergration
x y x2y2

x 1
log − = log c
y xy

Compiled by www.studyeasy.in
Compiled by www.studyeasy.in

∂M

∂N
∫f(x)dx
Rule III: When ∂ y ∂x is a function x alone say f(x) then e is an Integrating factor
N

Problem:

Solve (x2+y2+2x)dx + 2ydy = 0

∂M ∂N

= e ∫
∂y ∂x 2 y 1dx x
= 2 y
= 1 I.F. = e
N

Multiplying given equation by I.F. and it becomes exact

∫ e (x )
2
Solution of given equation is given by
x
+ y 2 + 2 x dx = c

.in
x 2e x
+ y 2e x
= c
∂N ∂M
sy
e∫
− F ( y ) dy
∂x ∂y
ea
Rule IV: When is a function y alone say F(y) then is an Integrating
M
factor
y

Example: solve (y 4
) (
+ 2 y dx + xy 3 + 2 y 4 − 4 x dy = 0 )
ud

∂N ∂M

− 3 y2 + 2 ( )
st

∂x ∂y −3
= =
M y y2 + 2 ( y )
I .F . = e ∫
F ( y ) dy 1
=
y −3
Multiply given equation by I.F. and it becomes

 2   4x 
 y + 2  dx +  x + 2 y − 3  dy = 0
 y   y 

Compiled by www.studyeasy.in
Compiled by www.studyeasy.in

 2 
Solution is given by  y + 2  x + y 2 = c
 y 
Linear Differential Equation:

A differential equation is said to be linear if the dependent variable and its derivatives appear
only in the first degree.

dy
The first order linear differential equation is of the form + Py = Q where P and Q are
dx
functions of x only or constants.

Solution of Linear differential equation:


y (I . F . ) = ∫ Q (I . F . )dx + c

I.F. = e ∫
Pdx
where

.in
1 dy
Example: Solve + 4 y = 2 given the boundary conditions x =0 when y = 4.
x dx

dy
sy
+ 4 xy = 2 x
ea
Solution : Rearranging gives
dx

e∫
pdx 2
= e2x
y

Integrating factor .
ud

1

2 2 2
2 x 2 x 2 x
Solution gives: ye = e (2 x )dx = e + c.
2
st

1
Using B.C. Obtain C=7/2.Hence particular solution gives y =
2
(
1 + 7e − 2 x 2
).

Compiled by www.studyeasy.in
Compiled by www.studyeasy.in

dy
Example: Solve the differential equation = sec θ + y tan θ

given the boundary conditions y = 1 when θ = 0.

dy
Solution: Rearranging gives − (tan θ ) y = sec θ

e∫
pdx
Integrating factor = e ln (cos θ ) = cos θ .

Solution gives: y cos θ = ∫ cos θ (sec θ )d θ = θ + c .

θ y =
(θ + 1 ).
When = 0, y = 1, thus c=1. Hence particular solution is

.in
cos θ

sy
y ea
ud
st

dy y 2
Example: Solve the differential equation + = x
dx x

Solution:

e∫
pdx
Integrating factor = e ln x
= x .

y x = ∫ x 2 x dx
Solution gives: x3
y= +c
4

Compiled by www.studyeasy.in
Compiled by www.studyeasy.in

Equations Reducible to linear form Or (Bernoulli’s equation):

dy n
+ Py = Qy Dividing by yn
dx

dy

.in
y −n + Py − n +1 = Q − − − − − (1)
dx

dy dv
put v=y-1+n we get (− n + 1 )y (−
sy
n + 1−1 )
dx
=
dx

dy 1 dv
ea
(− n )
i.e y =
dx (− n + 1 ) dx

1 dv
y

+ Pv = Q
(− n + 1 ) dx
ud

dv
+ (− n + 1)Pv = (− n + 1)Q
st

dx

Which is linear in v

dy y (2 )
Example: Solve + = y
dx x

(− 2 ) dy (− 1 ) 1
Solution: y + y = 1 − − (1 )
dx x

(2 ) dy dv
-1
Put y =v , − 1 y =
dx dx

Compiled by www.studyeasy.in
Compiled by www.studyeasy.in

dv 1
− + v = 1
dx x
From (1) dv 1 which is linear in v
i .e − v = − 1
dx x
1
∫ − dx
e∫
pdx 1
Integrating factor = e x =
x

1 1
Solution is v
x
= ∫ −1
x
dx + c

.in
1
= − log x + c
xy
sy
y ea
ud

dy
− 2 y tan x = y 2 tan 2 x
st

Example: solve
dx

− 2 dy −1 2
Solution: y − 2y tan x = tan ()
−− 1
dx

−2 dy dv
Put y-1=v − y =
dx dx
dv 2
Fron (1) + 2 v tan x = − tan x which is linear in v
dx

∫ pdx ∫ 2 tan xdx 2 log sec x 2


Integrating factor e =e =e = sec x

Compiled by www.studyeasy.in
Compiled by www.studyeasy.in

2 2 2
∴ Solution is v sec x = ∫ − tan x sec xdx + c

3
− 1 2 tan x
y sec x = − + c
3

Orthogonal trajectory

Introduction:

.in
The word “Orthogonal” comes from the Greek “right angle” and the word “trajectory”
comes from Latin “cut-across”. Hence the curve that curve that cut across the other at
right angle is called orthogonal trajectory.
sy
y ea
ud
st

Rules to find the equation of orthogonal trajectories of a family of cartesian


curves:

• Diff f(x ,y , c)=0 and eliminate c.

• Resulting equation gives the DE of the family f(x ,y , c)=0


 
• i.e. F  x, y. dy  = 0
 dx 
dy  dy 
dx  
• Replace dx by - dv in F x, y,  = 0
 dx   
• The differential equation of orthogonal trajectory is F  x, y,- dx 
 =0
 dy 

 
• Integrate F  x , y , − dx 
 = 0 to get the equation of the required orthogonal
 dy 
trajectory 

Compiled by www.studyeasy.in
Compiled by www.studyeasy.in

Rules to find the equation of orthogonal trajectories of a family of polar curves:

Let the equation of given family of curves be f(r , θ , c)= 0

• Diff f(r , θ , c)= 0 and eliminate c.

• Resulting equation gives the DE of the family f(r , θ , c)= 0


 
• i.e. F  r, θ . ddrθ  = 0
 

• Replace dr by - r 2 d θ in F  r, θ , dr  = 0


dθ dr  d θ 
 d 
• The DE of OT is F  r, θ ,-r 2 θ  = 0
 dr 
 dθ 
• Integrate F  r , θ , − r 2 dr 

= 0 to get the equation of the required

orthogonal trajectory

.in
Example :Find the orthogonal trajectories of parabolas y=c x2

Solution: Differentiate given equation we get dx


dy dy x
sy
dy = 2 Cx = 2  y  x = 2 y

 x
2  x
dx
Replace dx = − dy we get dx = − 2 y is the DE of OT.
ea
 V.S  2
2 y dy = − x dx 
On integration we get y2 + x = c
2
y
ud
st

Example: Find the orthogonal trajectories of the series of hyperbolas xy= k2

Solution: Differention given equation w.r.t x we get y + x y1=0 Replace y1 by − y1


1
The differntial equation of the OT is
dx dx
y−x = 0 or - x = −y ⇒−xdx= −ydy⇒ xdx− ydy= 0
dy dy

Compiled by www.studyeasy.in
Compiled by www.studyeasy.in

x 2 − y 2 = c ⇒ x 2 − y 2 = 2c = c Which the required OT


2 2 1

Example: Find the orthogonal trajectories of circles

x2+(y-c)2=c2 (1)

.in
solution: Differentiation w.r.t x , 2x + 2(y-c)y1=0

or x + (y-c)y1 = 0 (2)
sy
From (1) x2+y2 – 2cy = 0 or c = (x2+y2) / 2y Substitute this in (2) we get
 2 2   2 2 
y − x + y  y = 0 or x +  y − x  y = 0
ea
 
x+
 2y  1  2y  1
   


 y 2 − x 2  y = - x ⇒ 
 x 2 − y 2  y = x ⇒ y = 2xy
y

 
 2y  1  2y  1 1 x − y2
2 (3)
   
ud

1
Replace y1 by − y The differential equation of the OT is
1
st

2 xy y2 − x2
− y1 = ⇒ = y or 2 xyy − y 2 + x 2 = 0 (4)
2
1 x −y 2 2 xy 1 1

dy 2 2 dy y − x
2xy − y + x = 0 dividen by 2xyor 2x ⇒ − =
dx dx 2x 2y

dy y 2 − x dy dz
y − = y
dx 2 x 2 (5) Put y2/2 = z Diff w. r . t. x =
dx dx
dz z −x
Equation (5) becomes − =
dx x 2
linear in z I.F.=1/x solution is
1 1 x y2 x
z = ∫ − dx + c = − + c or = − + c or y 2 + x 2 − 2 cx = 0
x 2 2 2x 2

Compiled by www.studyeasy.in
Compiled by www.studyeasy.in

• Or (x −c)2 − y2 = c2 be the equation of OT

Example: The orthogonal trajectories of cardiod r=a(1+cos(θ)) where a is a parameter.

.in
Solution: Given equation r = a(1+cos(θ)) Diff. w.r.t. θ
dr = −asinθ ⇒ a = − 1 dr
dθ sinθ dθ

r = − 1 dr (1 + cos θ ) ⇒ dr = − r sin θ =
sin θ d θ d θ 1 + cos θ
sy
− 2 r sin θ  cos θ 
 2
2
2 cos  θ 
 2  = − r tan θ 
 
 2
 2
ea
Which is the DE of family r=a(1+cos(θ))
dr   dr  
= cot   ⇒ r = cot   d θ Integrate this we get
θ θ
Replace  2  2
d θ
y

  
 sin θ  
log r = log   2   + c on simplification r = c(1- cos(θ))
ud


 1 
 2 
 
st

Compiled by www.studyeasy.in
Compiled by www.studyeasy.in

.in
sy
y ea
ud
st

Compiled by www.studyeasy.in
Compiled by www.studyeasy.in

Unit-VII: Linear Algebra-I

 Purpose of lession :

 To show what are the matrices , why they are useful, how they are classified as various types
and how they are solved.

 Introduction: Matrices is a powerful tool of modern Mathematics and its study is becoming
important day by day due to its wide applications in almost every branch of science and
especially in Physics and Engineering. Matrices are used by Sociologists in the study of
dominance within a group, by Demographers in the study of births and deaths, etc, by
Economist in the study of inter-industry economists, input-output tables and for various
practical business purposes, by statisticians in the study of ‘design of experiments and for
various practical business purposes, by statisticians in the study of ‘design of experiments and
multivariate analysis by Engineers in the study of ‘network analysis’ which is used in electrical
and communication engineering.

.in
Application:

sy
The following table shows the number of transistors and resistors purchased by a manufacturer
from suppliers A and B for the first week of January
ea
A B
y

Transistor 400 800


ud

Resistors 600 500


st

a) Write the data in the table as 2 x 2 matrix S1

b) b)Use scalar multiplication to find a matrix S2 whose entries are all 10 % large than the
corresponding entries of S1

c) Suppose that S2 is the supply matrix for the second week of January. Find S1 + S2 and explain
what its entries represent.

Solution: a) The supply table can be written as the following 2 x 2 matrix

 
 400 800 
S= 



1 600 500




Compiled by www.studyeasy.in
Compiled by www.studyeasy.in

b) If the entries of S2 are 10 % large than the entries of S1, then


S2 = S1 + 0.10 S1 = 1.1 S1
S2=1.1 
 400 800 440 880


 
= 

 600 500 660 550

c) The entries of S1 + S2 give the total number of transistors and resistors purchased from
suppliers A and B for the first two weeks of January.


 400 800   440 880   840 1680 
s1 + s2 =  +
 
=
  



600 500    660 550 
  1260 1050 

Example : The following table 1 Shows the number of economy, mid-size and large cars rented by

.in
individuals and corporations at a rental agency in a single day Table 1

Economy mid-size large

Individuals 3 2
sy 6
ea
Corporations 5 2 4
y
ud

Table 2
st

Bonus points Free miles

Economy 20 50

Mid-size 30 100

Large 40 150

Table 2 shows the number of bonus points and free miles given in a promotional program for each of
the three car types. Find
a) Total bonus points for individuals
b) Total free miles for individuals
c) Total bonus points for corporations
d) Total free miles for corporations

[3 2 6]

Compiled by www.studyeasy.in
Compiled by www.studyeasy.in

The 1 x 3

row matrix from the Table 1 – Represents the number of economy, medium size and large rented
by Individuals
 20 
• The 3 x 1 column matrix  
30 
 40 

From table 2 represents the bonus points given for each economy, mid-size and large car that is
rented.
The product of these two matrices is a 1 x 1 matrix whose entry is the sum of the products of the
corresponding entries
 20 
[3 2 6] 30  = [360]
 40 

.in
The product of above two matrices, gives the total number of bonus points given to individuals on the
rental of the 11 cars.

Eco Mid Lar Bonus Pts


sy Free Miles


3 2 6  20 50 
ea
  Eco    360
   Mid   
1250

 5 2 4 

X    30 100 =  
 320 1050
y

 Lar    
  40 150
ud


st

Traffic control in the future:

The above picture shows the intersections of four one way streets. 400 cars per hour want to enter
intersection P from the north on First Avenue while 300 cars per hour want to head east from
intersection Q on Elm Street. The letters w, x, y, and z represent the number of cars per hour passing
the four points between these four intersections, as shown in picture.

A) Find values for w, x, y and z that would realize this desired traffic flow.

Compiled by www.studyeasy.in
Compiled by www.studyeasy.in

B) If construction on Oak Street limits z to 300 cars per hour, then how many cars per hour would have
to pass w, x, and y.

Solution: The solution to the problem is based on the fact that the number of cars entering an
intersection per hour must equal the number leaving that intersection per hour, if the traffic is to keep
flowing

Since 900 cars ( 400 + 500) enter intersection P, 900 must leave, x + w = 900. Writing a similar equation
for each intersection yields the system of equations.

W + x = 900
W + y = 1000
X + z = 850
y + z = 950

R4=R4 – R2 R23

.in
1 1 0 0 900  1 1 0 0 900
    1 1 0 0 900
1 0 1 0 1000 0 − 1 1 0 100   
  ≈  0 − 1 1 0 100 
0
0

1 0 1
0 1 1
850 
950 
0
0

1 sy
0 1 850
− 1 1 0 100 
≈
0
0
1 0 1 850

 0 0 0 0 
ea
1 1 0 0 900 1 1 0 0 900 
   
0 1 0 1 850 0 1 0 1 850 
y

≈  ≈
0 − 1 1 0 100  
ud

0 0 0 0 0 
0 0 1 1 950 
 0
 0 0 0 0 
st

w + x = 900 w = 50 + z

x + z = 850 x = 850 - z

y + z = 950 y = 950 – z

The system is a dependent system does not have a unique solution.(z is non negative integer)

b)If z is limited to 300 because of construction, then the solution is ( 350,550,650,300) To keep traffic
flowing when z = 300, the system must route 350 cars past w, 550 past x and 650 past y.

Compiled by www.studyeasy.in
Compiled by www.studyeasy.in

Definition Matrices and Types of Matrices:

Twenty six female students and twenty-four male students responded to a survey on income in a college
algebra class. Among the female students , 5 classified themselves as low-income, 10 as middle-income,
and 11 as high-income. Among the male students, 9 were low-income, 2 were middle-income, and 13
were high-income. Each student is classified in two ways, according to gender and income. This
information can be written in a matrix:

L M H

Female  5 10 11

Male  
9 2 13


Square matrix: In this matrix we can see the class of makeup according to gender and income. A matrix

.in
provides a convenient way to organize a two way classification of data.

A matrix is a rectangular array of real numbers. The rows of a matrix run horizontally, and the
columns run vertically. sy
A matrix with m rows and n columns has size m x n (read “ m by n”). The number of rows is
ea
always given first. For example, the matrix used to classify the students is a 2 x 3 matrix

When m = n i. e the number of rows is the same as the number of columns the array is called a square
y

matrix. The others are called rectangular matrix.


ud

Row-matrix: In a matrix if there is only one row it is called a row matrix

Column matrix: In a matrix if there is only one column it is called a column matrix
st

Null matrix : In a matrix, if all the elements of matrix are zero, it is called null matrix and is denoted by o

Equal Matrix: Two matrices are said to be equal if

i) They are of the same type i.e they have same number of rows and columns

ii) The elements in the corresponding positions of the two matrices are equal.

Location of an element: To locate any particular element of a matrix the elements are usually denoted
by a letter followed by two suffixes which respectively specify the row and the column in which it
appears. Thus the element occurring in pth row and qth column will be written as apq.

Note: The matrices are generally denoted by capital letters.

Diagonal Elements of a Matrix: An element aij of a square matrix is said to be a diagonal element if i = j.

Compiled by www.studyeasy.in
Compiled by www.studyeasy.in

Scalar Multiple of a matrix: If k is a number and A is a matrix, then kA is defined as the matrix each
element of which is k times the corresponding element of A.

Triangular Matrix: if every element above (or below ) the diagonal is zero, the matrix is called a
triangular matrix

Example:

 1 0 0
 1 2 4  
  
   2 3 0lower tria ngular
 0 3 5  upper tria ngular  
 



0 0 6  
4 5 6

Transposed Matrix: The matrix of order n x m obtained by interchanging the rows and columns of a

.in
matrix A of order m x n is called the transpose matrix of A or transpose of the matrix and is denoted by
A’ or At.

sy
Representation of Points: A point is represented in two dimensions by its coordinates. These two values
are specified as the elements of a 1 x 2 matrix i.e. x 
[x , y ] or  

y  2 x 1
ea


 x
y

 
[x y z ] is used or y  

ud

 
In three-dimensions a 1 x 3 matrix  z  3 x1

st

 x 
[x y ] or  
Row matrix or column matrix like 
y are frequently called position vectors.

 

Elementary Transformations:

An elementary transformation, which is also known as E-transformation, is an operation of any one of


the following types.

i) Interchange of two rows (columns)

ii) The multiplication of the elements of rows (columns) by a non-zero number.

iii) The addition to the elements of a row (column), the corresponding elements of a row ( column)
multiplied by any number

An elementary transformation is said to be row-transformation or column transformation according


as it applies to rows or columns.

Compiled by www.studyeasy.in
Compiled by www.studyeasy.in

Symbols to be used for elementary transformation:

i) Rij for the interchange of ith and jth rows

ii) Ri(k) for the multiplication of ith row by k ≠ 0

iii) Rij(k) for the addition to the ith row, the products of the jth row by k. Similarly we use the symbols
Cij, Ci(k), Cij(k) for the corresponding column operations.

Equivalent Matrices:

Two matrices A and B are said to be equivalent if one can be obtained from the other by a
sequence of elementary transformations. Two equivalent matrices have the same order. The
symbol ~ is used for equivalence.

Minor : Let A be a matrix square or rectangular, from it delete all rows leaving a certain t-rows
and all columns leaving a certain t-column. Now if t > 1 then the elements that are left,

.in
constitute a square matrix of order t and the determinant of this matrix is called a minor of A of
order t

Rank of a Matrix:

Definition: A matrix is said to be rank r when


sy
ea
I ) It has at least one non-zero minor of order r and

ii) Every minor of order higher than r vanishes.


y

Briefly, the rank of a matrix is the largest order of any non-vanishing minor of the matrix
ud

 1 2 3
For example:1) A =   , ρ(A) = 1
 2 4 6
st

 1 2 3 4 
2)   , ρ(A) = 1
 
A = 2 4 6 8 
 
 
3 6 9 12 

Note : i) If a matrix has a non-zero minor or order r, its rank is ≥ r


ii) If all minors of a matrix of order of r+1 are zero, its rank is ≤ r


1 2 3

Example:  
A 2 3 4
= A = 2 ≠ 0 ρ (A) = 3
 

0 2 2


 0 1 0 0 
  1 0 0 
  
 0 0 1 0  
A=    0 1 0 ≠ 0 ρ (A) = 3

 0 0 0 1  



  
0 0 1 
 0 0 0 0 

Compiled by www.studyeasy.in
Compiled by www.studyeasy.in

Example:. Find the rank of the matrix


 0 1 − 3 − 1   0 1 − 3 − 1 
 
   
 1 0 1 1  1  1 − 2 0 
=   R = R + R ;R = R + 2R ≈   operate

3 1 0 2  2 2 1 3 3 1 3 
3 − 6 0 
  
   
1 1 − 2 0  1  1 − 2 0 
0 1 − 3 − 1  0 1 − 3 − 1 
 
   
1 1 − 2 0  1 1 − 2 0 
≈   R = R - R ;R = R − R ≈   R
  3 3 2 4 4 2 
0  12
1 1 − 2 0  0 0 0
   
 1 1 − 2 0   0 0 0 0 
1 1 − 2 0  

 
0 1 − 3 − 1
≈   ρ ( A ) = 2

0 0 0 0 
 

.in
 0 0 0 0 
2  6 5 
 
 
Example:. Reduce the matrix A = 2  5 4  to a unit matrix by only elementary
 

row transformations

 5 16 13 sy 

ea
Solution:

2  6 5 

 
A= 2  5 4 R = R − R :R = R − 2R
2 2 1 3 3 1
y

 

5  16 13 
ud


2 6 5  1 4 3 
 
   
~ 0
 −1 − 1 R ~ 0 −1 − 1 R = R − 2 R
  13   3 3 1

1
 4 3  
 2 6 5 
st

 1 4 3   1 4 3 
   
   
~ 0 −1 −1 R = R − 2R ~ 0
  −1 −1 R = R − 3R R = R − R

 3 3 2  1
 1 3 2 2 3
   
0
−2 −1 
0 
0 1 
1 
4 Find0 the
Example:.
  rank of the matrix 1 0 0 
   
~  0 1 0  R = R 0 + 4 R2 ~30 −1 0 ρ (A) = 3
  1 1 2  

 0 0 1   
 0 0 1 
A = 0 4 6  c = c - c
  3 3 2
 

0 6 9 
 0 2 1  0 0 1
 
   
~  0 4 2 c = c − 2c ~ 0 0 2 R = R − 2R ; R = R − 3R 
  2 2 3  2 2 1 3 3 1 

 0 6 3  0 0 3  

 0 0 1  1 0 0
 
  I 0
   

~  0 0 0c ~ 0 0 0 ~ 1
    
 13  
0 0  

    
 0 0 0
0  0
Compiled
 by www.studyeasy.in

Compiled by www.studyeasy.in

Normal form of a Matrix:

By means of a sequence of elementary transformations every matrix A of order m x n can be reduced to


one of the following form

 Ir 0 

 
 0 0 

0 1 2 − 2

Example: Reduce the matrix A to its normal form  
A= 4 0 2 6 

 
 
 2 1 3 1 

.in
Solution:
0  1 2 − 2  1 0 2 − 2 
 
   
A= 4  0 2 6 c ~ 0 4 2 6 R = R − R

1
2






0
1
2
3
− 2 
 12
1 


 1 2
sy
3
1

 3
1 
3 1

0 0 0 
   
R
ea
~ 0  4 2 6 c = c − 2 c ; c = c + 2 c ~ 0 4 2 6 2
  3 3 1 4 4 1   2

0 
2 1 3  

0 2 1 3 
1  0 0 0  1 0 0 0 
 
y

   
~ 0  2 1 3  R = R − R ~ 0 2 1 3 c = c − 2 c ; c = c − 3c
  3 3 2   2 2 3 4 4 3
ud


0  2 1 3  
 0 0 0 0 
 1 0 0 0  1 0 0 0 
 
   
~  0 0 1 0 c ~ 0 1 0 0ρ ( A) = 2
st

  23  


0 0 0 0  

0 0 0 0 
Consistency of a system of linear equations:

Consider the following system of m linear equations in n-unknowns x1, x2, x3,…..xn

a11 x1 + a12 x2 + a13 x3 + …….a1n xn = b1

a21x1 + a22 x2 + a23 x3 + …….a2n xn = b2

…………………………………………

am1x1 + am2 x2 + am3 x3 + …….amn xn = bn The system of equations can be written as

Compiled by www.studyeasy.in
Compiled by www.studyeasy.in

 a a .. ,, a   x   b 
 11 12 1n   1  
 1 

   
 a a .. ,, a  x    b 
 21 12 2n  2   2 
 a a a   
A =  31 32
.. ..
3n  X = x 



B = b
 
  3   3 
 .. .. .. .. ..  ..   .. 
     
     

a a .. .. a mn  x n  bn 
m1 m 2    
 a a .. a b 
 11 12 1n 1 
 
 a a .. a b 
 21 12 2n 2 
D =  a 31 a
32
.. a
3n
b
3


 
 .. .. .. .. .. 
 
 
 a a .. a mn bn 
 m1 m 2 

Conditions for Consistency:

.in
 Here the system of equations AX = B is called the non homogeneous set of equations and the
system of equations A X = 0 is called the homogeneous set of equations.

sy
 A system of linear equations is said to be consistent if it possesses a solution and inconsistent (
not consistent) if it does not possess a solution
ea
 A homogeneous system always possesses at least one solution (namely the trivial solution ), a
homogeneous system is always consistent. But a non–homogeneous system may or may not be
consistent.
y

 The equation A X = B is consistent i.e. has a solution (unique or infinite ) iff the matrices A and D
ud

are of the same rank i.e. ρ(A) = ρ(D).

 i) If ρ(A) = ρ(D)= n, the number of unknowns, then only the given system of equations will have a
st

unique solution

 ii) If ρ(A) = ρ(D) < n, then the given system will have infinite solutions.

 iii) If ρ(A) ≠ ρ(D) then the given system of equations will be inconsistent or will not have any
solution.

Example: Test the consistency of the following equations


4x – 2y + 6z = 8
x + y – 3z = -1
15x – 3y + 9z = 21
Solution:  4

− 2 6  x 
 
 8 
 
    
A =  1 1 − 3 , X =  y  , B =  - 1 
     

15

− 3 9  
 z  

21 

 4 − 2 6 8 

  R
D = 

A :D 

= 1
 1 − 3 − 1 R ,R = 3
  12 3 3
 
15
 − 3 9 21 

Compiled by www.studyeasy.in
Compiled by www.studyeasy.in

1 1 − 3 −1

 
4 −2 6 8  R = R − 4R ; R = R − 5R
  2 2 1 3 3 1
 
5 −1

3 7 
1 1 − 3 −1

 
0 − 6 18 12 R = R − R
  3 3 2

0 − 6 18 12

1 1 − 3 −1

  1
0 − 6 18 12 R = − R
  2 6 2

0 0 0 0 

.in

1 1 − 3 −1 1 0 0 1 
  
   
0 1 − 3 − 2R = R − R ⇒ 0 1 − 3 − 2  sy 
  1 1 2  
   
0 0 0
 0  0 0 0 0  
ea
If ρ(A) = ρ(D)= 2< number of unknowns, the system is consistent and has an infinite number of solutions
y

Example: Show that the following equations are inconsistent


ud

2x + y + 5z = 4

3x – 2y+2z = 2
st

5x – 8y -4z =1

2 1 5  x 4
  
Soln.: Ax = B i.e.    
3 −2 2  y = 2

    
D=[A/B] = 2 1 5 4 
 5 −8 −4 z  1

 
3 − 2 2 2R = R − R ;R = R − R
  1 2 1 3 3 2
 
5 − 8 − 4 1

 1 − 3 − 3 − 2

 
 3 −2 2
2  R = R − 3R ; R = R − 2R
  2 2 1 3 3 1
 
2 − 6 − 6 −1
1 − 3 − 3 − 2

 
0 7 11 8
 

0
 0 0 3  Compiled by www.studyeasy.in
Compiled by www.studyeasy.in

ρ(A) = 2, ρ(D)=3 ρ(A) ≠ ρ(D) Hence the given equations are inconsistent

Example: Show that the system of equations


2x –y +z = 4
3x –y +z = 6
4x – y +2z =7
-x + y -z = 9 is consistent.
Solution:
 2 −1 1  4 
 x    
 
   
 3 −1 1 
 6  
A=  X = y , B =  
 
   

.in
 4 −1 2 
 7  
 z 
   

−1 1 −1   9 
 2 − 1 1 4   R = R + 2 R 
  

 3
D =  A : B  = 

 4
− 1 1
−1 2 7 
6
  1
 
 R
  2
1
= R + 3R 
2
4 
4

sy
   R = R + 4 R 
ea
 0 1 − 1 − 11
 − 1 9   3
22
 3

4

  R = R − 2R
   
 0 2 − 2 33   2 2 1 
 
   R = R − 3R 

0 3 − 2 43   3 3 1 
   
R = R − R
y

   
 −1 1 −1 9   4 4 1 

 0 1 −1 22 
ud

   R = R
+ R 
   1 1 3 
 0 0 0 − 11   
   R = ( − 1) R 

0 0 1 − 23   4 4 
   
   
 −1 0 0 − 13 
st

 0 1 0 − 1   1 0 0 13   1 0 0 13 
    
     
 0 0 0 − 11   0 0 0 − 11   0 1 0 −1 
  R ⇒   R ⇒   R

 0 0 1 − 23  14 
 0 0 1 − 23 

24 
 0 0 1 − 23 

14
     
 1 0 0 13   0 1 0 −1   0 1 0 − 11 

ρ(A) = 3, ρ(D)=4 ρ(A) ≠ ρ(D) The system is inconsistent

Example : Investigate the values of λ and µ so that the equations


2x + 3y + 5z = 9
7x + 3y – 2z = 8
2x + 3y + λ z = µ
have i) Unique solution ii) no solution iii) an infinite number of solutions

Compiled by www.studyeasy.in
Compiled by www.studyeasy.in

Solution:
 2 3 5   x   9 
     
     
 7 3 − 2   y  =  8  (1)
     
   

2 3 λ 

 z 
 
µ 

i) If equation (1) has a unique solution then the determinant of the coefficient matrix A≠0

2 3 5
A = 7 3 −2 =155-λ ∴A ≠ 0. λ ≠ 5
2 3 λ
i.e the given system of equations will have a unique solution if λ≠ 5 whatever may the value of µ.

.in
2  3 5 9 

 
D =  A / B  = 7  3 −2 8 R =R −R




2 3 λ
sy
µ



3


3 1


2 3 5 9 
ea
 
 7 3 -2 8 
 
 

0 0 λ -5 µ -9 
y
ud

If λ = 5 and µ≠ 9, then ρ(A) = 2, ρ(D) = 3 ρ(A) ≠ ρ(D) Hence the given system of equations has no
solution

2 8 5 9

 
c) If in the augmented D, λ = 5 and µ= 9 then D= 7 3 −2 8
st


 

 0 0 0 0

It is clear that ρ(A) = ρ(D) = 2 (< 3 the number of unknowns ) Therefore their exist infinite number of
solutions to the given set of equations

Compiled by www.studyeasy.in
Compiled by www.studyeasy.in

Basically, there are TWO types of methods available, viz, Direct methods and Indirect methods or
Iterative methods.
Direct Method:
i) Cramer’s rule
ii) Gaussian Elimination
iii) Gauss-Jordan method
iv) Choleskey Method
Indirect Method:
i) Jacobi Method
ii) Gauss seidal method
iii) Successive Over-Relaxation Technique

Gauss Elimination method:


Gauss Elimination method applied to three linear equations. First we explain this method applied to a

.in
particular systems of order three given by

a11 x1 + a12 x2 + a13 x3 = b1


a21x1 + a22 x2 + a23 x3 = b2 (1)
a31x1 + a32 x2 + a33 x3 = b3 sy
The augmented matrix of the above system
ea

 a a a b 
 11 12 13 1

 a a a b 
 21 22 23 2
 a a a b 

y

 31 32 33 3
ud

a
− a 21
To eliminate x1 from the second equation multiplying the first equation by 11 and add it to
st

the second equation.

Similarly to eliminate x1 from the third equation multiply the first equation by a and add it to the
third equation . Thus we get the matrix.
−a31
11
 
 
 
 
 a a a b 
 11 12 13 1 
 a 
 a a a 
 a − 21 a a − a 21 a a − a 21 a b − a 21 b 


21 a 11 22 11 12 23 11 13 2 11 1


 11 
 a a a a 

 a − 31 a a − 31 a a − 31 a b − a 31 b 

 31 a 11 32 a 12 33 a 13 3 11 1 
 
 11 11 11 

 a a a b 
 11 12 13 1 

~
 0 a′ a′ b′ 
 22 23 2 
 0 a′ a′ b′ 

 32 33 3 
Compiled by www.studyeasy.in
Compiled by www.studyeasy.in

a
− a 21 a
Where 11 , − 31are called the multipliers for the first stage of elimination. The first
a  
equation and a
equation is called the pivotal11
11 a11≠0 is called the first pivota ′ is the new pivot
 

and the multiplier is a 32




22
a′
22

a′
− 32
i.e. multiply the second row by a′ and add it to the third row.
22

.in
 
 
 
 
 a a a b 

~





11
0
12
a′
22
a′
13
a′
23
a′
sy 1
b′
2
a′





 
a ′ − 32 a ′ a ′ − 32 a ′ b ′ − 32 b ′
ea
 0 


32 a ′ 22 33 a ′ 23 3 a′ 2 

 22 22 22 


b 
y

 a a a
 11 12 13 1 
ud

~

 0 a′ a′ b′ 
 22 23 2 
 0 0 a ′′ b′′ 
st


 33 3 
The values of x1, x2 and x3 can be obtained by the substitution

a11x1 + a12x2 + a13x3 = b1


a22
′ x2 + a23
′ x3 = b2′
a33
′′ x3 = b3′′
b3′′
Fromthelast equation x3 =
a′′
Remark: This method fails if one of the pivots say a1133, a22
′ or a33
′′
Vanishes. We can modify by rearranging the rows so that the pivot is non-zero.

Example: Solve by Gauss elimination Method


2 x + y + z = 10
3 x + 2 y + 3 z = 18
Compiled
x + 4by +
www.studyeasy.in
9 z = 16
Compiled by www.studyeasy.in

Solution: The augmented matrix is


 2 1 1 10 

  3 1
 3 2 3 18 R = R − R ; R = R − R
 2 2 2 1 3 3 2 1
 


1 4 9 16 

Eliminating x from second and third equation

The augmented matrix is  


 
 
 2 1 1 10 
~  3 3 3 3 
 3− 2 2− 1 3− 1 18 − 10 

.in
 
 2 2 2 2 
 1 1 1 1 
 1− 2 4− 1 9− 1 16 − 10 
 
 2 2 2 2 




2 1 1
sy
10



 
ea
 
 1 3 

0 3 
 2 2 
 7 17 
 0 11 
y

 
~ 2 2 
ud

Eliminating y from the third equation R3=R3-7R2

 2 1 1 10  2 x + y + z =10
st


 1 3 
 0 3  1 y+3 z =3

 2 2 
2 2

 0 0 − 2 −10  − 2 z = 10

z = 5, y = -9, x = 7

Compiled by www.studyeasy.in
Compiled by www.studyeasy.in

Gauss-Jordan Method:
Instead of eliminating x2 only in the third equation, we could have eliminated it from the first
equation also, so that at the end of the second stage, the augmented matrix becomes.

Gauss-Jordan Method: consider

a11 x1 + a12 x2 + a13 x3 = b1


a21x1 + a22 x2 + a23 x3 = b2 (1)
a31x1 + a32 x2 + a33 x3 = b3

The augmented matrix of the above system



(2)
 a a a b 
 11 12 13 1

 a a a b 
 21 22 23 2
 a a a b 

.in
 31 32 33 3
 
 
 
 
 a a a b 





11
0
12
a′
22
a′
13
a′
23
a′
sy 1
b′
2
a′





 
ea
 0 a ′ − 32 a ′ a ′ − 32 a ′ b ′ − 32 b ′ 


32 a ′ 22 33 a ′ 23 3 a′ 2 

 22 22 22 
y


 a′ 0 a′ b ′ 
 11 13 1
ud


 0 a′ a′ b ′ 
 22 23 2
 0 0 a′ b ′ 

 33 3
st

This modification of Gaussian elimination is called the Gauss-Jordan method

Example: Solve by Gauss-Jordan method

2x + y + z =10
3x + 2y + 3z =18
x + 4y +9z =16

Solution: The augmented matrix is


 2 1 1 10 

  3 1
 3 2 3 18  R = R − R ; R = R − R
 2 2 2 1 3 3 2 1
 


1 4 9 16 

Compiled by www.studyeasy.in
Compiled by www.studyeasy.in

Eliminating x from second and third equation


 
 
 
 2 1 1 10 
 
 3 3 3 3 3 1

3− 2 2− 1 3 − 1 18 − 10 i.e R = R − R ; R = R − R

 2 2 2 2 2 2 2 1 3

3 2 1
 1 1 1 1 
 1− 2 4− 1 9 − 1 16 − 10 
 
 2 2 2 2 

 
 
   
 2 1 1 10   2 0 −2 4 
   
 0
1 3 
3 R = R −2R ;R = R −7R 
1 3 
 1 1 2 3 3 2  
0 3 
 2 2   2 2 
 7 17   
 0 11   0 0 −2 − 10 
 
 2 2 

.in
2x -2z = 4
sy
ea
(½) y +(3/2) z = 3

-2z = -10 z = 5, y = -9, x = 7


y

Example :Solve the following system of linear equations


x + 2y + z = 8
ud

2x + 3y + 4z = 20
4x + 3y + 2z = 16 Using Gauss-Jordan method.
Solution: Let the given system of equations be written in the matrix form AX= B where
st


1 2 1 x 8
    
   
A= 2 3 4 ,X= y andB= 20

     
   
4 3 2
 z
 
16
The augmented matrix is

1 2 1 8 

 
 A: B = 2 3 4 20

   
 
4 3 2 16
 
1 2 1 8 

 
 A: B = 2 3 4 20
 R =R - 2R R =R − 4R
  
2 2 1 3 3 1
 
4 3 2 16
 

Compiled by www.studyeasy.in
Compiled by www.studyeasy.in

1 2 1 8 
  R 3 = R 3 − 5R2
 
 0 −1 2 4 
 
 
 0 − 5 − 2 -16 

1 2 1 8 
 
  R +1/12R R +1/ 6R
0 −1 2 4  1 3 2 3
 
 
0
 0 −12 -36 

1 2 0 5 
  R = R + 2R
  1 1 2

.in
0 −1 0 -2 
 
 
0 0
 −12 -36 
sy
1 0 0 1 
ea
 
 
0 −1 0 -2 
 
y

 
0 0
 −12 -36 
ud

We get the following system of equations X = 1, y = 2 and 12z = 36


st

Then required solution is x =1 , y = 2, z =3

Compiled by www.studyeasy.in
Compiled by www.studyeasy.in

Unit-VIII: Linear Algebra-II

Linear Transformations: Definition:


A transformation ( or function or mapping):
A transformation T from Rn to Rm is a rule that assigns to each vector x in Rn a vector T(x) in Rm.
The set Rn is called the domain of T, and Rm is called the co-domain of T. The notation T:Rn Rm indicates
that the domain of T in Rn and the co-domain is Rm.
For x in Rn, the vector T(x) in Rm is called the image of x (under the action of T).
The set of all images T(x) is called the range of T.

For example: AX = b and Au = o


1  
 
1 
   
 4 − 3 1 3 1 5   4 −3 1 3   4   0 
 =        = 
         
  2 0 5 1  -1 0 
 2 0 5 1 1 8  
   

.in
   
1
 
 3 
and
Say that multiplication by A transforms X into b and Similarly multiplication by A transforms u into the o
(zero) vector. sy
y ea
ud
st

Matrix Transformation : T:Rn Rm for each x in Rn, T(x) is computed as A(x), where A is an m x n
matrix, For simplicity, we sometimes denote such a matrix transformation by x Ax.
 1 − 3   3 
  
 2 
Example: Let A =  3    
5  , u =  , b =  2 

   − 1  
  
 −1 7   
 − 5 


 1 − 3 x − 3x 
  x   1 2
And define a transformation T:R2 R3 by T(x) = Ax so that T(x)= A(x)=

 3 5


 1  =  3x + 5 x 

   x   1 2

−1 7   2  − x + 7 x 
 
 1 2

Find T(u), the image of u under the transformation T.

Compiled by www.studyeasy.in
Compiled by www.studyeasy.in


1 − 3  5 
  
  2   
T (u) = Au = 3 5    =  1 

 −1
   
−1 7    − 9



Definition: A transformation ( or mapping ) T is linear if


i) T(u + v) = T(u) + T(v) for all u, in the domain of T
ii) T(cu)=c T(u) for all u and all scalars c.

Characteristic Equation of a Matrix :


If A = (aij) is any square matrix of order n and λ be any variable, then the matrix [ A- λI] is called the
characteristic matrix of A. The determinant of this matrix i.e. Is called the characteristic function.
The equation =0 is the characteristic equation of A and of degree n. The roots of the equation
= 0 are called the characteristics or Eigen values of A.
Example: Determine the characteristic roots of 2 2 1 

.in
 
A =  1 3 1 
 
1 2 2 

Solution: The characteristic equation is A − λI


sy =0
ea
iE

2−λ 2 1
y

1 3− λ 1 =0
− λ 3 + 7 λ 2 − 11 λ + 5 = 0
ud

1 2 2−λ
By synthetic division method
1) -1 7 -11 5
st

-1 6 -5
-1 6 5 0

-λ2+6 λ-5 = 0 (-λ+1)(λ-5)=0


λ1= 1, λ2 =1 λ3 = 5 are the require roots or Eigen values
5 4
Example: Find the eigen values and the eigenvector of the matrix A =
1 2
Solution: The characteristic equation of A is A − λ I =0
5−λ 4
i.e
1 2−λ
= 0 expanding we get λ 2 − 7λ + 6 = 0
The roots of this equation are, λ = 6 λ = 1 which are the eigen values of A.
To find the eigen vectors, consider the matrix equation is

Compiled by www.studyeasy.in
Compiled by www.studyeasy.in

x 


A − λ I  X = 0 where X =  1 
x 
 2
  x   

5−λ 4   1  0 
i.e     = 
 1 2 − λ   x 2  0 

This gives 2 homogeneous linear equations

(5-λ) x1 +4x2 = 0
x1+(2-λ)x2 =0
i) For λ = 6 above equations becomes
-x1 + 4x2 = 0
x1- 4x2 =0 Both represent the same equation x1-4x2 =0 x x
∴ 1= 2
4 4 1
giving the eigenvector

.in
X =  
1 1

ii) Again for λ = 1,


4x1 + 4x2 = 0
sy
x x
∴ 1= 2
ea
x1 + x2 = 0 Both represent the same x1 + x2 = 0 1 −1
giving the eigenvector
1
X =  
y

2 −1
ud

 8 −6 2 
Example: Find the eigen value and the eigen vectors of the matrix  
st

 
 −6 7 −4 
 
 

2 −4 3 

8 −6 2 

Solution:  
Let A = − 6
 7 − 4
 

2 −4 3 

The characteristic equation of the matrix A is A − λI = 0

Compiled by www.studyeasy.in
Compiled by www.studyeasy.in

 8−λ −6 2 
 
 
 −6 7−λ −4 =0 
 
 
 2 −4 3−λ 
i.e

Expanding, we get λ3-18λ2+45λ=0 : λ3-18λ2+45λ=0 λ(λ2-18λ+45) = 0

i.e λ = 0 or (λ2-18λ+45) = 0

i.e λ = 0 or (λ -3) (λ -15)=0

i.e λ1 = 0 , λ2 = 3, λ3 = 15 These are the eigen values of A

To find the eigen vector consider the matrix equation is

.in

x1  
 
 A − λ I  X = 0 .whereX = x 
  2
x 3 




sy
 
ea
i.e
   
8− λ −6 2 x 0
y

  1 

    
ud

 −6 7−λ −4   
 2 
x = 0
  
    
 2 −4 3− λ x  
 0
 3   
st

This gives the three homogeneous equations


(8-λ)x1 - 6x2+ 2x3 = 0
-6x1 + (7-λ)x2-4x3 = 0 (1)
2x1 - 4x2 + (3-λ)x3 = 0

i)λ1 = 0 equation (1) become


8x1 - 6x2+ 2x3 = 0
-6x1 + 7x2-4x3 = 0 (2)
2x1 - 4x2 + 3x3 = 0
Solving any two non-identical equations of (2) for x1, x2, x3 by the rule of cross multiplication. We get

x
1 =
−x
2 =
x
3
x
1=
−x x
2 = 3 x −x x
24 − 14 − 32 + 12 56 − 36 10 20 20 1= 2 = 3
1 2 2

Compiled by www.studyeasy.in
Compiled by www.studyeasy.in

1 


 
x = 2

1  

2

Therefore the eigenvector corresponding to λ1 = 0 is

ii)λ2 = 3 equation (1) become


5x1 - 6x2+ 2x3 = 0
-6x1 + 4x2-4x3 = 0 (3)
2x1 - 4x2 + 0x3 = 0
Solving by the rule of cross multiplication We get
x x x
1= 2 = 3 2  
2 1 −2 
 
Therefore the eigenvector corresponding to λ2= 3 is x = 1 
2  

.in

−2 

iii) λ3 = 15 equation (1) become


-7x1 - 6x2+ 2x3 = 0
sy
-6x1 - 8x2-4x3 = 0 (3)
ea
2x1 - 4x2 - 12x3 = 0
Solving by the rule of cross multiplication We get

x x x
y

1= 2 = 3 2 
2 −2 1 
ud



x = −2 
Therefore the eigenvector corresponding to λ3= 15 is 3  

1 

st

 Orthogonal Matrix:
A square matrix A is said to be orthogonal if AA’=I=A’A
 Singular matrix and Nonsingular matrix:
A square matrix A is said to be singular if its determinant is zero and A is said to be nonsingular if its
determinant is not equal to zero.
 Inverse of a square matrix:
If A and B are two square matrices of the same order such that AB-BA = I then B is called the inverse of A
denoted by A-1.
 Similarity of Matrices:
Two square matrices A and B of the same order are said to be similar if there exists a non singular
matrix P such that B= P-1AP
Here B is said to be similar to A.

Compiled by www.studyeasy.in
Compiled by www.studyeasy.in

Example: 
 −1 3
Reduce the matrix A=   to the diagonal form .
 
 −2 4 

Solution: The characteristic equation of A is A − λI = 0


−1 − λ 3 
 =0⇒ λ
i.e  2 − 3λ + 2 = 0 ⇒ λ = 1 and 2
 
 − 2 4 − λ 

Are the eigen values of A.

     
 A − λ I  
 
X 

=  0 
Consider   

−1−λ 3  x 0


.in

i.e    =  

−2 4−λ y 0

i.e -2x+(4-λ)y=0 sy
Casei) let λ=1, we get -2x+3y=0 or 2x=3y or x/3 = y/2
ea
X1 = (3,2)’ is the eigen vector corresponding to λ=1
Caseii) let λ=2,
we get -3x +3y = 0 or x = y or x/1 = y/1
X2 = (1,1)’ is the eigen vector corresponding to λ=2.
y


1 
ud

   3
P =  X X 

=  
 1 2  
 2 1 
we have P = 1 and p - 1 = 1  AdjP 

st


p

1 1 − 1
P = 
− 

 − 2 3 

-1  1 − 1  − 1 3   3 1  1 − 1  3 2  1 0 
p AP =    =   =  
       
 − 2 3   − 2 4   2 1  − 2 3   2 2  0 2 

-1 1 0 
Thus p AP = D =   is the diagonal matrix
 
 0 2 

Compiled by www.studyeasy.in
Compiled by www.studyeasy.in

Reduce the following matrices into diagonal matrices


11 −4 −7 
0 0 0

   
   
A= 7 −2 −5
  Ans.: 0 1 0
 
   
   


10 −4 −6 

0 0 2





 8 −6 2  
0 0 0 

   
   
A= −6 7 −4
  Ans.: 0 3 0
 
   
   
2 −4 3






0 0 15 

.in
sy
y ea
ud
st

Compiled by www.studyeasy.in

You might also like